You are on page 1of 179

11

General
Mathematics
First Quarter

LEARNING ACTIVITY SHEETS

i
Note: Practice Personal Hygiene protocols at all times
COPYRIGHT PAGE
Learning Activity Sheet in General Mathematics
Grade 11

Copyright @ 2020

DEPARTMENT OF EDUCATION
Regional Office No. 02 (Cagayan Valley)
Regional Government Center, Carig Sur, Tuguegarao City, 3500

“No copy of this material shall subsist in any work of the government of the Philippines. However, prior
approval of the government agency or office wherein the work is created shall be necessary for
exploitation of such work for profit.

This material has been developed for the implementation of K to 12 Curriculum through the Curriculum
and Learning Management Division (CLMD). It can be reproduced for educational purposes and the
source must be acknowledged. Derivatives of the work including creating an edited version and
enhancement of supplementary work are permitted provided all original works are acknowledged and
the copyright is attributed. No work may be derived from the material for commercial purpose and profit.

Consultants:
Regional Director : ESTELA L. CARIÑO, EdD, CESO
Assistant Regional Director : RHODA T. RAZON, EdD, CESO V
Schools Division Superintendent : MADELYN L. MACALLING, PhD, CESO VI
Assistant Schools Division Superintendents
: DANTE MARCELO, PhD, CESO VI
: EDNA P. ABUAN, PhD
Chief Education Supervisor, CLMD : OCTAVIO V. CABASAG, PhD
Chief Education Supervisor, CID : RODRIGO V. PASCUA, EdD

Development Team
Writers : ALJON S. BUCU, PhD, REGIONAL SCIENCE HS- ISABELA
MAI RANI ZIPAGAN, PhD, GAMU RURAL HS-ISABELA
JONEL ASUNCION, LUNA NATIONAL HS-ISABELA
NENITA MARQUEZ, TUMAUINI NATIONAL HS-ISABLEA
MARYJANE BUCAG, SANTO TOMAS NATIONAL HS-ISABLEA
CAYSELYN GUITERING, ALFREDA ALBANO NHS-ISABELA
CORAZON BAUTISTA, LUNA NATIONAL HS-ISABELA
CINDY LACANARIA, LUNA GENERAL COMPREHENSIVE HS-ISABELA
JACKSON MICHAEL GAMAYON, RAGAN SUR NATIONAL HS-ISABELA
LEONOR BALICAO, DELFIN ALBANO STAND ALONE SHS-ISABELA
JAYBEL B. CALUMPIT, REGIONAL SCIENCE HS- ISABELA
JEREMAEH C. LOZANO, JONES RURAL SCHOOL-ISABELA
CHRISTIAN JULIAN, ROXAS NATIONAL HS-ISABELA

Content Editors : ALJON S. BUCU, PhD


MAI RANI ZIPAGAN, PhD
LEONOR BALICAO, DOMINGO PEROCHO, PhD.
JUAN LAPPAY, LYNDON LAZARO
Focal Persons : INOCENCIO T. BALAG, Division EPS- Mathematics
MA. CRISTINA ACOSTA, Division EPS- LRMDS, SDO ISABELA
ISAGANI R. DURUIN, PhD., Regional EPS- Mathematics
RIZALINO CARONAN, Regional EPS- LRMDS
Printed in DepEd Regional Office No. 02
Regional Government Center, Carig Sur, Tuguegarao City

ii
Note: Practice Personal Hygiene protocols at all times
Table of Contents
Learning Competency Page Number

Represents real-life situations using functions, including piecewise


functions 1
Evaluates a function 9
Performs addition, subtraction, multiplication, division, and
composition of functions 14
Solves problems involving functions 20
Represents real-life situations using rational functions 26
Distinguishes rational function, rational equation, and rational
Inequality 33
Solves rational equations and inequalities 38
Represents a rational function through its: a) table of values, b) graph,
c) equation. 51
Finds the domain and range of a rational function 56
Determines the (a) intercepts; (b) zeroes (c) asymptotes of rational
Functions 61
Solve Problems Involving Rational Functions, Equations and Inequalities 66
Represents real-life situations using one-to-one functions 72
Determines the inverse of a one-to-one function 79
Represents an inverse function through its (a) table of values, and
(b) graph and solves problems involving inverse functions 87
Represents an exponential function through its: (a) table of values,
(b) graph and (c) equation 93
Represents real-life situations using exponential functions 100
Distinguishes between exponential function, exponential equation,
and exponential inequality 106
Solves exponential equations and inequalities 110
Finds the domain and range of an exponential function 119
Determines the intercepts, zeroes, and asymptotes function 128
Represents real-life situations using logarithmic functions 139
Distinguishes logarithmic function, logarithmic equation, and logarithmic
Inequality 144
Solves logarithmic equations and inequalities 149
Determines the intercepts, zeroes, and asymptotes of
logarithmic functions and Represents a logarithmic function through
its: (a) table of values, (b) graph and (c) equation 155
Determine the intercepts, zeroes and asymptotes of logarithmic functions 163
Solves problems involving logarithmic functions, equations, and
Inequalities 172

iii
Note: Practice Personal Hygiene protocols at all times
GENERAL MATHEMATICS 11
Name of Learner: _____________________ Grade Level: __________________
Section: _____________________________ Date: ________________________

LEARNING ACTIVITY SHEET


REPRESENTS REAL-LIFE SITUATIONS USING FUNCTIONS, INCLUDING
PIECEWISE FUNCTIONS

Background Information for Learners


After learning about linear functions, quadratic functions and polynomial functions, this
time, we shall consider functions in general. The phrase “is a function of” is used to describe a
situation or relationship between two variables. Thus, in describing this relationship, it is
natural to represent it using ordered pairs.

A relation is a set of ordered pairs (𝑥, 𝑦). The set of all first entries of these pairs is called
the domain of the relation while the set of all second entries of these pairs is called the range
of the relation.
A function is a relation that assigns to each element of the domain exactly one element of
the range.

The illustration below shows a function 𝑓 mapping, relating or pairing a domain element 𝑥 of
the set D to range element 𝑓(𝑥) read as “𝑓 of 𝑥” of the set R.

𝑥 𝑓(𝑥)

𝐷 𝑅

1
Note: Practice Personal Hygiene protocols at all times
Example 1.
Given the set of ordered pairs {(1, 2), (3, 4), (5, 6), (5, 7)} , determine the domain and range.
Answer:
By definition, the domain is set of all elements of x coordinate, thus, the domain is {1, 3, 5},
whereas its range is the set of all elements of y coordinate, hence, the range is {2, 4, 6, 7}.

Example 2.
Decide whether the following relations are functions
𝑓 = {(−1, 2), (4, 3), (−2, 3), (5, 8)}
𝑔 = {(𝑎, 𝑏), (𝑐, 𝑑 ), (𝑑, 𝑒), (𝑓, 𝑔)}
ℎ = {(1, 𝑎), (2, 𝑏), (3, 𝑐 ), (4, 𝑑 )}
𝑠 = {(4, 5), (4, −2), (−4, 5), (1, 7)}
𝑝 = {(1, 5), (2, 10), (3, 15), … , (𝑛, 5𝑛)}

Answer:
Only relation 𝑠 is not a function since the element 4 of the domain is mapped to two elements
of the range. Relations 𝑓, 𝑔, ℎ 𝑎𝑛𝑑 𝑝 are functions because no two ordered pairs have the same
domain but different range.

The Vertical Line Test


A graph is said to be a function if and only if each vertical line intersects the graph at exactly
one point.

Example 3.
Tell whether the graph of each relation is the graph of a function.
1. 2.

2
Note: Practice Personal Hygiene protocols at all times
3. 4.

5.

Answer:
The only function from the relations above are numbers 1, 4 and 5 while 2 and 3 are not since
they intersect the graph at more than one point.

Example 4.
State a function P that will describe the total distance of a student when he runs 5 km per day
at the end of 5 days.
Answer:
Since each day makes him 5 km, then the distance function is 𝑃 (𝑥 ) = 5𝑥.

Example 5.

3
Note: Practice Personal Hygiene protocols at all times
In a certain city, the Philippine Statistics Office has recorded a total population of 680, 000 in
the year 2018. The population increases at the rate of 0.25% annually. Determine an equation
that represents the population with respect to the number of years after 2018. Suppose the rate
of increase is constant.
Answer:
The equation will be of the form, 𝑃 = 𝑎(1 + 𝑟)𝑛 such that a is the initial population, r is the
rate of increase and n is the time in years and p is the population. Hence, the we have the
function 𝑃 = 680, 000(1.0025)𝑛 .

A piecewise function is a function that contains at least two equations “pieces” each of which
depends on the value of the independent variable or the domain.

Example 6.
Driving lessons require a rental car fee of P 500. 00 for the first 8 km. and for every kilometer
added charges an additional fee of P 50.00. Express a piecewise function for the problem.

Answer:

a. 𝑓 (𝑥 ) =
{ 500 ,
500 + 50(𝑥 − 8),
0<𝑥≤8
𝑥>8

Example 7.
An online seller charges a certain amount for the shipping fee of purchased products/items. For
orders 10 or fewer items, she charges P20.00 each, P15.00 per item for orders of 20 or fewer
but more than 10 items, and P10.00 per item for orders of more than 20 items. Write a function
representing the cost 𝑓 for the number of 𝑥 items/products.
Answer:
a. 𝑓 (𝑥 ) =
{ 20𝑥
15𝑥
10𝑥
, 0 < 𝑥 ≤ 10
, 11 < 𝑥 ≤ 20
, 𝑥 > 20
Learning Competency
Represents real-life situations using functions including piecewise functions (GM_M11GM-
Ia-1)

4
Note: Practice Personal Hygiene protocols at all times
EXERCISE 1
Directions: Determine the domain and range of the given relation and decide whether the
relation is function. [3 points each]
1. {(4, 2), (5, 0), (−2, 6), (0, 1)} 6. {(0.02, 0.002), (0.20, 0.02 )}
1 3
2. {(−8, 2), ( , )} 7. {(5,2), (−1, 5), (5, 6)}
2 5
3. {(7, 2), (7, −2), (7, 11), (10, −3)} 8. {(1, 6), (2, 12), (3, 18), … (𝑛, 6𝑛)}
4. {(𝑤, 𝑠)| 𝑠 2 = 𝑤} 9. {(𝑥, 𝑦)|𝑦 4 = 𝑥}
5. {(𝑥, 𝑦)|𝑦 = 𝑥 2 − 2𝑥 + 1} 10. {(𝑐, 1), (𝑐, 2), (𝑐, 3), (𝑐, 4)}

EXERCISE 2
Directions: Identify the domain for each relation using the set builder notation.
[2 points each]
1. 𝑦 = 5𝑥 − 4 6. 𝑔(𝑥) = ⟦𝑥⟧ − 𝑥
3 2
2. 𝑦 = 𝑥 3 + 2𝑥 2 + 𝑥 + 7 7. 0 = 4 𝑦 + 6𝑥
3. 𝑓(𝑥) = |3𝑥 + 2| 8. 6𝑥 2 = 15𝑦
𝑥 2 +4
4. 𝑥 2 + 𝑦 2 = 16 9. 𝑓 (𝑥 ) =
(3𝑥+5)(𝑥+8)

{
5. 𝑓(𝑥) = √𝑥 2 − 24𝑥 + 144 10. 𝑔(𝑥) = 𝑥 − 1 , 𝑥<3
5 , 𝑥=3
2𝑥 + 1 , 𝑥 > 3

EXERCISE 3
Directions: Give a function that represents each situation. [2 points each]
1. Mang Ambo, a mango farmer, sells ripe mangoes either per piece or bulk. He sells mangoes
at P10.00 each for orders less than 50 pieces and P450.00 for a bulk of 50 pieces and P8.00
for each excess mango after that. Write the required piecewise function.

2. A certain Liquor is sold for P150.00 each. With an increasing public demand, a vendor
decides to sell it for P140.00 each if someone buys more than 15 bottles. Express the cost
with respect to the number of bottled liquors sold.

3. A rectangular box is to be made from a piece of cardboard 20 cm long and 8 cm wide by


cutting out identical squares with side x from the four corners and turning up the sides.
Define a function representing the volume of the box.

4. A horseback riding charges P50.00 for the first 300 meters and additional P10.00 for a ride
greater than 300 meters. Express the function describing the amount of horseback riding.

5
Note: Practice Personal Hygiene protocols at all times
5. Rental car charges P100.00 for the first three hours and an excess of P20.00 for each hour
(or a fraction of it) after that. If you rent a car for more than ten hours, a fee of P500.00
shall be charged. Represent the rental car fee in piecewise function.

Reflection:
What have you learned from this topic?
___________________________________________________________________________
___________________________________________________________________________
___________________________________________________________________________
_______________________________________________________

References:
Verzosa, D.B, et.al (2016). General Mathematics. Quezon City, Manila
Alferez, G. S. (2014). Introduction to Calculus. Quezon City, Manila
Leithold, L. (1996). The Calculus 7. New York City.

6
Note: Practice Personal Hygiene protocols at all times
Answer Key
Exercise 1

1. D: {4, 5, -2, 0} 6. D: {0.02, 0.20}


R: {2, 0, 6, 1} R: {0.002, 0. 02}
Function Function
2. D: {-8, ½} 7. D: {5, -1}
R: {2, 3/5} R: {2, 5, 6}
Function Not Function
3. D: {7, 10} 8. D: {1, 2, 3, …n}
R: {2, -2, 11, -3} R: {6, 12, 18, …, 6n}
Not function Function
4. D: 𝑥 ≥ 0 9. D: 𝑥 ≥ 0

R: 𝐴𝑙𝑙 𝑅𝑒𝑎𝑙𝑠 R: 𝐴𝑙𝑙 𝑅𝑒𝑎𝑙𝑠


Not a function Not a function
5. D: 𝐴𝑙𝑙 𝑅𝑒𝑎𝑙𝑠 10. D: {c}
R: 𝑦 ≥ 0 R: {1, 2, 3, 4}
Function Not Function

Exercise 2

1. {𝑥: 𝑥 ∈ ℝ} 6. {𝑥: 𝑥 ∈ ℝ}
2. {𝑥: 𝑥 ∈ ℝ} 7. {𝑥: 𝑥 ∈ ℝ, 𝑥 ≤ 0}
3. {𝑥: 𝑥 ∈ ℝ} 8. {𝑥: 𝑥 ∈ ℝ}
5
4. {𝑥: 𝑥 ∈ ℝ, −4 ≤ x ≤ 4} 9. {𝑥: 𝑥 ∈ ℝ, x ≠ − 3 𝑎𝑛𝑑 − 8}
5. {𝑥: 𝑥 ∈ ℝ} 10. {𝑥: 𝑥 ∈ ℝ}

Exercise 3

1. 10𝑥 , 0 < 𝑥 < 49

{
𝑓(𝑥) = 450
450 + 8(𝑥 − 50)
, 𝑥 = 50
, 𝑥 > 50

2.
{
𝑓(𝑥) =
150 𝑥
140𝑥
, 0 < 𝑥 ≤ 15
, 𝑥 ≥ 15

3. 𝑉(𝑥 ) = 160𝑥 − 48𝑥 2 + 4𝑥 3

4.
𝑓(𝑥) =
{50
50 + 10(𝑥 − 300)
, 0 < 𝑥 ≤ 300
, 𝑥 > 300

7
Note: Practice Personal Hygiene protocols at all times
100 ,0 < 𝑥 ≤ 3
{
5.
𝑓(𝑥) = 100 + 20⌈𝑥 − 3⌉ ,3 < 𝑥 ≤ 10
500 , 𝑥 > 10

8
Note: Practice Personal Hygiene protocols at all times
GENERAL MATHEMATICS 11
Name of Learner: _____________________ Grade Level: __________________
Section: _____________________________ Date: ________________________

LEARNING ACTIVITY SHEET


EVALUATES A FUNCTION

Background Information for the Learners

In this topic, you will learn about evaluating functions. To evaluate a function is
simply to substitute/replace a certain variable with a given number or expression, in this case
we refer this as the x variable. We then write 𝑓(𝑎) to show that 𝑓 is being evaluated at 𝑎 for
some 𝑎 in the domain of 𝑓.

Example 1.
Evaluate the given functions at 𝑥 = 2
1. 𝑓 (𝑥 ) = 5𝑥 + 8
2. 𝑔(𝑥 ) = 𝑥 2 + 3𝑥 − 4
3. ℎ(𝑥 ) = |𝑥 + 3| − 2
4. 𝑡(𝑥 ) = 𝑥 3 − 5𝑥 2
5. 𝑝(𝑥 ) = ⌈𝑥 ⌉ + 5, where ⌈𝑥 ⌉ is the smallest integer greater than or equal x.
Solution:
1. 𝑓 (2) = 5(2) + 8 = 10 + 8 = 18
2. 𝑔(2) = (2)2 + 3(2) − 4 = 4 + 6 − 4 = 6
3. ℎ(2) = |2 + 3| − 2 = |5| − 2 = 5 − 2 = 3
4. 𝑡(2) = (2)3 − 5(2)2 = 8 − 20 = −12
5. 𝑝(2) = ⌈2⌉ + 5 = 2 + 5 = 7

Learning Competency
Evaluates a Function (GM_M11GM-Ia-2)

9
Note: Practice Personal Hygiene protocols at all times
EXERCISE 1
Directions: Evaluate the following functions at 𝑥 = 3.5. [1 point each]
1. 𝑓 (𝑥 ) = 7𝑥 − 1
2. 𝑔(𝑥 ) = 𝑥 2 − 4
3. ℎ(𝑥 ) = |4𝑥 − 21| − 2
4. 𝑡(𝑥 ) = 2 • 3𝑥−1.5
5. 𝑠(𝑥 ) = √𝑥 2 − 2𝑥 + 4
6. 𝑟(𝑥 ) = −2−2𝑥
7. 𝑤 (𝑥 ) = 32𝑥−5
8. 𝑞(𝑥 ) = ⌊𝑥 ⌋ + 4
𝑥+4
9. 𝑓 (𝑥 ) = 𝑥2 −2

10. 5𝑥 ,𝑥 < 2
{
𝑓(𝑥) = 𝑥 2 − 1
𝑥3 + 2
,2 ≤ 𝑥 < 5
,𝑥 ≥ 5

EXERCISE 2
Directions: Evaluate each function. [1 point each]
1. 𝑓 (𝑥 ) = |𝑥 + 2| + 4, find 𝑓(−8)
2. ℎ(𝑥 ) = 33𝑥−2 , find ℎ(2)
1
3. 𝑞(𝑥 ) = 43𝑥 , 𝑓𝑖𝑛𝑑 𝑞(2)

For numbers 4-5, use the function below


𝑓 (𝑥 ) = 𝑥 2 + 5𝑥 + 4
4. Find 𝑓 (2)
5. Find 𝑓 (𝑥 + 2)
For numbers 6-8, use the function below

{
𝑥+2 , 𝑥 ≤ −4
𝑓(𝑥) = √16 − 𝑥 2 , −4 < 𝑥 < 4
2−𝑥 ,𝑥 ≥ 4
6. Find 𝑓 (−5)
7. Find 𝑓 (2)
8. Find 𝑓 (5)

10
Note: Practice Personal Hygiene protocols at all times
For items 9-10, given 𝑓(𝑥 ) = √𝑥 + 4,
9. Find 𝑓 (𝑥 2 − 4)
𝑓(𝑥+ℎ)−𝑓(𝑥)
10. Find ,ℎ ≠ 0

EXERCISE 3
Directions: Solve the given problems [3 points each]
1. An internet café charges P25.00 per hour (or a fraction of an hour) for the first four
hours and an extra of P15.00 per hour for each next hour. What is the amount paid for
following time?
a. 2 hrs.
b. 3. 5 hrs.
c. 6 hrs.
d. 7. 2 hrs.

2. In a certain city, the Philippine Statistics Office has recorded a total population of 680,
000 in the year 2018. The population increases at the rate of 0.25% annually. Suppose
the rate of increase is constant. Determine the population in the following years.
a. 10 yrs.
b. 14 yrs.
c. 17 yrs.

3. Driving lessons require a rental car fee of P 500. 00 for the first 8 km. and for every
kilometer added charges an additional fee of P 50.00. How much will it cost for the
following distance.
a. 7 km.
b. 9 km.

4. A stone is thrown straight up from a building with a velocity, in m/s, given by 𝑉 (𝑡) =
15 − 9.8𝑡. After t seconds, calculate the following velocities. Make an interpretation of
the results.
a. 𝑉(0)
b. 𝑉(1)

5. In particular bacteria culture, if 𝑝(𝑥) bacteria are present at 𝑥 minutes, then


𝑝(𝑥 ) = 𝐴𝑒 0.03𝑥 such that 𝐴 is a constant. Initially, there are 1200 bacteria present,
calculate the number of bacteria after:
a. 30 minutes
b. 1 hr.
c. 1.5 hrs.

11
Note: Practice Personal Hygiene protocols at all times
Reflection:
What have you learned from this topic?
________________________________________________________________________
________________________________________________________________________
________________________________________________________________________
____________________________________________________
References:
Verzosa, D.B, et.al (2016). General Mathematics. Quezon City, Manila
Alferez, G. S. (2014). Introduction to Calculus. Quezon City, Manila
Leithold, L. (1996). The Calculus 7. New York City.
https://cdn.kutasoftware.com/Worksheets/Alg2/Evaluating%20Functions.pdf

12
Note: Practice Personal Hygiene protocols at all times
Answer Key
Exercise 1

1
1. 23.5 6. − 128
2. 8.25 7. 9
3. 5 8. 7
4. 18 9. 0.73
5. 9.25 10. 11.25

Exercise 2
1. 10 6. −3
2. 8 7. 2√3
3. 8 8. −3
4. 18 9. 𝑥
1
5. 𝑥 2 + 9𝑥 + 18 10.
√𝑥+ℎ+4 + √𝑥+4

Exercise 3
1. a. P50.00
b. P100.00
c. P130.00
d. P160.00

2. a. P = 697, 193
b. P = 704, 191
c. P = 709, 485
3. a. P500.00
b. P550.00

4. a. 15 m/s
b. 5.2 m/s
This implies that the initial velocity is 15 m/s and gets more slowly at 5.2 m/s.
5. a. P(x) = 2, 952
b. P(x) = 7, 260
c. P(x) = 17, 856

13
Note: Practice Personal Hygiene protocols at all times
GENERAL MATHEMATICS 11
Name of Learner: _____________________ Grade Level: __________________
Section: _____________________________ Date: ________________________

LEARNING ACTIVITY SHEET


PERFORMS ADDITION, SUBTRACTION, MULTIPLICATION, DIVISION, AND
COMPOSITION OF FUNCTIONS

Background Information for the Learners

OPERATIONS ON FUNCTIONS
Given two functions 𝑓 and 𝑔, then:
Sum of 𝑓 and 𝑔: (𝑓 + 𝑔)(𝑥 ) = 𝑓 (𝑥 ) + 𝑔(𝑥 )

Difference of 𝑓 and 𝑔: (𝑓 − 𝑔)(𝑥 ) = 𝑓 (𝑥 ) − 𝑔(𝑥)

Product of 𝑓 and 𝑔: (𝑓 • 𝑔)(𝑥 ) = 𝑓(𝑥) • 𝑔(𝑥)


𝑓 𝑓 (𝑥 )
Quotient of 𝑓 and 𝑔: ቀ𝑔ቁ (𝑥) = 𝑔(𝑥), where 𝑔(𝑥) ≠ 0

Each of the four operations is defined for all 𝑥 in the domains of both 𝑓 and 𝑔, with the
exception that in quotient, we exclude the values of 𝑥 for which 𝑔(𝑥 ) = 0.

Example 1.
Given the two functions, 𝑓 (𝑥 ) = 𝑥 2 − 1 and 𝑔(𝑥 ) = 𝑥 2 − 𝑥. Compute 𝑓 (𝑥 ) + 𝑔(𝑥 ), 𝑓 (𝑥 ) −
𝑓 (𝑥 )
𝑔(𝑥), 𝑓(𝑥) • 𝑔(𝑥) and . Determine the domain of each operation.
𝑔(𝑥)

Solution:
a. 𝑓(𝑥 ) + 𝑔(𝑥 ) = (𝑥 2 − 1) + (𝑥 2 − 𝑥) = 2𝑥 2 − 𝑥 − 1, D: {𝑥: 𝑥 ∈ ℝ}
b. 𝑓 (𝑥 ) − 𝑔(𝑥 ) = (𝑥 2 − 1) − (𝑥 2 − 𝑥) = −1 + 𝑥, D: {𝑥: 𝑥 ∈ ℝ}
c. 𝑓(𝑥 ) • 𝑔(𝑥 ) = (𝑥 2 − 1)(𝑥 2 − 𝑥 ) = 𝑥 4 − 𝑥 3 − 𝑥 2 + 𝑥, D: {𝑥: 𝑥 ∈ ℝ}
𝑓 (𝑥 ) 𝑥 2 −1 (𝑥−1)(𝑥+1) 𝑥+ 1
d. = = = D: {𝑥: 𝑥 ∈ ℝ, 𝑥 ≠ 0, 1}
𝑔(𝑥) 𝑥 2 −𝑥 𝑥(𝑥−1) 𝑥

14
Note: Practice Personal Hygiene protocols at all times
THE COMPOSITE OF FUNCTIONS
The composite of 𝑓 and 𝑔, denoted by 𝑓 ∘ 𝑔, is defined by two conditions:
1. (𝒇 ∘ 𝒈)(𝒙) = 𝒇(𝒈(𝒙)), which is read as “𝒇 circle 𝒈 of 𝒙 equals 𝒇 of 𝒈 of 𝒙”.
2. 𝒙 is in the domain of 𝒈 and 𝒈(𝒙) is in the domain of 𝒇.
𝑔 𝑓

𝑥 𝑔(𝑥) 𝑓(𝑔(𝑥))

𝑓∘𝑔
The domain of 𝑓 ∘ 𝑔 is the set of x satisfying condition (2). The operation that combines
𝑓 𝑎𝑛𝑑 𝑔 to produce their composite is called function composition

Example 2.
Given 𝑓(𝑥 ) = 𝑥 2 + 3𝑥 − 4 and 𝑔(𝑥 ) = 𝑥 − 2, evaluate the following.
a. 𝑓(−1) b. 𝑔(4) c. 𝑓(𝑔(𝑥) d. 𝑔(𝑔(2) e. (𝑓 ∘ 𝑔)(2)

Solution:
a. 𝑓(−1) = (−1)2 + 3(−1) − 4 = 6
b. 𝑔(4) = 4 − 2 = 2
c. 𝑓(𝑔(𝑥 ) = 𝑓(𝑥 − 2)
= (𝑥 − 2)2 + 3(𝑥 − 2) − 4
= 𝑥 2 − 4𝑥 + 4 + 3𝑥 − 6 − 4
= 𝑥2 − 𝑥 − 6
d. 𝑔(𝑔(2) = 𝑔(2 − 2)
= 𝑔(0)
= 0 − 2 = −2
e. (𝑓 ∘ 𝑔)(2) = 𝑓 [𝑔(2)]

15
Note: Practice Personal Hygiene protocols at all times
= 𝑓 [2 − 2]
= 𝑓 [0]
= 02 + 3(0) − 4
= −4
Example 3.

Given the functions, 𝑓 (𝑥 ) = 2𝑥 − 1, 𝑔(𝑥 ) = √2𝑥 + 1, ℎ(𝑥 ) = ⌊𝑥 ⌋ + 1


a. Find and simplify (𝑔 ∘ 𝑓 )(𝑥 ).
b. Find and simplify (ℎ ∘ 𝑔)(10)
c. Find and simplify ℎ(2.1)[𝑓 (3) + 𝑔(4)]

Solution:

a. (𝑔 ∘ 𝑓 )(𝑥 ) = 𝑔(𝑓(𝑥 )) = √2𝑥 − 1 + 1 = √2𝑥

b. (ℎ ∘ 𝑔)(12) = ℎ(𝑔(10))

= ⌊𝑔(10)⌋ + 1

= ⌊√2(10) + 1⌋ + 1

= ⌊√21⌋ + 1

=4+1
=5

c. ℎ(2.1)[𝑓 (3) + 𝑔(4)] = [⌊2.1⌋ + 1][(2(3) − 1) + √2(4) + 1]


= [2 + 1][ 5 + 3]
= 24

Learning Competency
Performs addition, subtraction, multiplication, division, and composition of functions
(GM_M11GM-Ia-3)

EXERCISE 1
Directions: Perform the indication operation in the following functions. [1 point each]
A. Given the functions 𝑓(𝑥 ) = 3𝑥 + 4 and 𝑔(𝑥 ) = 3𝑥 2 , find:
1. (𝑓 + 𝑔)(𝑥 )

16
Note: Practice Personal Hygiene protocols at all times
2. (𝑓 − 𝑔)(𝑥 )
3. (𝑓𝑔)(𝑥 )
4. (𝑓 − 𝑔)(2)
5. (𝑓 + 𝑔)(−3)
𝑓
6. ቀ𝑔ቁ (𝑥)

1
B. Given 𝑓(𝑥 ) = 𝑥 2 + 1, 𝑔(𝑥 ) = 2 − 𝑥 and 𝑞(𝑥 ) =
𝑥 2 +4𝑥−3
7. (𝑓 + 𝑔 + 𝑞)(𝑥 )
8. (𝑞 − 𝑔)(−1)
9. (𝑓𝑔𝑞)(0)
𝑓𝑔
10. ቀ ቁ (2)
𝑞

EXERCISE 2
Directions: Solve the following functions [2 points each]
A. Let 𝑓 (𝑥 ) = 2𝑥 + 4, 𝑔(𝑥 ) = 𝑥 2 − 16 and ℎ(𝑥 ) = 𝑥 3 . Find:
1. (𝑓 ∘ 𝑔)(𝑥)
2. (ℎ ∘ 𝑔)(𝑥)

3. 𝑓(𝑓 ቀ𝑓(𝑓(5))ቁ)
1
B. Let 𝑓 (𝑥 ) = 𝑥 2 + 1, 𝑔(𝑥 ) = 𝑥 Find:

4. (𝑓 ∘ 𝑓)(𝑥)
5. (𝑔 ∘ 𝑔)(4)

EXERCISE 3
Directions: Perform the indicated conditions in each function. [2 points each]

A. Let 𝑓 (𝑥 ) = 𝑥 4 , 𝑔(𝑥 ) = √𝑥, ℎ(𝑥 ) = 𝑥 − 2 and 𝑝(𝑥 ) = 3𝑥. Express each function 𝑡 as a
composite of three of these four functions.
1. 𝑡(𝑥) = 3(𝑥 − 2)4
2. 𝑡(𝑥) = (3𝑥 − 6)4

3. 𝑡(𝑥 ) = √(𝑥 − 2)4

4. 𝑡(𝑥 ) = √𝑥 4 − 2

17
Note: Practice Personal Hygiene protocols at all times
5. 𝑡(𝑥 ) = (3𝑥 )2
𝑥+3
B. Let 𝑓 (𝑥 ) = 2𝑥 − 3, 𝑔(𝑥 ) = and ℎ(𝑥 ) = 3𝑥 + 2 [5 points each]
2

1. Show that 𝑓(𝑔(𝑥 )) = 𝑔(𝑓 (𝑥 )) for all 𝑥.

2. Show that 𝑓(ℎ(𝑥 )) ≠ ℎ(𝑓(𝑥 )) for any 𝑥.

Reflection:
Please share your insights in this topic.
________________________________________________________________________
________________________________________________________________________
________________________________________________________________________
____________________________________________________
References:
Verzosa, D.B, et.al (2016). General Mathematics. Quezon City, Manila
Alferez, G. S. (2014). Introduction to Calculus. Quezon City, Manila
Leithold, L. (1996). The Calculus 7. New York City.
Brown, R.G (1994). Advanced Mathematics, Precalculus with Discrete Mathematics and
Data Analysis, Houghton Mifflin, Boston.
Rolando, M.A, et.al (2002). Differential Calculus. Philippines.

18
Note: Practice Personal Hygiene protocols at all times
ANSWER KEY
EXERCISE 1
A. B.
1
1. 3𝑥 2 + 3𝑥 + 4 7. 𝑥 2 − 𝑥 + 3 + ቀ ቁ
𝑥 2 +4𝑥−3
7
2. −3𝑥 2 + 3𝑥 + 4 8. −
6
2
3. 9𝑥 3 + 12𝑥 2 9. −
3

4. −2 10. 0
5. 22
3𝑥+4
6.
3𝑥 2
EXERCISE 2
A. B.
1. 2𝑥 2 − 28 1. 𝑥 4 + 2𝑥 2 + 2
2. 𝑥 6 − 48𝑥 4 + 768𝑥 2 − 4096 2. 4
3. 140

EXERCISE 3
A. B.

1. 𝑝 ቀ𝑓(ℎ (𝑥 ))ቁ 1. 𝑓(𝑔(𝑥 )) = 𝑔(𝑓(𝑥 ) = 𝑥

2. 𝑓 ቀ𝑝(ℎ (𝑥 ))ቁ 2. 𝑓(ℎ(𝑥 )) ≠ ℎ(𝑓 (𝑥 )) ∶ 6𝑥 + 1 ≠ 6𝑥 − 7

3. 𝑔 ቀ𝑓(ℎ(𝑥 ))ቁ

4. 𝑔 ቀℎ(𝑓 (𝑥 ))ቁ

5. 𝑔 ቀ𝑓(𝑝(𝑥 ))ቁ

19
Note: Practice Personal Hygiene protocols at all times
GENERAL MATHEMATICS 11
Name of Learner: _____________________ Grade Level: __________________
Section: _____________________________ Date: ________________________

LEARNING ACTIVITY SHEET


SOLVES PROBLEMS INVOLVING FUNCTIONS

Background Information for the Learners

In solving word problems involving functions is basically similar as evaluating


a function for a given value.

Example 1.
The square garden below needs to be seeded around the four identical circular pools. Write a function
for the area (A) that needs to be seeded in terms of the radius of the pools (r). Find the area if r = 2.

5𝑟

Solution:
𝐴𝑟𝑒𝑎 = 𝐴𝑟𝑒𝑎 𝑜𝑓 𝑡ℎ𝑒 𝑠𝑞𝑢𝑎𝑟𝑒 − 4(𝐴𝑟𝑒𝑎 𝑜𝑓 𝑡ℎ𝑒 𝑐𝑖𝑟𝑐𝑢𝑙𝑎𝑟 𝑝𝑜𝑛𝑑)
𝐴 = (5𝑟)2 − 4(𝜋𝑟 2 )
𝐴 = 25𝑟 2 − 4𝜋𝑟 2
If 𝑟 = 2, then
𝐴 = 25(2)2 − 4𝜋(2)2
𝐴 = 100 − 16𝜋 square units

20
Note: Practice Personal Hygiene protocols at all times
Example 2.
Let 𝑓 be a function defined by

𝒇 (𝒙 ) =
f {3𝑥 − 2
𝑥2
,𝑥 < 2
,𝑥 ≥ 2

Determine the domain and range.

Solution:

The graph consists a portion of a line 𝑦 = 3𝑥 − 2 for 𝑥 < 2 and a portion of a parabola 𝑦 = 𝑥 2
that opens upward for 𝑥 ≥ 2. Thus, the domain of 𝑓 is all reals and range is also all reals.

Example 3.
Suppose that 𝑃(𝑥 ) = 𝑥 shows the number of boxes delivered by a Shipping Company. The
shipping fee for each box is presented by 𝑅(𝑥 ) = 250 − 10𝑥, for 0 < 𝑥 ≤ 15. Assume further
that the cost of producing 𝑥 boxes is given by 𝑊 (𝑥 ) = 150𝑥. Find:
a. (𝑃 • 𝑅)(𝑥 )
b. (𝑃 • 𝑅 − 𝑊 )(𝑥 )

Solution:
a. (𝑃 • 𝑅)(𝑥 ) = 𝑥 (250 − 10𝑥 ) = 250𝑥 − 10𝑥 2 , gross income for shipping 𝑥 boxes
𝑏. (𝑃 • 𝑅 − 𝑊 )(𝑥 ) = 250𝑥 − 10𝑥 2 − 150𝑥 = 100𝑥 − 10𝑥 2 , net income from shipping 𝑥
boxes

Example 4.
Given ℎ(𝑥 ) = (2𝑥 2 + 3)3 , express ℎ as the composition of the two functions 𝑓 𝑎𝑛𝑑 𝑔.

Solution:
We can write the function ℎ as
𝑓 (𝑥 ) = 𝑥 3 and 𝑔(𝑥 ) = 2𝑥 2 + 3

Since
(𝑓 ∘ 𝑔)(𝑥 ) = 𝑓(𝑔(𝑥 ))

= 𝑓(2𝑥 2 + 3)

21
Note: Practice Personal Hygiene protocols at all times
= (2𝑥 2 + 3)3
Another pair of functions holds true if:
𝐹 (𝑥 ) = (2𝑥 2 + 3)3 and 𝐺 (𝑥 ) = 𝑥 2
Since
(𝐹 ∘ 𝐺 )(𝑥 ) = 𝐹(𝐺 (𝑥 ))

= 𝐹(𝑥 2 )
= (2𝑥 2 + 3)3

Learning Competency
Solves problems involving functions (GM_M11GM-Ia-4)

EXERCISE 1
Directions: Solve the function in the given condition. [1 point each]

Consider the function 𝑓 (𝑥 ) = 4𝑥 − 1, find:


1. 𝑓 (3) 6. 𝑓 (2𝑥 )
2. 𝑓 (−3) 7. 2𝑓(𝑥)
3. 𝑓 (0) 8. 𝑓 (𝑥 + ℎ)
4. 𝑓 (𝑎 + 1) 9. 𝑓 (𝑥 ) + 𝑓(ℎ)
𝑓(𝑥+ℎ)−𝑓(𝑥)
5. 𝑓 (𝑥 + 1) 10. ,ℎ ≠ 0

EXERCISE 2
Directions: Solve the function in the given condition. Also, determine the domain of the
composite function in each part [2 points each]
Given that 𝑓 𝑎𝑛𝑑 𝑔 are defined by

𝑓 (𝑥 ) = √𝑥 and 𝑔(𝑥 ) = 𝑥 2 − 4
1. 𝑓 ∘ 𝑓
2. 𝑔 ∘ 𝑔
3. 𝑓 ∘ 𝑔
4. 𝑔 ∘ 𝑓

22
Note: Practice Personal Hygiene protocols at all times
1
Given 𝑞 (𝑥 ) = , express 𝑞 as the composition of two functions in two ways.
√2𝑥 3+5

1. the function 𝑓 contains the radical


2. the function 𝑔 contains the radical

EXERCISE 3
Directions: Solve the given problems. [3 points each]
1. A fruit vendor charges P150. 00 per kilogram of grapes plus a fixed delivery fee of P15.00.
How many kilograms of grapes can be delivered for P1000.00?
2. A bus travels along the national highway from station A to station B. It leaves station A at
10:00 am and travels at a constant speed of 70 miles per hour (mph). A car makes the same
route, travelling 15 mph faster but leaving 10 minutes later. When does the car overtake the
bus?
3. An open rectangular box is to be formed by cutting identical squares, each of side 2 in, one
from each corner of a rectangular piece of cardboard, and then turning up the ends. If the area
of the piece of cardboard is 216 in² and the box is to have volume 224 in³, what should have
been the dimensions of the cardboard used?
4. In Puerto Galera Island, a group of tourists decides to ride in a boat. The boat owner charges
a boat ride of P1000.00 for 20 passengers only. However, for additional passengers a fee of
P70.00 per head. Express the function using piecewise and how much will it cost if there are
25 passengers in all?
5. An online shop delivers soap items within and outside Isabela. An initial shipping fee of
P80.00 is collected for orders amounting to P500.00 or less, an extra charge of P40.00 for items
with total value between P500.00 and P1000.00 and a double initial shipping for orders costing
to P1000.00 and up. Write a piecewise function representing the 𝑓 amount of shipping fee for
𝑥 total value of orders. How much is the shipping fee if the total amount of orders are P600.00
and P1350.00?

Reflection:
Please share your insights in this topic.
________________________________________________________________________
________________________________________________________________________
________________________________________________________________________
References:
Verzosa, D.B, et.al (2016). General Mathematics. Quezon City, Manila
Alferez, G. S. (2014). Introduction to Calculus. Quezon City, Manila
Leithold, L. (1996). The Calculus 7. New York City.
Brown, R.G (1994). Advanced Mathematics, Precalculus with Discrete Mathematics and
Data Analysis, Houghton Mifflin, Boston.
Rolando, M.A, et.al (2002). Differential Calculus. Philippines.

23
Note: Practice Personal Hygiene protocols at all times
ANSWER KEY
EXERCISE 1
1. 11 6. 8𝑥 − 1
2. −13 7. 8𝑥 − 2
3. −1 8. 4𝑥 + 4ℎ − 1
4. 4𝑎 + 3 9. 4𝑥 + 4ℎ − 2
5. 4𝑥 + 3 10. 4

EXERCISE 2
1
1. 4√𝑥 1. 𝑓 (𝑥) = and 𝑔(𝑥 ) = 𝑥 3
√2𝑥+5

2. 𝑥 4 − 8𝑥 2 + 12
1
3. √𝑥 2 − 4 2. 𝑓 (𝑥) = and 𝑔(𝑥 ) = √2𝑥 3 + 5
𝑥

4. 𝑥 − 4

EXERCISE 3
1. 6. 57 kg
2 2
2. 𝑡 = 54 3 𝑚𝑖𝑛𝑠. The car overtakes the bus 54 3 𝑚𝑖𝑛𝑠. after the bus departure, i.e.
2
10: 54 3 𝑎. 𝑚.

3. 8 in. by 14 in.
4.

𝒇 (𝒙 ) =
f { 1000
1000 + 70(𝑥 − 20)
, 0 < 𝑥 ≤ 20
, 𝑥 > 20

At x = 25, the charge is P 1, 350.00

24
Note: Practice Personal Hygiene protocols at all times
5.

{
80 ,0 < 𝑥 ≤ 500
𝒇 (𝒙 ) = 80 + 40 , 500 < 𝑥 < 1000
f 2(80) , 𝑥 ≥ 1000

At x = 600, the shipping fee is P120.00 and at x = 1350, the fee is P160.00

25
Note: Practice Personal Hygiene protocols at all times
GENERAL MATHEMATICS 11
Name: _____________________ Grade Level: ____
Date: ______________________ Score: _________

Learning Activity Sheet


Rational Functions; Real-life Applications

Background Information for Learners

Rational expressions and rational equations can be useful tools for representing
real life situations and for finding answers to real problems. In particular, they are quite good
for describing distance-speed-time questions, and modeling multi-person work problems.
Rational equations can be used to solve a variety of problems that involve rates, times
and work. Using rational expressions and equations can help us answer questions about how
to combine workers or machines to complete a job on schedule.(MontereyInstitute.org)

Learning Competency
Represents real-life situations using rational functions. (M11GM-Ib-1)

Activity 1: IncrediBULB Installation


Directions: Answer the problem involving real life representation of rational equation.

An ISELCO I lineman takes 2 hours to plant 500 streetlight bulbs. An ISELCO II lineman
takes 3 hours to plant 450 streetlight bulbs.

a. Working together, how long should it take them to plant 1500 bulbs?
b. If a MERALCO lineman who can install 400 streetlight bulbs in two hour replaced
the ISELCO II lineman in the installation, how long does it take for them to install
1500 streetlight bulb?
c. Working together, how long does it take for the three linemen to install 2000
streetlight

Activity 2:Paint Paint De SaraPaint


Directions: Answer the problem involving real life representation of rational equation.

Sarah, Ara and Shar can paint a room together. If Ara does the job alone she can paint the
room in 5 hours. If Shar works alone, he can paint the room in 6 hours. Sarah can paint the
room in 7 hours.

26
Note: Practice Personal Hygiene protocols at all times
a. If Shar and Ara work together, how long does it take for them to finish painting the
room?
b. If Sarah and Ara work together, how long does it take for them to finish painting the
room?
c. If Shar and Sarah work together, how long does it take for them to finish painting the
room?
d. If the three work together, how long does it take for them to paint the room?

Activity 3: PETER, PIPE and POOL


Directions: Answer the problem involving real life representation of rational equation.

1. Peter’s house owns a pool. Later that day Peter decided to take a bath using that pool
but in order to swim he must fill the pool with water. The pool has two pipes. One
pipe can fill a pool 1.5 times faster than a second pipe. If both pipes are open, the
pool can be filled in 6 hours. If only the slower pipe is open, how long would it take
to fill the pool?

Guide Questions
1. How does rational function help solves problems in real life?
2. Do you think you can answer the problems without having a background idea on
rational functions?

Rubrics for scoring


Rubric Description Weight
Correctness of the Answer The answer presented is exact to what 70%
is agreed which means it follows the
number of decimals present in the
final answer to avoid confusion.
Detailed Procedure The procedure was a carefully step by 30%
step procedure not ignoring the
importance of each step.

Generalization
Direction: Complete the following statement by the things that you learned on today’s
discussion.

Today, I learned that rational functions can be used in real life through
_________________________________________________________________________
_________________________________________________________________________
_________________________________________________________________________

27
Note: Practice Personal Hygiene protocols at all times
_________________________________________________________________________
_________________________________________________________________________
_________________________________________________________________________
_________________.

References for the Learners


1. (https://www.onlinemathlearning.com/math-work-problems.html)
2. (https://www.youtube.com/watch?v=fnSkN61cxQM)

28
Note: Practice Personal Hygiene protocols at all times
Answer Key
Activity 1: IncrediBULB Installation

a. ISELCO I lineman: 500


bulbs/2 hours = 250
bulbs/hour

ISELCO II lineman: 450


bulbs/3 hours =
150 bulbs/hour
Two lineman working
together:
250 + 150 bulbs/hour = 400
bulbs/hour
o
400 1500
• =
1 𝑡
400 1500
• (1𝑡) = (1𝑡)
1 𝑡
• 400𝑡 = 1500
400𝑡 1500
• =
400 400
• T=3 ¾ hours
Answer: It takes 3 hours and 45 minutes for the two linemen to plant 1500 streetlight bulbs.

b. ISELCO I lineman: 500


bulbs/2 hours = 250
bulbs/hour

MERALCO lineman: 400


bulbs/2 hours =
200 bulbs/hour
Two lineman working
together:
250 + 200 bulbs/hour = 450
bulbs/hour
o
450 1500
• =
1 𝑡
450 1500
• (1𝑡) = (1𝑡)
1 𝑡
• 450𝑡 = 1500
450𝑡 1500
• =
450 450
• T=3 1/3 hours

29
Note: Practice Personal Hygiene protocols at all times
Answer: It takes 3 hours and 20 minutes for the two linemen to plant 1500 streetlight bulbs.

c. ISELCO I lineman: 500


bulbs/2 hours = 250
bulbs/hour

ISELCO II lineman: 450


bulbs/3 hours =
150 bulbs/hour
MERALCO lineman: 400
bulbs/2 hours= 200 bulbs/hour
Three linemen working
together:
250 + 150+200 bulbs/hour = 600
bulbs/hour
o
600 2000
• =
1 𝑡
600 2000
• (1𝑡) = (1𝑡)
1 𝑡
• 600𝑡 = 2000
600𝑡 2000
• 600 = 600
• T=3 1/3 hours
Answer: It takes 3 hours and 20 minutes for the three linemen to plant 2000 streetlight bulbs.

Activity 2: Paint Paint De SarahPaint

Ara = 1/5 room/hour

Shar = 1/6 room/hour

Sarah = 1/7 room/hour

1 1 1
a. +6=
5 𝑡

1 1 1
(30𝑡) + (30𝑡) = (30𝑡)
5 6 𝑡

6t+5t=30

30
Note: Practice Personal Hygiene protocols at all times
11𝑡 30
= 11
11

t=2.73 hours
Answer: It takes 2 hours and 43.63 seconds for the two to paint the room.
b.
1 1 1
+5=
7 𝑡

1 1 1
(35𝑡) + (35𝑡) = (35𝑡)
7 5 𝑡

5t+7t=35
12𝑡 35
=
12 12

t=2.92 hours
Answer: It takes 2 hours and 55 seconds for the two to paint the room.

c.
1 1 1
6
+7= 𝑡

1 1 1
(42𝑡) + (42𝑡) = (42𝑡)
6 7 𝑡

6t+7t=42

13𝑡 42
= 13
13

t=3.23 hours
Answer: It takes 2 hours and 13.84 seconds for the two to paint the room.

d.
1 1 1 1
+6+7=
5 𝑡

1 1 1 1
(210𝑡) + (210𝑡) + (210𝑡) = (210𝑡)
5 6 7 𝑡

42t + 35t + 30t= 210

107𝑡 210
=
107 107

t=1.96 hours

31
Note: Practice Personal Hygiene protocols at all times
Answer: It takes 1 hours and 57.76 seconds for the three to paint the room.

Activity 3: PETER, PIPE and POOL

1
fast pipe = 𝑃

1
slow pipe = 1.5𝑃

1
fast pipe + slow pipe = 6

1 1 1
+ 1.5𝑃 = 6
𝑃
1 1 1
(6) + 1.5𝑃 (4) = 6 (𝑃)
𝑃
6 4 𝑃
+ 6𝑃 = 6𝑃
6𝑃
6+4=p

p =10 hours
10 • 1.5 = 15 hours
Answer
The slower pipe will take 15 hours to fill the pool alone
Prepared by:

32
Note: Practice Personal Hygiene protocols at all times
GENERAL MATHEMATICS 11
Name: _____________________ Grade Level: _______________
Date: ______________________ Score: _____________________

Learning Activity Sheet


Distinguishing Rational Functions, Rational Equations and Rational
Inequality

Background Information for Learners

Rational Function or expression, Rational Equations and Rational Inequalities are


three sister terms in the fields of mathematics. These three terms are so connected to each
other but differ by some elements
This learning activity sheet aims to distinguish these three terms and find the
uniqueness of each term. Also, this also aims to familiarize the students into these
mathematical ideas before proceeding to further topics which inclused solving of these.
Learning Competency
Distinguishes rational function, rational equation, and rational inequality. (M11GM-Ib-2)
Activity 1: Describe me.
Directions: write words that you think best describes the following terms.

rational
Function

33
Note: Practice Personal Hygiene protocols at all times
Rational
Equation

rational
inequality

Activity 2:Where do I Belong?


Directions: Identify on which group do the following expressions and functions belong.
Identify whether they are rational function/expression, rational equation or rational
inequality.

34
Note: Practice Personal Hygiene protocols at all times
𝑥+3 𝑥 𝑥+3 4𝑥 2 + 2𝑥 + 8 5𝑥 2 + 2𝑥 + 3 4𝑥 2 + 6𝑥 + 3
+ =1 ≥2 =9
𝑥 3 2 2 + 3𝑥 3
𝑥+3
𝑥+1 9𝑥 + 16 2𝑥 + 9 𝑥 + 1 𝑥2 3 𝑥 + 16
+ ≤0
>2 3𝑥 7𝑥 + =4 ≤0
3 𝑥 3 2𝑥 3𝑥
𝑥+2 4𝑥 2 +2𝑥+8
=2
𝑥 + 9 2𝑥 + 3 8𝑥 + 1 4𝑥 2 + 8𝑥 + 2
=1 2+3𝑥 + >0
𝑥 3 7 𝑥 2𝑥 + 3

Rational Function or Rational Equation Rational Inequality


Expression

𝒙+𝟑 𝑥 𝑥+3 𝑥+1


+ =1 >2
𝒙 3 2 3
𝟗𝒙 + 𝟏𝟔 𝑥+2 4𝑥 2 + 2𝑥 + 8
=1 ≥2
𝒙 𝑥 2 + 3𝑥
4𝑥 2 +2𝑥+8 2𝑥 + 9 𝑥 + 1
𝟓𝒙𝟐 + 𝟐𝒙 + 𝟑 =2 + ≤0
2+3𝑥 3𝑥 7𝑥
𝒙+𝟑
𝟒𝒙𝟐 + 𝟖𝒙 + 𝟐 𝑥2 3 8𝑥 + 1
+ =4 >0
𝟐𝒙 + 𝟑 3 2𝑥 𝑥
𝒙 + 𝟗 𝟐𝒙 + 𝟑 4𝑥 2 + 6𝑥 + 3 𝑥 + 16
+ =9 ≤0
𝟑 𝟕 3 3𝑥

Activity 3: Compare and Contrast


Directions: Using all the knowledge gathered through all the activities, compare and
contrast rational function or expression, rational equation and rational inequality using Venn-
Euler Diagram

Rational Function or
Expression

Rational Equation
Rational Inequality

35
Note: Practice Personal Hygiene protocols at all times
Guide Questions
3. How are the three terms related?
4. How to distinguish each term?
Rubrics for scoring (activity 3)
Rubric Description Weight
Conciseness of the answer The answers given are concise which 20%
are less words more thought.
Organization The answers are organize in a way 30%
that it is easy to understand.
Relevance The answers given are relevant and 40%
cited from a reliable source.
Creativeness The students put an effort to make a 10%
visual improvement on his Venn-
Euler Diagram.

Generalization
Direction: Write a brief summary on the characteristics of rational function or expression,
rational equality and rational inequality.

_________________________________________________________________________
_________________________________________________________________________
_________________________________________________________________________
_________________________________________________________________________
_________________________________________________________________________
_________________________________________________________________________
_________________.
References for the Learners
3. (https://study.com/academy/lesson/rational-function-definition-equation-examples.html)

36
Note: Practice Personal Hygiene protocols at all times
4. (https://www.wyzant.com/resources/answers/10941/what_distinguishes_a_rational_express
ion_from_a_rational_equation)

Answer Key
Activity 1:

“No Specific answer”

Activity 2:

Rational Function or Rational Equation Rational Inequality


Expression

𝒙+𝟑 𝑥 𝑥+3 𝑥+1


+ =1 >2
𝒙 3 2 3
𝟗𝒙 + 𝟏𝟔 𝑥+2 4𝑥 2 + 2𝑥 + 8
=1 ≥2
𝒙 𝑥 2 + 3𝑥
4𝑥 2 +2𝑥+8 2𝑥 + 9 𝑥 + 1
𝟓𝒙𝟐 + 𝟐𝒙 + 𝟑 =2 + ≤0
2+3𝑥 3𝑥 7𝑥
𝒙+𝟑
𝟒𝒙𝟐 + 𝟖𝒙 + 𝟐 𝑥2 3 8𝑥 + 1
+ =4 >0
𝟐𝒙 + 𝟑 3 2𝑥 𝑥
𝒙 + 𝟗 𝟐𝒙 + 𝟑 4𝑥 2 + 6𝑥 + 3 𝑥 + 16
+ =9 ≤0
𝟑 𝟕 3 3𝑥

Activity 3:
“No specific Answer”

37
Note: Practice Personal Hygiene protocols at all times
GENERAL MATHEMATICS 11
Name: _____________________ Grade Level: ______________
Date: ______________________ Score: ____________________

Learning Activity Sheet


Solving Rational Equations and Inequalities

Background Information for Learners

A rational expression is an expression that can be written as a ratio of two


polynomials.
A rational expression can be described as a function where either the numerator,
denominator, or both have a variable on it.
A rational equation or inequality can be solved for all x values that satisfy the equation or
inequality. Whereas we solve an equation or inequality, we do not "solve" functions. Rather,
a function (and in particular, a rational function) expresses a relationship between two
variables (such as x and y), and can be represented by a table of values or a graph.
A basic application of rational equation is problems from the multiplication-division
operations. It is given by all the multiplicative formulas of the form AB=C may be written
𝐵
as A=𝐶 . This divisional from lead to rational equations.

The calculation of “per unit” such as “cost per unit” or amount per unit and even production
problems are good examples of rational equation.

Learning Competency
Solves rational equations and inequalities.( M11GM-Ib-3)

Activity 1:The Hidden Message


Directions: A Hidden Message was encrypted and can only be deciphered by answering
some rational equations. Each answer corresponds to one word. The arrangement of the
words will be given. Your task is to decode the message. GOODLUCK!

(LIVES)
𝑦 6
+ =1
𝑦−3 𝑦+3

38
Note: Practice Personal Hygiene protocols at all times
(HOME)
1 2𝑛+1 2
= 𝑛2 +2𝑛−8 + 𝑛+4
𝑛−2

(SAVE)
10 4
=𝑥+2
3

(SAFE)
6 5 −7
+4=
𝑥 4

39
Note: Practice Personal Hygiene protocols at all times
(STAY)
1 1 1
+ (𝑥−8) = 3
𝑥

12 7/3 12 -2 3 1
Activity 2:Think Rational; Solves Inequalities
Directions: Solve the rational inequality

𝑥 2 +8𝑥+16
1. ≥0
𝑥 2 −𝑥−20

𝑥+5
2. >0
𝑥 2 +6𝑥+9

𝑥 2 −49
3. ≤0
𝑥 2 +13𝑥+42

40
Note: Practice Personal Hygiene protocols at all times
Activity 3: Sail!Sale!Sayl
Directions: Answer the problem involving real life representation of rational equation.

2. Sayl Corporation experts on the sale of sails used in small boats. Their monthly
revenue (in thousands) for the nth week after they opened is given by the equation
120𝑡
𝑆 = 𝑡 2 +100.
a. What are their sales in their 5th week?

b. What is their sale after a quarter?(assuming that a month is composed of 4 weeks)

c. What is their sale after half a year? (assuming that a month is composed of 4 weeks)

d. In which week would the sale (s)have been 6?(note that the sales is express in
thousands)

e. In which week would the sale(s) have been 8? (note that the sales is express in thousa

Rubrics for scoring


Simple pointing scoring;
Activity 1; 1 point each item and 5 points on the decoded message.
Activity 2; 5 points each item.
Activity 3; 3 points each item.

41
Note: Practice Personal Hygiene protocols at all times
Reflection
Direction: Complete the table below that shows your learning on rational equations and
inequalities.

What I learned about…


Rational Equations Rational Inequalities

References for the Learners


5. (https://study.com/academy/lesson/solving-equations-inequalities-involving-rational-
functions.html)
6. (http://teachtogether.chedk12.com/teaching_guides/view/6)

42
Note: Practice Personal Hygiene protocols at all times
Answer Key
ACTIVITY 1
(LIVES)
𝑦 6
+ =1
𝑦−3 𝑦+3

LCD is (y-3)(y+3)

𝑦 6
(𝑦 − 3)(𝑦 + 3)]( + = 1)
𝑦−3 𝑦+3
y(y+3)+(6)(y-3)=(y+3)(y-3)
y2+3y+6y-18=y2-9
y2+9y-18=y2-9
9y=9
9
Y=1

(HOME)
1 2𝑛+1 2
= 𝑛2 +2𝑛−8 + 𝑛+4
𝑛−2

1 2𝑛+1 2
[(𝑛 − 2)(𝑛 + 4)](𝑛−2 = 𝑛2 +2𝑛−8 + 𝑛+4)

1(n+4)=2n+1+2(n-2)
n+4=2n+1+2n-4
n+4=4n-3
7=3n
3
n= 7/3

43
Note: Practice Personal Hygiene protocols at all times
(SAVE)
10 4
=𝑥+2
3
10 4
(3𝑥)( = + 2)
3 𝑥

10x=12+6x
4x=12
4
x=3

(SAFE)
6 5 −7
+4=
𝑥 4
6 5 −7
(4𝑥)(𝑥 + 4 = )
4

24+5x=-7x
24=-12x
-12
x=-2

(STAY)
1 1 1
+ =
𝑥 (𝑥−8) 3

1 1 1
[(𝑥 )(𝑥 − 8)(3)](𝑥 + (𝑥−8) = 3)

(3)(1)(x-8)+(x)(3)=(x)(x-8)
3x-24+3x=x2-8x
X2-8x-6x+24=0
X2-14x-24=0

44
Note: Practice Personal Hygiene protocols at all times
(x-12)(x-2)
X=12;x=2

Stay Home Stay Safe Save Lives


12 7/3 12 -2 3 1

ACTIVITY 2
𝑥 2 +8𝑥+16
1. 𝑥 2 −𝑥−20
≥0

45
Note: Practice Personal Hygiene protocols at all times
(𝑥+4)(𝑥+4)
≥0
(𝑥−5)(𝑥+4)
𝑥+4
≥0
𝑥−5

X+4=0 x-5=0
x=-4 x=5

+ - +

-4 5

4 5

Answer: (−∞, 4] ∪ (5, ∞)

𝑥+5
2. >0
𝑥 2 +6𝑥+9

𝑥+5
>0
(𝑥+3)(𝑥+3)
𝑥+5
>0
(𝑥+3)2

X+5=0 x+3=0
x=-5 x=-3

46
Note: Practice Personal Hygiene protocols at all times
- + +

-5 -3

-5 -3

Answer: (−5, −3] ∪ [−3, ∞)

𝑥 2 −49
3. ≤0
𝑥 2 +13𝑥+42

(𝑥+7)(𝑥−7)
≤0
(𝑥+6)(𝑥+7)
𝑥−7
≤0
𝑥+6

X-7=0 x+6=0
x=7 x=-6

+ - +

-6 7

47
Note: Practice Personal Hygiene protocols at all times
-6 7

Answer: (−6, 7]

ACTIVITY 3
1. Sayl Corporation experts on the sale of sails used in small boats. Their monthly
revenue (in thousands) for the nth week after they opened is given by the equation
120𝑡
𝑆 = 𝑡 2 +100.
a. What are their sales in their 5th week?

120𝑡
𝑆 = 𝑡 2 +100.
120(5)
𝑆 = 52 +100.
600
𝑆 = 25+100.
600
𝑆 = 125.

𝑆 = 4.8 𝑜𝑟 4,800 𝑠𝑎𝑖𝑙𝑠


Answer: The sales will be 4,800 sails after 5 weeks.

b. What are their sales after a quarter?

120𝑡
𝑆 = 𝑡 2 +100.
120(12)
𝑆 = 122 +100.
1440
𝑆 = 144+100.
1440
𝑆= .
244

𝑆 = 5.90 𝑜𝑟 5,900 𝑠𝑎𝑖𝑙𝑠

48
Note: Practice Personal Hygiene protocols at all times
Answer: The sales will be 5,900 sails after a quarter.

c. What is their sales after half a year?

120𝑡
𝑆 = 𝑡 2 +100.
120(24)
𝑆 = 242 +100.
2880
𝑆 = 576+100.
2880
𝑆= .
676

𝑆 = 4.26 𝑜𝑟 4,260 𝑠𝑎𝑖𝑙𝑠


Answer: The sales will be 4,260 sails after half a year.

d. In which week would the sale (s)have been 6?(note that the sales is express in
thousands)
120𝑡
𝑆 = 𝑡 2 +100.
120𝑡
6 = 𝑡 2 +100.

6(𝑡 2 + 100) = 120𝑡


. 6(𝑡 2 + 100) = 120𝑡
. 6𝑡 2 + 600 = 120𝑡
6𝑡 2 − 120𝑡 + 600 = 0
𝑡 2 − 20𝑡 + 100 = 0
(x-10)2
X=10

Answer: it takes 10 weeks for the sale to reach 6,000 sails.

49
Note: Practice Personal Hygiene protocols at all times
e. In which week would the sale (s)have been 8?(note that the sales is express in
thousands)
120𝑡
𝑆 = 𝑡 2 +100.
120𝑡
8 = 𝑡 2 +100.

8(𝑡 2 + 100) = 120𝑡


. 8(𝑡 2 + 100) = 120𝑡
. 8𝑡 2 + 800 = 120𝑡
8𝑡 2 − 120𝑡 + 800 = 0
𝑡 2 − 15𝑡 + 100 = 0
(x-20)(x+5)
X=20

Answer: it takes 20 weeks for the sale to reach 8,000 sails.

EAM Fourth Year, Module 13 (TG), EASE Module Fourth Year · Triangle
Trigonometry, Mo, Module 2 (L

50
Note: Practice Personal Hygiene protocols at all times
GENERAL MATHEMATICS 11
Name: ______________________________ Grade Level: ____________________
Date: _______________________________ Score: _______________________

LEARNING ACTIVITY SHEET


REPRESENTS A RATIONAL FUNCTION THROUGH ITS TABLE OF VALUES,
GRAPH, EQUATION
Background Information for Learners

This activity serves as a learning guide for the learners. It facilitates lesson
comprehension as it specifically aims for student’s mastery in representing rational functions.
Rational functions are expressed in the ratio of two polynomial functions P(x) and Q(x),
where P(x)is the numerator and Q(x) is the denominator. The function is defined for all values
of the variable except the zeros of Q(x).
There are different ways of representing rational function such as a) table of values b)
graph, and c) equation.
Constructing a table of values for a given rational function is vital in sketching its graph.
Through the graph of rational function, one can easily grasp the behavior of the function values
and also its other characteristics such as zeros, vertical and horizontal asymptotes and
intercepts.
Learning Competency: The learner represents a rational function through its: a) table of
values, b) graph, c) equation. (GM_M11GM-Ib-4)

Activity 1

Example:
1
Represents f(x) = 𝑥 by its table of values. Using the values of x from -5 to 5.
Answer:
X -5 -4 -3 -2 -1 0 1 2 3 4 5
1 1 1 1 1 1 1 1
f(x) -5 -4 -3 -2 -1 Undefined 1 2 3 4 5

Represents each of the following rational function by its table of values.


1 𝑥 2
a.) f(x) = 2𝑥 b.) g(x) = 𝑥−1 c.) h(x) = 𝑥+1

Directions:
Represent each rational function by its table of values using the values of x
from -5 to 5.
1
a. f(x)=2𝑥

x -5 -4 -3 -2 -1 0 1 2 3 4 5
f(x)

51
Note: Practice Personal Hygiene protocols at all times
𝑥
b. g(x) =
𝑥−1

x -5 -4 -3 -2 -1 0 1 2 3 4 5
g(x)
2
c. h(x) =
𝑥+1

x -5 -4 -3 -2 -1 0 1 2 3 4 5
h(x)

Activity 2

Example:
1
Represents f(x)= 𝑥 by its graph. Then identify the zeros, intercepts and asymptotes.

Answer:

Zeros: none
Intercepts
x-intercepts: none
y-intercepts: none
Asymptotes
Vertical: x=0
Horizontal: y=o

Represents each rational function by its graph.Then identify the zeros,intercepts and
asymptotes.
1 𝑥 𝑥−2
a.) f(x) = 2𝑥 b.) g(x) = 𝑥−1 c.) h(x) = 𝑥+1

52
Note: Practice Personal Hygiene protocols at all times
Activity 3

Example:
Represent this problem in a rational function, then answer what is asked.

In an inter-barangay basketball league, the team from barangay 1 has won 9 out of 20
games a winning percentage of 45%. What would be their winning percentage if they win 5
games consecutively?

Solutions:
Let x be the number of wins of Barangay 1 needs to win in a row. Then the function P is a
function of the number of wins that the team needs to win. The function is
9+𝑥
P(x)= 20+𝑥

9+5 14
P(5)= 20+5 =25 =.56=56%
Therefore, the winning percentage of Barangay 1, if they win 5 games in a row is 56%

Represent the following problems below in a rational function, then answer what
is asked:

1. Consider a 100-meter track used for foot races. The speed of the runner can be
computed by taking the time it will take him to run the track. Appling it to the formula
𝑑
of average speed s= 𝑡 , what is the rational function represented by the speed as a
function of time? What is the speed of the runner in 20 seconds?
2. Let’s say you are taking an exam. You already got 18 questions correctly out of 23,
which is a grade percentage of 78%.What would be your grade percentage if you got
the last 2 consecutive questions correctly?

53
Note: Practice Personal Hygiene protocols at all times
Answer Key
(Activity 1)
1
a. f(x)=2𝑥

x -5 -4 -3 -2 -1 0 1 2 3 4 5
1 1 1 1 1 1 1 1 1 1
f(x) - 10 -8 -6 -4 -2 Undefined
2 4 6 8 10

𝑥
b. g(x) = 𝑥−1

x -5 -4 -3 -2 -1 0 1 2 3 4 5
5 4 3 2 1 3 4 5
f(x) 5 4 3 2
0 Undefined 2 2 3
6 4

2
c. h(x) =
𝑥+1

x -5 -4 -3 -2 -1 0 1 2 3 4 5
1 2 2 1 2 1
f(x) -2 -3 -1 -2 Undefined 2 1 3 2 5 3

(Activity 2)

𝟏 𝒙 𝒙−𝟐
a. f(x) = 𝟐𝒙 b. g(x) = 𝒙−𝟏 c. h(x) = 𝒙+𝟏

Zeros: none Zeros:0 Zeros:2


Intercepts: Intercepts: Intercepts:
x-intercepts: none x-intercepts:(0,0) x-intercepts:(2,0)
y-intercepts: none y-intercepts:(0,0) y-intercepts:(0,-2)
Asymptotes: Asymptotes: Asymptotes:
Vertical: x=0 vertical: x=1 Vertical: x=-1
Horizontal: y=0 horizontal: y=1 Horizontal: y=1

54
Note: Practice Personal Hygiene protocols at all times
(Activity 3)

1.Solutions:
Let x the time it takes the runner to run 100 meters. Then the
function S is a function of the time it takes by the runner to run 100 meters. The
function is
100
S(x)= 𝑥

100
S(20)= 20 = 5m/s
Therefore, the speed of the runner for 20 minutes is 5m/s.

2. Solution:
Let x be the additional number of consecutive questions correctly answered.
Then the function f is a function of the number of questions that you need to
get correctly. The function is
18+𝑥
f(x)= 23+𝑥

18+2 20
f(2)= 23+2 =25 =.8=80%
Therefore, your grade percentage , if you got the last 2 questions correctly is 80% .

Reflection:
___________________________________________________________________________
___________________________________________________________________________
___________________________________________________________________________
___________________________________________________________________________
___________________________________________________________________________
______________________________

References:

GENERAL MATHEMATICS (LM),First Edition 2016, DIWA Senior High School


Series:General Mathematics, Next Century Mathematics 11 General Mathematics Copyright
2016, MSA ADVANCED ALGEBRA with Trigonometry

55
Note: Practice Personal Hygiene protocols at all times
GENERAL MATHEMATICS
Name:____________________ Grade Level:____________________
Date:______________________ Score:_______________________

LEARNING ACTIVITY SHEET


THE DOMAIN AND RANGE OF RATIONAL FUNCTION

Background Information for Learners


This activity serves as a learning guide for the learners. It facilitates lesson
comprehension as it specifically aims for student’s mastery in finding the Domain and the
Range of a Rational Functions.
The Domain of a function f(x) is the set of all values for which the function is defined
and the range of the function is the set of all values that f takes. The Domain and Range of a
Rational Function may be found by the following: a) using algebraic solution b) looking at its
graph c) sketching its graph.
Learning Competency: The learner finds the domain and range of a rational function.
(MIIGM-lb-5)

Activity 1
Example: Find the domain and range using algebraic method of the rational function
𝑥
f(x)= 𝑥+2
Solutions: To find the domain, let the denominator
x+2 = 0
x =-2
Therefore, the domain is
D: {𝒙⁄𝒙 is a set of all real numbers, x≠-2}

To find the range, solve x in terms of y


𝑥
𝑦 = 𝑥+2
y(x+2)=x
xy-2y=x
xy-x=2y
x(y-1)=2y
2𝑦
𝑥 = 𝑦−1
Therefore, the range is
𝒚
R: { ⁄𝒚 is set of all real numbers,y≠1}

Find the domain and range of the following rational functions using algebraic method.
2 𝑥−2
a. 𝑦 = 𝑥−1 b. . y = 2𝑥+4

56
Note: Practice Personal Hygiene protocols at all times
Activity 2
Find the asymptotes and domain and range of this given graph below.

Asymptotes
Vertical: x=5
Horizontal: y= 0
Domain:{𝑥⁄𝑥 is a set of all real
numbers,x≠5}
𝑦
Range:{ ⁄𝑦 is a set of all real
numbers, y≠ 0}

Continuation:
Find the domain and range of the given graph of rational function.
a. b.

57
Note: Practice Personal Hygiene protocols at all times
Activity 3
𝟐𝒙−𝟒
Example: Sketch the graph of f(x)= 𝒙+𝟐 then, find the domain and range.

Answer:

Domain: {𝑥⁄𝑥 is a set of all real


numbers, x≠-2}

𝑦
Range:{ ⁄𝑦 is a set of all real numbers,
y≠ 2}

Sketch the graph of each rational function. Then, find their domain and range
1 𝑥+2
A. 𝑓(𝑥 ) = 4𝑥−1 B. 𝑔(𝑥 ) = 𝑥−3

Reflection:
What have I learned from the topic?
__________________________________________________________________________________
________________________________________________________________________.

References:
GENERAL MATHEMATICS (LM),First Edition 2016, DIWA Senior High School
Series:General Mathematics, Next Century Mathematics 11 General Mathematics Copyright
2016

58
Note: Practice Personal Hygiene protocols at all times
Answer key
(Activity 1)
2
a. 𝑦 = 𝑥−1

To find the domain, let the denominator


x- 1 = 0
x =1
Therefore, the domain is

D: {𝒙⁄𝒙 is a set of all real numbers,x≠1}

To find the range, solve x in terms of y


2
𝑦=
𝑥−1
y(x-1)=2
xy-y=2
xy=y+2
𝑦+2
𝑥= 𝑦
Therefore, the range is
𝒚
R: { ⁄𝒚 is set of all real numbers,y≠0}

𝑥−2
𝑏. 𝑦 = 2𝑥+4

To find the domain, let the denominator


2x+4 = 0
2x =-4
X=-2
Therefore, the domain is

D: {𝒙⁄𝒙 is a set of all real numbers,x≠-2}

To find the range, solve x in terms of y


𝑥−2
𝑦 = 2𝑥+4
y(2x+4)=x-2
2xy+4y=x-2
2xy-x=-4y-2
x(2y-1)=-4y-2
−4𝑦−2
x= 2𝑦−1
Therefore, the range is

𝒚 𝟏
R: { ⁄𝒚 is set of all real numbers,y≠𝟐}

59
Note: Practice Personal Hygiene protocols at all times
(Activity 2)
A. a. x=1
b. y=0
c. D: {𝑥⁄𝑥 is a set of all real numbers, x≠1}
𝑦
d. R: { ⁄𝑦 is set of all real numbers, y≠0}

B. a. x=-1
1
b.y = 2
c. D: {𝑥⁄𝑥 is a set of all real numbers, x≠-1}
𝑦 1
d. R: { ⁄𝑦 is set of all real numbers, y≠ } 2

(Activity 3)
1
𝑎. 𝑓 (𝑥 ) = 4𝑥−1

Domain: {𝑥⁄𝑥 is a set of all real


1
numbers, x≠ }
4

𝑦
Range:{ ⁄𝑦 is a set of all real
numbers, y≠ 0}

𝑥+2
𝑏. 𝑔(𝑥 ) = 𝑥−3
Domain: {𝑥⁄𝑥 is a set of all real
numbers, x≠3}

𝑦
Range:{ ⁄𝑦 is a set of all real
numbers, y≠ 1}

60
Note: Practice Personal Hygiene protocols at all times
GENERAL MATHEMATICS 11
Name of Learner:________________ Grade Level:______
Section:________________________ Date:____________

LEARNING ACTIVITY SHEET


DETERMINING THE INTERCEPTS, ZEROES AND ASYMPTOTES OF
RATIONAL FUNCTIONS

Background Information for Learners


Rational Functions can be written in the form f(x)= N(x)/D(x) where N(x) and D(x) are
polynomials and D(x) is not the zero polynomial.
The intercepts are the x-or y- coordinates of the points at which the graph crosses the
x-axis of y-axis respectively. The zeros of the rational function f are the values of the
independent variable that make the numerator zero but are not restrictions of the rational
function f. Moreover, the asymptote is a line (or a curve) that the graph of a function gets close
to but not touch either x- or y-axis.
Remember:
1. To find the x-intercept, substitute 0 for y and solve for x.
To find the y-intercept, substitute 0 for x and solve for y.
2. To find the zeros of rational functions
➢ Factor the numerator and the denominator of the rational function f if possible
➢ Identify the restrictions of the rational function f. (The restrictions are the values
of the independent variable that make the denominator equal to zero.)
➢ Identify the values of the independent variable that make the numerator equal
to zero.
➢ The zeros of the rational function f are the values of the independent variable
that make the numerator zero but are not restrictions of the rational function f.
3. On vertical asymptotes they are the restrictions on the x-values of a reduce rational
function by equating the denominator to 0 and solve for x.

On horizontal Asymptotes
➢ If n<m, the graph of f has the line y=0
➢ If n=m, the graph of f has the line y=a n/bm where an and bm are the leading
coefficients of the numerator and denominator, respectively
➢ If n>m, the graph has no horizontal asymptote.

Learning Competency
Determines the (a) intercepts; (b) zeroes (c) asymptotes of rational functions (Quarter 1, Week
2, M11GM-1c-1)

61
Note: Practice Personal Hygiene protocols at all times
Illustrative Examples:
A. Which of the following are rational functions?
1. f(x)= (𝑥 + 2)/(1 + 𝑥)
2. f(x)= (𝑥 2 + 14𝑥 − 15)⁄(𝑥 + 7)
3. g(x)=(𝑥 2 + √𝑥)⁄(2𝑥 2 − 1)
3
4. g(x)= 5 + 𝑥⁄𝑥 2 + √𝑥 + 1
Solution:
1. Rational Function: Both numerator and denominator are polynomials
2. Rational Function: Both numerator and denominator are polynomials
3. Not a Rational Function: Numerator is not a polynomial
4. Not a Rational Function: Denominator is not a polynomial

B. Find the x -and y intercepts, zeros and the asymptotes of


1. f(x)= (𝑥 + 6)/(𝑥 − 3)
Solution:
For x intercept
f(x)= (𝑥 + 6)/(𝑥 − 3)
0 = (𝑥 + 6)/(𝑥 − 3) Substitute 0 for y or f(x)
0= (𝑥 + 6) Multiply both sides by x-3
x= -6 Simplify
Therefore, the x-intercept is -6 or (-6,0)

For y intercept

f(x)= (𝑥 + 6)/(𝑥 − 3)
f(0)= (0 + 6)/(0 − 3) Substitute 0 for x
= 6/−3 Simplify
= -2
Therefore, The y intercept is -2 or (0,-2)

For the zeros

f(x)= (𝑥 + 6)/(𝑥 − 3)
𝑥−3= 0
x=3 Identify restrictions by making the denominator equal
to 0
𝑥+6= 0
x=-6 identify the values of x that make the numerator equal to
0
Since -6 make the numerator zero but not restrictions of
the rational function f, therefore -6 is the zero of the
function.

62
Note: Practice Personal Hygiene protocols at all times
Vertical asymptote
𝑥−3= 0
x=3
Horizontal asymptote
y= 6/-3
y = -2

2. f(x)= (𝑥 2 − 9)/(𝑥 2 − 𝑥 − 6)
For x intercept
f(x)= (𝑥 2 − 9)/(𝑥 2 − 𝑥 − 6) Factor
f(x)= (𝐱 − 𝟑)(𝑥 + 3)/ (𝒙 − 𝟑)/(𝑥 + 2) Cancel (𝐱 − 𝟑)
f(x)= (𝑥 + 3)/ (𝑥 + 2)
f(0)= (𝑥 + 3)/ (𝑥 + 2) Substitute 0 for y or f(x)
0=(𝑥 + 3) Multiply both sides by (𝑥 + 2)
x=-3 Simplify
Therefore, the x intercept is -3
For y intercept
f(x)= (𝑥 2 − 9)/(𝑥 2 − 𝑥 − 6)
f(0)= (02 − 9)/(02 − 0 − 6) Substitute 0 for x
= -9/-6 Simplify
=3/2
Therefore, the y intercept is 3/2
For the zeros
f(x)= (𝑥 2 − 9)/(𝑥 2 − 𝑥 − 6)
f(x)= (x − 3)(𝑥 + 3)/ (𝑥 − 3)/(𝑥 + 2 ) Factor the numerator and denominator of f
f(x)= (𝐱 − 𝟑)(𝑥 + 3)/ (𝒙 − 𝟑)/(𝑥 + 2) Cancel (𝐱 − 𝟑)
𝑥 = 3 𝑎𝑛𝑑 𝑥 = − 2 Identify restrictions by making the denominator
equal to 0
𝑥 = 3 𝑎𝑛𝑑 𝑥 = − 3 identify the values of x that make the numerator
equal to 0
Since -3 make the numerator zero but not
restrictions of the rational function f, therefore -
3 is the zero of the function

63
Note: Practice Personal Hygiene protocols at all times
Vertical Asymptote
𝑥 = 3 𝑎𝑛𝑑 𝑥 = − 2
Horizontal asymptote: Y=1

Exercise 1: Know Me Better


Directions: Identify which of the following are rational functions.
5. f(x)= (4 + 𝑥)/(𝑥 + 3)
6. f(x)= (𝑥 2 − 4𝑥 − 5)⁄(𝑥 − 1)
7. g(x)=(√𝑥 + 3)⁄(𝑥 2 − 1)
3
8. i(x)= 5 + 𝑥 − 4𝑥 2⁄𝑥 2 + √2𝑥 + 1
Exercise 2. Place Me on the Table
Directions: Find (a) the zeros; (b) the x-and y-intercepts and (c) the asymptotes of the
following rational functions.
Rational Function Zeros x-and y- Asymptotes
Intercepts
1. f(x)= (𝑥 − 5)/(𝑥 + 2)

2. f(x)= (𝑥 − 5)/(𝑥 2 − 25)

3.f(x)= (𝑥 2 − 5𝑥 + 4)/(𝑥 2 − 4𝑥 + 4)

.
Exercise 3: How Well Did I understand?
Create a rational function with a vertical asymptote of x= -5 and a hole at x=4

Reflection: I learned in this topic that rational function is_____________________


______________________________________________________________

References:
Oronce, Orlando. RBS General Mathematics First Edition
Learner’s Material for Mathematics Grade 9

64
Note: Practice Personal Hygiene protocols at all times
Answer key
Activity 1: Know Me Better
1. Rational Function: Both the numerator and denominator are polynomials.
2. Rational Function: Both the numerator and denominator are polynomials.
3. Not a Rational Function: Numerator is not a polynomial
4. Rational Function: Denominator is not polynomial.
Activity 2: Place Me On the Table
Zeros x-and y-Intercepts Asymptotes
1 X=5 x-intercept: (5,0) Vertical Asymptote: x=-2
y-intercept: ( 0,-5/2) Horizontal Asymptote: y=1
2 The function f has x-intercept: ( 5, 0) Vertical Asymptote: x=-5
no zero
y-intercept: ( 0, 1/5) Horizontal Asymptote: y=0
3 X=4 and x=1 x-intercept: (4,0) and (1,0) Vertical Asymptote: x=2
y-intercept: ( 0,1) Horizontal Asymptote: y=1

Activity 3: How Well Did I understand?


There are many possible answers (Hint: Be sure that one factor in the numerator is x-4 and the
denominator is (x+5 ) and (x-4)
Illustrations:
𝑥 2 −3𝑥−4
1. f(x)=(𝑥 − 4)(𝑥 + 1)⁄(𝑥 + 5)(𝑥 − 4) 𝑜𝑟 f(x)= 𝑥2 +𝑥−20
𝑥 2 _______
2. f(x)=(𝑥 − 4)(______)⁄(𝑥 + 5)(𝑥 − 4) 𝑜𝑟 f(x)=
𝑥 2 +𝑥−20
𝑥 2 ________
3. f(x)=(𝑥 − 4)(_______)⁄(𝑥 + 5)(𝑥 − 4) 𝑜𝑟 f(x)= 𝑥2 +𝑥−20

65
Note: Practice Personal Hygiene protocols at all times
GENERAL MATHEMATICS 11
Name of Learner: ___________________ Grade Level: ______
Section: ___________________________ Date : ___________

LEARNING ACTIVITY SHEET


SOLVING PROBLEMS INVOLVING RATIONAL FUNCTIONS,
EQUATIONS AND INEQUALITIES
Background Information of Learners
Solving problem involving rational functions, equations and inequalities requires an
analytical ability, creative thinking, logical reasoning and critical thinking. Hence, it is the
real-life application of the concepts you have learned in solving rational functions, equations
and inequalities.

Learning Competency
Solve Problems Involving Rational Functions, Equations and Inequalities
(Quarter 1, Week 2, M11GM-1c-3)

Illustrative Examples:
1. Mark can do a job in 4 days. When Mark and Jerry work together, it would take them
2 1/3 days. Find the number of days if Jerry will work alone?
Let X= be the numbers of days will Jerry work alone
Total Number of Days Fractional Part of the Job
Needed to Do a Job Done in 1 Day
Mark 4 ¼
Jerry X 1/x
Together 2 1/3 = 7/3 3/7

Since this is a work problem, it is assumed that the worker works at a fixed rate. Hence if a
worker needs n days to finish a job and he works at it for d days then the part of the job done
is d/n. therefore the equation is
¼ + 1/x= 3/7
7x + 28= 12 x Multiply both sides of the equation by the LCD: 28X
5x =28 Solving for x
X= 28/5 or 5 3/5 days.

66
Note: Practice Personal Hygiene protocols at all times
2. Annie leaves Santo Tomas for Baguio averaging 60 kph in her car. One hour later, JM
leaves for Baguio following the same route and travelling at an average speed of
80kph. How long would it take the JM to overtake Annie?

Let x= number of hours it takes JM to overtake Annie

Time Rate Distance


Annie x+1 60 60 (x+1)
JM x 80 80x

Equation :
60 (x+1)= 80x
60x + 60 = 80x Distributive Property
20x=60 Simplify
x= 3 hours (This means that JM will overtake Annie’s car in 3 hours. )
3. The length of a rectangular table in Mrs. Salas room is 20 cm. What widths will it
give a perimeter less than 160cm?
Solution: Let L= the length of the rectangular table, L=20
P= perimeter of the rectangular table, P< 160
W= width of the rectangular table
Perimeter of a Rectangle: P= 2L + 2W
Since the problem tells the perimeter is less than 160,
Hence: 2L + 2W<160
2(20) + 2w< 160 by Substitution
40 + 2w< 160
-40 +40 + 2w<-40 +160 Addition Property of Inequality
0 +2w<120 AIP
2w<120 IPA
½ (2w)< ½ (120) Multiplicative Inverse Property
w<60
Thus, the width of the rectangular table should be between 0 to 60 in as much as the
width cannot be 0 or less than 0.

67
Note: Practice Personal Hygiene protocols at all times
Exercise 1: Do You Still Remember?
Directions: Read the questions carefully, then identify what is being asked.
1. What is the reciprocal of 7?
2. What is the reciprocal of x?
3. What is the reciprocal of -10?
4. How do you find the reciprocal of a certain number?
5. What is the Least Common Denominator (LCD) of the equation 𝑥/5 + 1/4 =𝑥/2?
6. To clear the equation of fractions, what do we multiply to both sides of the equations?
Exercise 2: Step-by-Step!
Directions: Read and analyze the problems carefully then solve accurately the questions in
each of the given word problem.
1. The product of the reciprocals of two positive consecutive integers is 1/56. Find the
two integers.

Questions:
a. What is the first integer?
b. What is the second integer?
c. What is the resulting equation in the problem?
d. What is the Least Common Denominator (LCD) of the resulting equation?
e. What are the two consecutive integers?

2. Nognog a Grade 11 student can clean the room in 2 hours. His friend Christian can
clean the same room in 3 hours. How long will it take them to clean the room if they
work together?

Questions:
a. What is the fractional part of the job done in 1 hour by Nognog?
b. What is the fractional part of the job done in 1 hour by Christian?
c. What is the fractional part of the job done if they work together?
d. What is the resulting equation of the given problem?
e. What is the LCD of the resulting equation?
f. How long will it take them to clean the room if they work together?

3. A TVL student specialized in dressmaking ordered several red cloth from a vendor,
but the vendor only had 4 meters red cloth in stock. The vendor bought the remaining
lengths of green cloth from a wholesaler of php 1, 120.00. He then sold those lengths
of red cloth to the TVL student along with the original 4 meters of red cloth for a total
of php 1,600.00. If the vendor’s price per meter is at least 10 more than the
wholesaler’s price per meter, how many additional meters of red cloth did the vendor
purchase from the wholesaler?
Questions:
a. What is the length of the additional cloth purchased by the vendor from the
wholesaler?

68
Note: Practice Personal Hygiene protocols at all times
b. Expressed as rational expression the wholesaler’s price of the red cloth per meter.
c. Expressed as rational expression the vendor’s price of the red cloth per meter.
d. What is the resulting inequality basing from the problem?
e. What the LCD of the rational inequality?
f. What is the reduced form of the inequality?
g. What are the factors of the of n(x)?
h. Basing from the table of signs (intervals), how many meters does the vendor
bought and sold an additional length of red cloth to the TVL student?

Exercise 3: Represent then Solve!


Directions: Answer the following problems with accuracy. Show your complete solutions.
1. Jonel can clean a paint in 5 ½ hours. His brother can do same job in 7 ½ hours. How
long will it take them to paint the room together?
2. Two planes leave at the same time. One flies west at 300 km per hour. The other flies
east at 450 km per hour. In how many hours will they be 2 250 kilometers apart?
3. 50 pesos more than twice Laarni’s allowance is atmost 1200. What is her maximum
allowance?

Activity 4: Performance Tasks


Directions: Formulate at least two (2) problems related to our topic and solve.
Rubrics
CRITERIA 4 3 2 1
Mathematical Has shown full Has shown full Has shown Has no
Concept Used understanding understanding some degree of knowledge of
of the concept of the concept understanding the concept as
as evidenced in as evidenced in of the concept evidenced in
the work the work as evidenced in the work
presented. Also presented. the work presented
used other presented.
concept leading
to the same
solution
Accuracy of The The The The
Computations computations computations computations computations
were presented were presented were presented were incoherent
logically and logically and logically but and erroneous
done accurately. done accurately there were
Has presented minimal errors
alternative in the
solution/s computations.
leading to the
same answer

69
Note: Practice Personal Hygiene protocols at all times
Reflection
Complete the table below by answering the questions.
How do I find the What are the How did I learn How will I use these
performance task? values I learned them? What made learning/insights in
from the the task successful? my daily life?
performance task?

References:
Alferez, Merile S. et.al, MSA Advanced Algebra
DepEd General Mathematics Learner’s material
DepEd Learner’s Material for Grade 9
Ibe, Milagros D., et.al; BMS High School Mathematics Concepts and Operations
Oronce, Orlando A., RBS General Mathematics First Edition

Answer Key
Exercise 1
1. 1/7
2. 1/x
3. -1/10
4. a=1/a; -a=-1/a
5. 20
6. Least Common Multiple (LCD)
Exercise 2
1. a. x
b. x+1
c. (1/x)+(1/x+1)=1/56 Answers vary
d. (x)(x+1)(56) Answers vary
e. 7 and 8
2. a. ½ hr
b. 1/3 hr
c. 1/x
d. ½ +1/3=1/x
e. 6x
f. 1 1/5 hrs or 1 hr 12 minutes
3. a. x
b. 1120/x
c. 1600/x+4
1600 1120
d. 𝑥+4 ≥ 𝑥 +10
e. (x+4)(x)

70
Note: Practice Personal Hygiene protocols at all times
160 112
f. 𝑥+4
- 𝑥
≥1
g. (x-16)(x-28)
h. 16-18 meters
Exercise 3.
1. 3 9/52 hours
2. 3 hours
3. Maximum allowance is at most php 574

71
Note: Practice Personal Hygiene protocols at all times
GENERAL MATHEMATICS 11
Name of Learner:__________________________ Grade Level:________
Section:_________________________________ Score:_____________

LEARNING ACTIVITY SHEET


ONE- TO- ONE FUNCTIONS

Background Information for Learners


In this activity sheet we introduce another concept of a function. In general terms, a
function defines how one variable depends on one or more other variables. These variables
may be linked by some type of relationship.

Definition
A function 𝒇 is one–to-one if no two elements in the domain of 𝒇 correspond to the
same element in the range of 𝒇. This means, that each x-value must be matched to one and
only one y-value.
Algebraically, if 𝒇(𝒙𝟏 ) = 𝒇(𝒙𝟐 ) implies that, 𝒙𝟏 = 𝒙𝟐 then 𝒇 is one-to-one.

Determine whether the following examples are one-to-one or not. Explain why.
Example 1: 𝑔(𝑥 ) = {(0,2), (1,3), (2,4), (3,5)}
The function g is one-to-one since for every x value, correspond to unique y value.

Example 2: The relation pairing each learner to his or her Learner Reference Number (LRN).
Each learner is assigned to a unique Learner Reference Number (LRN). Further, no two
different learners have the same Learner Reference Number. Thus, the function is one-to-one.
Example 3. 𝑔(𝑥) = 2𝑥 + 1
If 𝑔(𝑎) = 𝑔(𝑏) implies that 𝑎 = 𝑏
2(𝑎) + 1 = 2(𝑏) + 1
2𝑎 = 2𝑏
𝑎=𝑏
Thus, g (x) is a one-to-one function.

72
Note: Practice Personal Hygiene protocols at all times
Graph of One- to- One Function

A graph of a function can also be used to determine whether a function is one-to-one


using the horizontal line test.

Horizontal Line Test


A function is one-to-one if each horizontal line does not intersect the graph at
more than one point.

y=x2

y=2x+1

A horizontal line intersects the graph A horizontal line intersects the graph
once; the function is one-to-one. twice; the function is not one-to-one.

Learning Competency
The learner is able to represent real-life situations using one-to-one functions.
M11GM-Id-1

Exercise 1.
Determine whether the following functions are one-to-one. (1 point each)
1. f(x)= {(2, 27), (3, 28), (4, 29), (5, 30)}
2. h(x)= {(11,14), (12, 14), (16, 7), (18, 13)}
3. g(x)= {(3, 12), (4, 13), (6, 14), (8, 1)}
4. x 1 2 3 4 5 6
f(x) 1.5 2.0 3.6 5.3 2.8 2.0

x 1 2 3 4 5 6
5.
f(x) 1.0 1.9 2.8 3.5 2.1 2.9
6. f(x) = 3x − 4
1
7. ℎ(𝑥 ) = 𝑥

73
Note: Practice Personal Hygiene protocols at all times
8. r(x) = x 2 − 2
9. g(x) = x 3

10. h
-3
0
-1
2
0
4
1

Exercise 2.
Use the Horizontal Line Test to determine whether each graph represents a one-to-one function.
Write your explanation on the box provided in each item. (2 points each)
1. 2. 3.

4.

74
Note: Practice Personal Hygiene protocols at all times
5. 6.

7. 8

9. 10.

75
Note: Practice Personal Hygiene protocols at all times
Rubric

Score Description
2 Answer is complete and correct; all parts of the question are addressed
1 Student gives a partially correct answer, or task is incomplete (i.e., one of two
parts answered correctly
0 Does not address task, unresponsive, unrelated or inappropriate

Source: North Carolina Department of Public Instruction


https://web.njit.edu/~ronkowit/presentations/rubrics/samples/math_probsolv_chicago.pdf

Exercise 3.
Determine whether the given situation represent a one-to-one function. (2 points each)

1. The force (measured in pounds) to stretch a certain spring x inches is given f(x)=3x.
2. The velocity in feet per second of a falling object is given by v(t) =-32t where t is the
time in seconds after the object was released.
3. The relation pairing a real number to its square.
4. The percent y (in decimal form) of battery power remaining x hours after you turn on a
laptop computer is y = −0.2 x + 1.
5. The function below represents the shipping charges that applies to orders from an
online company, where f(x) represents the shipping charges and x represents the total
cost of orders
300 0 < 𝑥 ≤ 3000
{
f(x)= 200 3000 ≤ 𝑥 ≤ 7000}
150 𝑥 > 7000
6. Susana invested an amount of 350,000 in her bank account with an interest rate of
4%. After three years, she earned an interest of 42,000; in 5 years, she earned 70,000.
7. The mobile applications like Facebook, Messenger, Shareit installed in a cellular phone.
8. The relation pairing a town to its zip code.
9. If Alexander rides a bike at an average of 11.5 mph, the distance that he rides can be
represented by d(t)=11.5t, where t is the time in hours.
10. Ms. Reyes is buying t-shirts for the math club officers. The t-shirt company will charge
her according to the function below, where c(x) is the total cost of the shirts and x is
the number of t-shirts she orders
12𝑥 0 < 𝑥 ≤ 10
c(x)= {10𝑥 11 ≤ 𝑥 ≤ 20}
8𝑥 𝑥 > 20

76
Note: Practice Personal Hygiene protocols at all times
Exercise 4.
Cite a real-life situation that can be represented by one-to-one function and explain why
is it important that the function is one-to-one.

Reflection
What insights and learnings have you gained in this topic?
___________________________________________________________________________
___________________________________________________________________________
___________________________________________________________________________
___________________________________________

Reference for Learners


Verzosa, D.B, et.al (2016). General Mathematics. Quezon City, Manila

77
Note: Practice Personal Hygiene protocols at all times
Answer Key
Exercise 1.
1. One-to-one function 6. One-to-one function
2. Not one-to-one function 7. One-to one function
3. One-to-one function 8. Not one-to-one function
4. Not one-to-one function 9. One-to-one function
5. One-to one function 10. Not one-to-one function
Exercise 2.
1. One-to-one, the horizontal line crosses the graph at exactly one point.
2. Not one-to-one, the horizontal line intersects the graph twice.
3. One-to-one, the horizontal line passes the graph once.
4. One-to-one, the horizontal line crosses the graph twice.
5. Not one-to-one, the horizontal line intersects the graph twice
6. Not one-to-one, the horizontal line intersects the graph twice
7. One-to-one, the horizontal line crosses the graph twice
8. Not one-to-one, the horizontal line intersects the graph twice
9. Not one-to-one, the horizontal line intersects the graph twice
10. One-to-one, the horizontal line passes the graph once.

Exercises 3.
1. One-to-one
2. One-to-one
3. Not One-to-one
4. One-to-one
5. Not One-to-one
6. One-to-one
7. Not One-to-one
8. One-to-one
9. One-to-one
10. Not One-to-one
11. Exercise 4.
12. Many possible answers.

78
Note: Practice Personal Hygiene protocols at all times
GENERAL MATHEMATICS 11
Name of Learner:_____________________ Grade Level:________
Section:_____________________________ Score:_____________

LEARNING ACTIVITY SHEET


INVERSE OF ONE-TO-ONE FUNCTIONS

Background Information for Learners


We have learned that a function can be regarded as taking an input, x, and processing
it in some way to produce a single output f(x). This time, we will find another function that
will start with f(x) and process it to produce x again.

Definition
Let 𝒇 be a one-to-one function with domain 𝑨 and range 𝑩. Then the inverse of 𝒇,
denoted 𝒇−𝟏 , is a function with domain 𝑩 and range 𝑨 defined by 𝒇−𝟏 (𝒚) = 𝒙 if and only if
𝒇(𝒙) = 𝒚 for any 𝒚 in 𝑩 .

𝑓𝑖𝑔𝑢𝑟𝑒 1. 𝑓 −1 reverses the process in 𝑓

Note: A function has an inverse if and only if it is one-to-one function.

Finding the inverse of a one-to-one function


1. Write the function in the form 𝒚 = 𝒇(𝒙);
2. Interchange 𝒙 and 𝒚 variables;
3. Solve for 𝒚 in terms of 𝒙 . The resulting equation is 𝒇−𝟏 (𝒙).

Example 1. Find the inverse of 𝒇(𝒙) = 𝟖𝒙 + 𝟓


Solution.
𝑦 = 𝑓 (𝑥 ) 𝑓 (𝑥 ) = 8𝑥 + 5 𝑦 = 8𝑥 + 5
Interchange 𝒙 and 𝒚 variables 𝑦 = 8𝑥 + 5 𝑥 = 8𝑦 + 5
Solve for 𝒚 in terms of 𝒙 𝑥 = 8𝑦 + 5
𝑥 − 5 = 8𝑦

79
Note: Practice Personal Hygiene protocols at all times
𝑥−5 𝑥−5
=𝑦 𝑦= .
8 8
𝑥−5
Therefore, the inverse of 𝑓 (𝑥 ) = 8𝑥 + 5 is 𝑓 −1 (𝑥 ) = 8
.

𝑥−5
Check whether 𝑓(𝑥 ) = 8𝑥 + 5 and 𝑓 −1 (𝑥 ) = are inverses.
8

suppose x=3 in 𝑓 (𝑥 ).

𝑓 (𝑥 ) = 8𝑥 + 5 𝒇(𝟑) = 𝟖(𝟑) + 𝟓 = 𝟐𝟗
𝑥−5 𝒙−𝟓
𝑓 −1 (𝑥 ) = 𝒇−𝟏 (𝟐𝟗) = =𝟑
8 𝟖

Hence, the two functions are inverses of each other.

Learning Competency

The learner is able to determine the inverse of a one-to-one function. M11GM-Id-2

Exercise 1.
State if the given functions are inverses. (2 points each)
𝑥−5 −2−2𝑥
1. 𝑓 (𝑥 ) = 10 6. 𝑓 (𝑥 ) = 𝑥
𝑔(𝑥 ) = 10𝑥 + 5 −2
𝑔 (𝑥 ) =
𝑥+2
8+9𝑥
2. 𝑓 (𝑥 ) = 2 3
5𝑥 − 9 7. 𝑓 (𝑥 ) = 4 − 2 𝑥
𝑔 (𝑥 ) =
2 1 3
𝑔 (𝑥 ) = 𝑥+
2 2
5 𝑥−1
3. 𝑓 (𝑥 ) = √ 8. 𝑓 (𝑥 ) = 4𝑥 + 16
2
𝑔(𝑥 ) = 2𝑥 + 1 5 −16 + 𝑥
2
𝑔 (𝑥 ) =
4. 𝑓 (𝑥 ) = 𝑥+3 4
9. 𝑓 (𝑥 ) = 3𝑥 + 1
3𝑥 + 2
𝑔 (𝑥 ) = 𝑔 (𝑥 ) = 3 + 𝑥
𝑥+2
2
4−𝑥 10. 𝑓 (𝑥 ) = − 𝑥 − 1
5. 𝑓 (𝑥 ) = 𝑥
4 −2
𝑔 (𝑥 ) = 𝑔 (𝑥 ) =
𝑥 𝑥+1

80
Note: Practice Personal Hygiene protocols at all times
Exercise 2.
Find the inverse of each function. (4 points each)
1. 𝑓 (𝑥 ) = 𝑥 − 6 2. 𝑔(𝑥 ) = −5𝑥 + 1

4
3. ℎ(𝑥 ) = 𝑥+2 4. 𝑓 (𝑥 ) = (𝑥 + 3)3

5. 𝑔(𝑥 ) = 2𝑥 3 + 3 3
6. 𝑓 (𝑥 ) = 𝑥−4

81
Note: Practice Personal Hygiene protocols at all times
−2𝑥+1 7−3𝑥
7. 𝑔 (𝑥 ) = 8. ℎ(𝑥 ) =
3 𝑥−2

4
9. 𝑓 (𝑥 ) = √
5 −𝑥+2 10. 𝑔(𝑥 ) = 5 𝑥 − 4
2

82
Note: Practice Personal Hygiene protocols at all times
Exercise 3.
Identify whether the inverse of each function is a function or not. Explain briefly your answer.
(4 points each)
1
1. 𝑓 (𝑥 ) = 2 𝑥 + 1
2. 𝑓 (𝑥 ) = 𝑥 2 + 6𝑥 + 9
3. 𝑓 (𝑥 ) = |2𝑥 |

Reflection
What significant learnings have you acquired in this lesson?
___________________________________________________________________________
___________________________________________________________________________
___________________________________________________________________________
___________________________________________

Reference for Learners


Verzosa, D.B, et.al (2016). General Mathematics. Quezon City, Manila
https://cdn.kutasoftware.com/Worksheets/Alg2/Function%20Inverses.pdf

83
Note: Practice Personal Hygiene protocols at all times
Answer Key
Exercise 1.

𝑥−5 −2−2𝑥
1. 𝑓 (𝑥 ) = 10 6. 𝑓(𝑥 ) = 𝑥
𝑔(𝑥 ) = 10𝑥 + 5 YES −2
𝑔(𝑥 ) = 𝑥+2 YES
8+9𝑥
2. 𝑓 (𝑥 ) = 2
5𝑥−9 3
𝑔 (𝑥 ) = NO 7. 𝑓(𝑥 ) = 4 − 𝑥
2
2𝑥 − 8 2
𝑓 −1 (𝑥 ) = 1 3
9 𝑔 (𝑥 ) = 2 𝑥 + 2 NO
5 𝑥−1
3. 𝑓 (𝑥 ) = √ 2 8 − 2𝑥
5 𝑓 −1 (𝑥 ) =
𝑔(𝑥 ) = 2𝑥 + 1 YES 3

2
4. 𝑓 (𝑥 ) =
𝑥+3
3𝑥+2 8. 𝑓(𝑥 ) = 4𝑥 + 16
𝑔 (𝑥 ) = NO
𝑥+2 −16+𝑥
2 − 3𝑥 𝑔 (𝑥 ) = YES
𝑓 −1 (𝑥 ) = 4
𝑥
4−𝑥
5. 𝑓 (𝑥 ) = 𝑥
4
𝑔 (𝑥 ) = 𝑥 NO 9. 𝑓(𝑥 ) = 3𝑥 + 1
4
𝑓 −1 (𝑥 ) = 𝑔 (𝑥 ) = 3 + 𝑥 3 NO
𝑥+1
𝑥−1
𝑓 −1 (𝑥 ) =
3
2
10. 𝑓 (𝑥 ) = − 𝑥 − 1
−2
𝑔(𝑥 ) = 𝑥+1 YES

Exercise 2.
1. 𝑓(𝑥) = 𝑥 − 6
4
𝑓 −1 (𝑥 ) = 𝑥 + 6 3. ℎ(𝑥 ) = 𝑥+2

4 − 2𝑥
ℎ−1 (𝑥 ) =
2. 𝑔(𝑥 ) = −5𝑥 + 1 𝑥
4. 𝑓 (𝑥 ) = (𝑥 + 3)3
−𝑥 + 1
𝑔−1 (𝑥 ) = 𝑓 −1 (𝑥 ) = 3√𝑥 − 3
5
84
Note: Practice Personal Hygiene protocols at all times
5. 𝑔(𝑥 ) = 2𝑥 3 + 3
3 𝑥−3
𝑔−1 (𝑥 ) = √
2

3
6. 𝑓 (𝑥 ) = 𝑥−4

4𝑥 + 3
𝑓 −1 (𝑥 ) =
𝑥

−2𝑥+1
7. 𝑔(𝑥 ) = 3
−3𝑥 + 1
𝑔−1 (𝑥 ) =
2

7−3𝑥
8. ℎ(𝑥 ) = 𝑥−2
2𝑥 + 7
ℎ−1 (𝑥 ) =
𝑥+3

5 −𝑥+2
9. 𝑓 (𝑥 ) = √ 2
−1 (
𝑓 𝑥 ) = −2𝑥 5 + 2
4
10. 𝑔(𝑥 ) = 5 𝑥 − 4

5
𝑔−1 (𝑥 ) = 5 + 𝑥
4

85
Note: Practice Personal Hygiene protocols at all times
Exercise 3.

1
1. 𝑓 (𝑥 ) = 2 𝑥 + 1
𝑓 −1 (𝑥 ) = 2𝑥 − 2
The given function is one-to-one function; thus, its inverse is also a one-to-one function.

2. 𝑓 (𝑥 ) = 𝑥 2 + 6𝑥 + 9
The given function is a quadratic function; thus, it is not a one-to-one function.
Therefore, the function 𝑓(𝑥 ) = 𝑥 2 + 6𝑥 + 9 has no inverse function.

Or

f(x)= x2+6X+9
y= x2+6X+9
x=y2+6y+9
x-9= y2+6y
x= y2+6y+9
x=(y+3)2
±√𝑥 = 𝑦 + 3
𝑦 = ±√𝑥 − 3

The equation does not represent a function because there are some x-value that
correspond to two different y-values (if x=4; y can be -1 or -5). Therefore, the function
f(x)= x2+6X+9 has no inverse function.
3. 𝑓 (𝑥 ) = |2𝑥 |
y= |2𝑥 |
x= |2𝑦|
x= √(2𝑦)2 |𝑥 | = √𝑥 2

𝑥2
= 𝑦2
2
𝑥2
𝑦 = ±√ 2 ; This is not a function, therefore, 𝑓 (𝑥 ) = |2𝑥 |, has no inverse function.

86
GENERAL MATHEMATICS 11
Name of Learner: ________________________________ Grade Level: _________
Section: ________________________________________ Score: ______________

LEARNING ACTIVITY SHEET


GRAPH OF INVERSE FUNCTIONS

Background Information for Learners

If f is a one-to-one function consisting of the ordered pairs (𝒙, 𝒚) of real numbers, then
i) the inverse of 𝒇 is the set 𝒇−𝟏 = {(𝒚, 𝒙): (𝒙, 𝒚)𝝐 𝒇},
ii) 𝒇−𝟏 is also a one-to-one function and its inverse is f,
iii) the domain of 𝒇−𝟏 is the range of 𝒇 and the range of 𝒇−𝟏 is the domain of 𝒇,
and
iv) the graphs of 𝒇 and 𝒇−𝟏 are symmetric with respect to the line y = x.

Examples
1. Find the domain and range of 𝑓(𝑥) = {(1,17), (2,11), (3,5), (4, −1), (5, −7)} and its
inverse.
𝑓(𝑥) = {(1,17), (2,11), (3,5), (4, −1), (5, −7)}
𝐷: {1, 2, 3, 4, 5}
𝑅: {17, 11, 5, −1, −7}

𝑓 −1 (𝑥) = {(17,1), (11,2), (5,3), (−1,4), (−7,5)}


𝐷: {17, 11, 5, −1, −7}
𝑅: {1, 2, 3, 4, 5}

2. Find the inverse of the function 𝑓(𝑥) = 5 − 9𝑥 then draw the graph in one Cartesian
plane.
𝑓(𝑥) = 5 − 9𝑥
5−𝑥
The inverse of 𝑓(𝑥) is 𝑔(𝑥) =
9
𝑥 𝑓(𝑥) 𝑥 𝑔(𝑥)
-2 23 23 -2
-1 14 14 -1
0 5 5 0
1 -4 -4 1
2 -13 -13 2

87
Learning Competency with code
The learner is able to represent an inverse function through its table of values and graph and
find the domain and range of an inverse function (M11GM-Id-3-4,Quarter 1)

Directions/Instructions:
A. Find the domain and the range of each function and its inverse.

1. 𝑓 (𝑥 ) = {(3, 15), (4, 9), (5, 3), (6, −3), (7, −9)}
2. 𝑔(𝑥 ) = {(3, 3), (4, 4), (5, 5), (6, 6), (7, 7)}
3. ℎ(𝑥 ) = {(3, −9), (4, −3), (5, 3), (6, −3), (7, −9)}
4. 𝑖 (𝑥 ) = {(2, 5), (5, 2), (3, 7), (7, 3)}
5. 𝑗(𝑥 ) = {(0, 0), (2, 6), (4, 12), (6, 18)}

B. Draw the graph of the function and its inverse using the same set of axes. Label the
graphs.

1. 𝑓 (𝑥 ) = 3𝑥 + 1
3–𝑥
2. 𝑓 (𝑥 ) = 4
3. 𝑓 (𝑥 ) = 2𝑥 + 1
4. 𝑓 (𝑥 ) = 𝑥 3
𝑥−2
5. 𝑓 (𝑥 ) = 𝑥+2

88
C. Find the domain and the range of the inverse of each function.

3𝑥+2
1. 𝑓 (𝑥 ) = 𝑥−4

2. 𝑓 (𝑥 ) = 𝑥 2 − 1, 𝑥 ≥ 0
3. 𝑓 (𝑥 ) = 4 − 𝑥 2
4. 𝑓 (𝑥 ) = 3 − 5𝑥
5. 𝑓 (𝑥 ) = 𝑥 + 8

D. If 𝑓 (𝑥 ) = 2𝑥, evaluate the following:

1. 𝑓 −1 (2)
2. 𝑓(2−1 )
3. 𝑓 −1 (2−1 )
4. [𝑓 −1 (2)]-1
5. [𝑓 −1 (2−1 )]-1

Reflection

Complete this statement:


What I learned in this activity
___________________________________________________________________________
___________________________________________________________________________
___________________________________________________________________________
___________________________________________________________________________
____________

References:

Orines, Fernando B. Next Century Mathematics (General Mathematics).Phoenix


Publishing House, Inc.2016

Oronce, Orlando A. General Mathematics. Rex Book Store.2016

89
Answer Key
A
1. 𝑓 (𝑥 ) = {(3, 15), (4, 9), (5, 3), (6, −3), (7, −9)}
𝐷: {3,4,5,6,7} R: {15,9,3, −3, −9}
−1 ( )
𝑓 𝑥 = {(15, 3), (9, 4), (3, 5), (−3,6), (−9, 7)}
𝐷: {15,9,3, −3, −9} R: {3,4,5,6,7}

2. 𝑔(𝑥 ) = {(3, 3), (4, 4), (5, 5), (6, 6), (7, 7)}
𝐷: {3,4,5,6,7} R: {3,4,5,6,7}
−1 ( )
𝑔 𝑥 = {(3, 3), (4, 4), (5, 5), (6,6), (7, 7)}
𝐷: {3,4,5,6,7} R: {3,4,5,6,7}

3. ℎ(𝑥 ) = {(3, −9), (4, −3), (5, 3), (6, −3), (7, −9)}
𝐷: {3,4,5,6,7} R: {−9, −3, 3}
−1 ( )
ℎ 𝑥 = {(−9, 3), (−3, 4), (3, 5), (−3,6), (−9, 7)}
𝐷: {−9, −3,3} R: {3,4,5,6,7}

4. 𝑖 (𝑥 ) = {(2, 5), (5, 2), (3, 7), (7, 3)}


𝐷: {2, 5, 3, 7} R: {5, 2, 7, 3}
−1 ( )
𝑖 𝑥 = {(5, 2), (2, 5), (7, 3), (3, 7)}
𝐷: {5, 2, 7, 3} R: {2, 5, 3, 7}

5. 𝑗(𝑥 ) = {(0, 0), (2, 6), (4, 12), (6, 18)}


𝐷: {0, 2, 4, 6} R: {0, 6, 12, 18}
−1 ( )
𝑗 𝑥 = {(0, 0), (6, 2), (12,4), (18,6)}
𝐷: {0, 6, 12, 18} R: {0, 2, 4, 6}

1.

90
2.

3.

4.

91
5.

C
1. 𝐷: {𝑥/𝑥 ≠ 3}
𝑅: {𝑦/𝑦 ≠ 4}

2. 𝐷: {𝑥/𝑥 ≥ 0}
𝑅: {𝑦/𝑦 ≥ 0}

3. 𝐷: {𝑥/𝑥 ≤ 4}
𝑅: {𝑦/𝑦 ≥ 0}

4. 𝐷: {𝑥/𝑥 𝜖ℝ}
𝑅: {𝑦/𝑦 𝜖 ℝ}

5. 𝐷: {𝑥/𝑥 𝜖ℝ}
𝑅: {𝑦/𝑦 𝜖 ℝ}

D
1. 1
2. 1
3. ¼
4. 1
5. 4

92
GENERAL MATHEMATICS 11
Name of Learner: ____________________________ Grade Level: _______________
Section: ____________________________________ Date: _____________________

LEARNING ACTIVITY SHEET


Solving Exponential Equations and Inequalities
Background Information for Learners
In solving exponential equation, remember the property of equality for exponential
functions that if the base b is a positive number other than 1, then 𝑏 𝑥 = 𝑏 𝑦 if and only if
𝑥 = 𝑦.
Steps in Solving Exponential Equation
A. Rewrite both sides of the equation using the same base.
B. Remove the bases and set the exponents equal to each other.
C. Simplify and solve for the x variable.
Example 1. 2𝑥 = 64
2𝑥 = 26
𝑥=6
On the other hand, an exponential inequality is an inequality involving exponential
expressions. It is very important to consider the properties of exponential inequalities which
are indicated on the table.
If 𝑏 > 1 If 0 < 𝑏 < 1
then 𝑏 𝑥 > 𝑏 𝑦 if and only if 𝑥 > 𝑦 then 𝑏 𝑥 > 𝑏 𝑦 if and only if 𝑥 < 𝑦
and 𝑏 𝑥 < 𝑏 𝑦 if and only if 𝑥 < 𝑦 and 𝑏 𝑥 < 𝑏 𝑦 if and only if 𝑥 > 𝑦
Steps in solving exponential inequality
A. Rewrite the inequality using the same base.
B. Remove the bases and set the exponents.
C. Reverse the inequality sign if the base is greater 0 but less than 1. If the base is greater
than 1, the direction of inequality sign is retained.
D. Simplify and solve for the x variable.
Example 2. 43𝑥+2 < 64
43𝑥+2 < 43
3𝑥 + 2 < 3
3𝑥 < 1
1
𝑥<3
1
Solution: 𝑥 < 3
1
Interval Notation: ቀ−∞, 3 ቁ

Example 3. 0.33𝑥−2 < 0.3𝑥+2


3𝑥 − 2 > 𝑥 + 2
2𝑥 > 4
𝑥>2
Solution: 𝑥 > 2
Intervalwith
Learning Competency code (2, ∞)
Notation:
The learner is able to solve exponential equations and inequalities. M11GM-Ie-f-1

93
Directions
Below are the solved examples of exponential equation and inequalities, which will be your
basis in answering the exercises.
Solve 9𝑥−3 = 3𝑥+7 . Solve 23𝑥+4 > 32
1 1 3𝑥+1 1 𝑥+2
Solve ቀ2ቁ > ቀ 4ቁ

1 3𝑥+1 1 𝑥+2
9 𝑥−3
=3 𝑥+7 ( ) >( )
1 2 4
23𝑥+4 >
(3)2(𝑥−3) = 3𝑥+7 32 1 3𝑥+1 1 𝑥+2
1 ( ) > ( 2)
2(𝑥 − 3) = 𝑥 + 7 23𝑥+4 > 2 2
25
2𝑥 − 6 = 𝑥 + 7 1 3𝑥+1 1 2(𝑥+2)
2 3𝑥+4
>2 −5 ( ) >( )
2 2
𝑥 = 13
3𝑥 + 4 > −5 1 3𝑥+1 1 2𝑥+4
( ) >( )
3𝑥 > −9 2 2

𝑥 > −3 3𝑥 + 1 < 2𝑥 + 4
𝑥<3
EXERCISE 1. “MAKE AN ANALYSIS”
Directions: Make an analysis on the solutions presented in each item.
1. Ligaya and Mutya are solving 42𝑥 = 2𝑥+6 .
LIGAYA’S SOLUTION MUTYA’S SOLUTION
2𝑥 𝑥+6
4 =2 42𝑥 = 2𝑥+6
22(2𝑥) = 2𝑥+6 2𝑥 = 𝑥 + 6
4𝑥 = 𝑥 + 6 2𝑥 − 𝑥 = 𝑥 + 6 − 𝑥
3𝑥 = 6 𝑥=6
𝑥=2
Question: Did anyone arrived with the correct answer? If there is, identify whose solution
is right and explain why his/her solution is the correct one.
________________________________________________________________________
________________________________________________________________________
________________________________________________________________________
________________________________________________________________________

1 𝑥+3
2. Mario and Antonio are solving 64 = 8𝑥−5 .
MARIO’S SOLUTION ANTONIO’S SOLUTION
1 𝑥+3 1 𝑥+3
= 8𝑥−5 = 8𝑥−5
64 64
1 𝑥+3 1 𝑥+3
= 23(𝑥−5) = 23(𝑥−5)
26 26
26(𝑥+3) = 23𝑥−15 2−6(𝑥+3) = 23𝑥−15
6𝑥 + 18 = 3𝑥 − 15 −6𝑥 − 18 = 3𝑥 − 15
3𝑥 = −33 −3 = 9𝑥
𝑥 = −11 1
𝑥 = −3

94
Question: Did anyone arrived with the correct answer? If there is, identify whose solution
is right and explain why his/her solution is the correct one.
________________________________________________________________________
________________________________________________________________________
________________________________________________________________________
________________________________________________________________________
3. Lorie and Daniel are solving 0.53𝑥+5 ≥ 0.25𝑥−2 .
LORIE’S SOLUTION DANIEL’S SOLUTION
0.53𝑥+5 ≥ 0.25𝑥−2 0.53𝑥+5 ≥ 0.25𝑥−2
0.53𝑥+5 ≥ 0.52(𝑥−2) 0.53𝑥+5 ≥ 0.52(𝑥−2)
3𝑥 + 5 ≥ 2𝑥 − 4 3𝑥 + 5 ≤ 2𝑥 − 4
𝑥 ≥ −9 𝑥 ≤ −9
Question: Did anyone arrived with the correct answer? If there is, identify whose solution
is right and explain why his/her solution is the correct one.
________________________________________________________________________
________________________________________________________________________
________________________________________________________________________
________________________________________________________________________
3 2𝑥−2 125 𝑥+2
4. Eva and Dave are solving ቀ5ቁ ≥ ቀ 27 ቁ .
EVAS’S SOLUTION DAVE’S SOLUTION
3 2𝑥−2 125 𝑥+2 3 2𝑥−2 125 𝑥+2
( ) ≥( ) ( ) ≥( )
5 27 5 27
3 2𝑥−2 5 3(𝑥+2) 3 2𝑥−2 5 3(𝑥+2)
( ) ≥( ) ( ) ≥( )
5 3 5 3
2𝑥−2
3 3 −3(𝑥+2) 3 2𝑥−2
3 −3(𝑥+2)
( ) ≥( ) ( ) ≥( )
5 5 5 5
2𝑥 − 2 ≥ −3𝑥 − 6 2𝑥 − 2 ≤ −3𝑥 − 6
4
− ≥𝑥 4
5 ≤𝑥
5
Question: Did anyone arrived with the correct answer? If there is, identify whose solution
is right and explain why his/her solution is the correct one.
________________________________________________________________________
________________________________________________________________________
________________________________________________________________________
________________________________________________________________________

EXERCISE 2. “HOW DID YOU GET ME?”


Directions: Solve each of the following exponential equations and inequalities. Your task is to
show the step-by-step process on how it arrive with the given answer.
1. 625𝑥−1 = 25𝑥+5 1 2𝑥
2. ቀ ቁ = 216𝑥−5
6

95
Answer: 𝒙 = 𝟕 Answer: 𝒙 = 𝟑
3. 0.43𝑥+2 > 0.0643−2𝑥 1 3𝑥−3
4. ቀ49ቁ ≤ 343𝑥+2

Answer: 𝒙 < 𝟗
𝟕 Answer: 𝒙 ≥ 𝟎

EXERCISE 3. “SOLVE ME”


A. Solve the following exponential equation and inequalities.
[3 points each]
1 𝑥−2 1 1
1. 93𝑥−8 = ቀ81ቁ Hint: can be written as or 9−2
81 92

2. 162𝑥−4 ≥ 32𝑥−1
1 𝑥+2 1 𝑥−4
3. =ቀ ቁ
343 49
1 2𝑥+4
4. 27𝑥+3 ≥ ቀ9ቁ

B. Solve the following worded problems involving exponential equations.


[3 points each]
1. At t = 0, there were initially 960 grams of an isotope. The half-life of an isotope is
30 hours. How much time has passed when only 60 grams of the isotope were left?

2. It is 8 o’clock in the morning when a scientist starts examining 25 samples of germs


that triples every 4 hours. What time of the day will he found out that the initial 25
germs had already grown to 675?

Rubric for Scoring


For Exercise 1
CRITERIA 4 POINTS 3 POINTS 2 POINTS 1 POINT
Explanation Explanation clearly Explanation Explanation Explanation is
of solution indicates indicates indicates a present but
procedures are procedures are minimal demonstrates a
understood at an understood understanding lack of
advanced level of procedure understanding

96
POSSIBLE TOTAL POINTS: 16

For Exercise 2
CRITERIA 4 POINTS 3 POINTS 2 POINTS 1 POINT
Equation Initial equation Initial equation Incorrect solution Partially
Solution and all steps and most of the started
are shown steps are shown. showing the
Arrived at the solution, but
correct/incorrect did not finish
Arrived at the Arrived at the answer it.
correct answer correct answer
POSSIBLE TOTAL POINTS: 16

Reflection
What have you learned from this topic and how can you relate it to real life situations?
___________________________________________________________________________
___________________________________________________________________________
___________________________________________________________________________
___________________________________________________________________________
___________________________________________________________________________

References for Learners


Verzosa, D. et. Al. (2016). General Mathematics. Lexicon Press Inc.
https://www.mathwarehouse.com/algebra/exponents/solve-exponential-equations-how-to.php
https://www.mathemania.com/lesson/exponential-inequalities/

Answer Key
EXERCISE 1
1. Ligaya is correct because she rewrite the equation with the same base before setting the
exponents equal to each other. Mutya’s solution is incorrect. Mutya should rewrite both
sides of the equations as powers of the same base before computing the value of x. The
correct answer is 𝑥 = 2, which was obtained by Ligaya.
2. Antonio is correct and Mario is not. Antonio used the law of negative exponent which
1 𝑥+3
Mario failed to do when he transformed 26
= 23(𝑥−5) to 26(𝑥+3) = 23𝑥−15 . The
1
correct answer was obtained by Antonio which is 𝑥 = − 3.

97
3. Daniel is correct and Lorie is not. It is right to reverse the direction of inequality sign
since the base is less than 1. The correct answer is
𝑥 ≤ −9, which was attained by Daniel.
4. No one got the correct answer. Eva should have reversed the direction of the inequality
sign. Dave reversed the inequality sign but failed in simplifying his answer. The correct
3 2𝑥−2 125 𝑥+2 4
solution for the inequality ቀ5ቁ ≥ ቀ 27 ቁ is ≥ − 5.
EXERCISE 2
1. 625𝑥−1 = 25𝑥+5 1 2𝑥
2. ቀ6ቁ = 216𝑥−5
53(𝑥−1) = 52(𝑥+5)
1 2𝑥 1 −3(𝑥−5)
6−1(2𝑥) = 63(𝑥−5) or ቀ6ቁ = ቀ6ቁ
3(𝑥 − 1) = 2(𝑥 + 5)
3𝑥 − 3 = 2𝑥 + 10 −1(2𝑥) = 3(𝑥 − 5) 2𝑥 = −3(𝑥 − 5)

𝑥=7 −2𝑥 = 3𝑥 − 15 2𝑥 = −3𝑥 + 15


𝑥=3 𝑥=3

Answer: 𝒙 = 𝟕 Answer: 𝒙 = 𝟑

3. 0.43𝑥+2 > 0.0643−2𝑥 1 3𝑥−3


4. ቀ49ቁ ≤ 343𝑥+2
0.43𝑥+2 > 0.43(3−2𝑥)
1 3𝑥−3
3𝑥 + 2 < 3(3 − 2𝑥) ( 2) ≤ 73(𝑥+2)
7
3𝑥 + 2 < 9 − 6𝑥 7−2(3𝑥−3) ≤ 73(𝑥+2)
9𝑥 < 7 −2(3𝑥 − 3) ≤ 3(𝑥 + 2)
7 −6𝑥 + 6 ≤ 3𝑥 + 6
𝑥<
9
0 ≤ 9𝑥
0≤𝑥

𝟕
Answer: 𝒙 ≥ 𝟎
Answer: 𝒙 < 𝟗

98
EXERCISE 3
A.
2
1. 𝑥 = 2 5 𝑜𝑟 2.4
11 2
2. 𝑥 ≥ 3 𝑜𝑟 3 3 𝑜𝑟 3.6
3. 𝑥 = −14
17
4. 𝑥 ≥ − 7 𝑜𝑟 − 2.42857

B.
1. t = 120 hours
2. t = 12 hours, therefore, the germs had grown to 675 at 8 o’clock pm.

99
GENERAL MATHEMATICS 11
Name of Learner:____________________________ Grade Level:__________________
Section:____________________________________ Score:________________________

LEARNING ACTIVITY SHEET


REPRESENTING REAL-LIFE SITUATIONS USING EXPONENTIAL
FUNCTIONS
Background Information for Learners
Exponential functions are very useful in real-world situations like population growth,
population decay and compound interest.
One of the best applications of exponential function is the rapid growth of Corona Virus
Disease (COVID-19) pandemic that we are currently experiencing all over the world. The
exponential growth of this disease may increase or decrease depending on the actions taken by
our department of health.

Example 1:
At day 0, there was initially one person infected by the Corona Virus Disease (COVID-
19) pandemic, two are infected tomorrow, four the day after tomorrow, and eight the day after
that.
a. How many infected persons are there on the 4th,5th, 6th and 7th day. (Show the
relationship of the number of days and the number of people infected by COVID-19
using table of values).
b. What pattern can you observe from the table?
c. Derive a formula that describes the number of days as a function of the number of
people infected by COVID-19.

Solution:
a. +1 +1 +1 +1 +1 +1

Number of days (x) 0 1 2 3 4 5 6 7

Number of infected persons (y) 1 2 4 8 16 32 64 128

x2 x2 x2 x2 x2 x2 x2
b. It can be observed from the table that as the number of days increases by +1,
the number of infected people doubles (x2).

c. The formula that will describe the given situation is y=(2) x

Example 2:
Suppose that the half-life of a substance is 250 years. If there were initially 100g of the
substance, how much will remain after 1000 years? (Hint: Half-life means it is the time it takes
for half of the substance to decay)

100
a) Using table of values, show the relationship of the number of years to decay and amount of
a substance.

b) What pattern can you observe from the table?

c) Derive a formula for the number of years it takes the substance to decay as a function of
the remaining amount of substance.

Solution:
a.

Number of years (x) 0 250 500 750 1000

Amount of substance (y) 100 50 25 12.5 6.25

b. It can be observed from the table that as the number of years increases by 250, half of the
amount of substance decayed.
1
c. The pattern tells us that this situation can be represented by y=100(2)𝑥

Learning Competency
The learner represents real-life situation using exponential functions. (M11GM-Ie-3)

Exercise 1: Match Me
Directions: Match each function found in Column A with its corresponding table of values
found in Column B. Write the letter of the correct answer on the space provided before the
number. [2 points each]
Column A Column B
_____1. y=2(2)x
x 0 1 2 3
a.
y 3 9 27 81

_____2. y= 3(1/3)x x 0 1 2 3
b. y 3 1 1/3 1/9

x 0 1 2 3
x
_____3. y= 2(0.45) c.
y 2 4 6 8

_____4. y=3(3)x d. x 0 1 2 3
y 2 0.9 0.405 0.18225

101
Exercise 2: Represent Me
Directions: Read and analyze the given problems below. Write the exponential model of each
item. [2 points each]
1. The initial enrolment of Luna National High School on the first day of enrolment is 500
and it doubles every 10 days.

2. Freggie borrowed ₱20,000.00 from ASKI lending company that yields an interest rate of
3% compounded annually.

3. At t=0, there were initially 30 bacteria. Suppose that the bacteria triples every 200 hours.

4. The half-life of a radioactive substance is 500 years and its initial amount is 200g.

5. A bacteria culture triples every 5 hours and starts with 5 000 bacteria.

Exercise 3: Solve Me
Directions: Read and analyze the given problems below then answer what is being asked.
Show your solution.
1. A population of Barangay starts with 500 individuals and it triples every 80 years.
a. Give an exponential model for the situation. [2 points]

b. What is the size of the population after 100 years? [3 points]

2. A bank offers ₱10,000 with an interest rate of 2% compounded annually for a certain
fund.
a. Write the exponential model to compute the future amount of the fund.
[2 points]

b. How much is the total savings after 5 years? [3 points]

3. Athena invested ₱50,000.00 at an annual rate of 6% compounded annually.


a. Find the total amount in the account after 10 years if no withdrawals and no
additional deposits are made. [5 points]

102
Exercise 4: Think, Write and Apply…
In this activity, your task is to apply the concept of exponential function by composing
a mathematics jingle about the effect of COVID-19 to our country. Write your jingle in a bond
paper (a4). Use English or Taglish (Tagalog-English) as your medium of writing. Using
Acapela or with accompaniment, perform your newly created jingle and then video yourself
while singing. Upload your video on youtube and send me the link of your video. Be guided
with the following criteria.
RUBRICS FOR SCORING
Category Needs Satisfactory Competent Exemplary
Improvemen (2points) (3points) (4points)
t
(1point)
Grammar There are There are a few There are There are no
several spelling/ hardly any spelling/
spelling/ grammatical spelling/ grammatical
grammatical errors. grammatical errors
errors. errors.
Performance/Audienc Video Video Video Video
e performance performance performance performance
Engagement was lacking was good but was great, was excellent-
and it was practice could obviously obviously
obvious that have been well very well
more practice done. practiced. practiced.
was needed. Audience was Audience was Audience was
Audience was engaged mostly engaged.
minimally somewhat. engaged.
engaged if at
all.
Length The song is The song is The song is The song is
less than 1 between 1:00 between 1:30 2:00 or longer.
minute. and 1:29. and 1:59
Lyrics/Rhythm Some of the Most of the The lyrics of
The lyrics of
lyrics fit with lyrics fit with the song fitthe song
the rhythm of the rhythm of with the
precisely fit
the song. the song. rhythm of the
with the
song. rhythm of the
song.
Information Information Information Information Information
not accurate. provided is provided is provided is
It was difficult somewhat mostly both detailed
to determine accurate. accurate. and accurate.
the Mathematical Mathematical Mathematical
mathematical concept/topic concept/ concept/topic
focus. is somewhat topic is clearly is clearly
communicated communicate communicate
. d and includes d and includes
some detail. every detail

103
and fact
needed to get
the rules of
concept across
to learners.
Vocabulary There is no The The The
vocabulary vocabulary vocabulary vocabulary
used from that was not used was mostly was
topic. correctly or used correctly accurately
there are only or there is used and there
1-2 words only 3-4 are 5 or more
used. words used. words
Lyrics The song does The song at The song The song
not connect times connects most follows a
and does not connects, but it of the song steady stream
follow a does follow a and the steady of thought
steady stream steady stream stream of throughout the
of thought. of thought at thought is not entire story.
times. clear.
Source:https://www.leonschools.net/cms/lib/FL01903265/Centricity/Domain/3141/Math%20song%20project
%208th.pdf

Reflection:
What is the importance of exponential function? Write at least 1-2 paragraph.
___________________________________________________________________
___________________________________________________________________
___________________________________________________________________
___________________________________________________________________
___________________________________________________________________
___________________________________________________________________
___________________________________________________________________

References of Learners
https://www.ck12.org/c/algebra/exponential-growth-and-decay/lesson/Exponential-
Functions/
Verzosa, D.B. , et.al., (2016). General Mathematics for Senior High School (First Edition).
Quezon City Manila; Lexicon Press Inc.

104
Answer Key

Exercise 1:

1. C
2. B
3. D
4. A

Exercise 2:
1. y= 500(2)t/10

2. y= 20 000(1+0.03)t

3. y= 30(3)t/200

4. y= 20(1/2)t/500

5. y= 5000(3)t/5

Exercise 3:

1. a. y=500(3)t/80

b. y=1,974. The barangay has a population of 1,974 after 100 years.

2. a. A=10,000 (1+0.02)t

b. A=Php11,040.81.

3. A=Php 89,542.38. Athena has a total savings of Php 89,542.38 after 10 years

Exercise 4:
(Students have different outputs)

105
GENERAL MATHEMATICS 11
Name of Learner:_____________________________ Grade Level:__________________
Section:_____________________________________ Score: _______________________

LEARNING ACTIVITY SHEET


EXPONENTIAL FUNCTION, EXPONENTIAL EQUATION, AND
EXPONENTIAL INEQUALITY

Background Information for Learners


Exponential expression is a mathematical expression which contains an exponent. This
expression can be written as a•𝑏 𝑥−𝑐 +d, where a, b and d are constant. The value of b must be
greater than 0 but should not be equal to 1 (b>0, b0).
In this learning activity sheet, you will distinguish among exponential function,
exponential equation and exponential inequality
In order to understand our topic, let us analyze the given example below.
Example
Directions: Evaluate the expression in each item below. What are the similarity/ies or
difference/s of the given expressions?
1. 5𝑥+1 = 25 2. y= 2𝑥 3. 16 > 83𝑥−1
Possible Answers: Items 1,2, and 3 involve expressions with a variable in the exponent. Items
1 and 3 involve only one variable but item 2 involves two variables. Items 1 and 2 involve
equality symbol while item 3 involves an inequality symbol.

Points to Ponder:

• Exponential Function- is a function that can be written as f(x)=𝑏 𝑥 or y=bx,


where b>0, b1.
Example: f(x)= 5x-1 or y= 5x-1
• ExponentialEquation- is an equation involving exponential expressions. The
base must be constant and its exponent must contain a variable.
Example: 32x+1=3x-5
• Exponential Inequality- is an inequality that involves exponential
expressions. The base must be constant and its exponent must contain a
variable.
Example: 26x-3+22x+5> 0

Note: The exponential equation and exponential inequality can be solved for all
values of x that will satisfy the equation or inequality while the exponential
function will just show us the relationship of two variables.

106
Solved Examples
Directions: Determine whether the given expression in each item is an exponential equation,
exponential inequality, exponential function or none of the above. Write EE for exponential
equation, EI for exponential inequality, EF for exponential function and NA for none of the
above.
1. y= 4x-1 (Answer: EF)
2. 83x+1= x (Answer: NA since the base on the right expression is a variable)
3. 5> 25x+4 (Answer: EI)
4. 100+ 10x-5=0(Answer: EE)
5. f(x)= 62(x-5) (Answer: EF)

Learning Competency
Distinguishes among exponential function, exponential equation, and exponential
inequality. (M11GM-Ie-4)
Exercise 1:
Directions: Fill in each blank with the appropriate mathematical symbol (=, <, >, ,  ) to
satisfy the given exponential expression in each item. [2 points each]

1. Exponential Equation : 36 x+1 ____ 6 x

2. Exponential Function: y____27𝑥+5


3. Exponential Inequality: 9 ____81𝑥+5
Exercise 2:
Directions: Determine whether the given expression in each item is an exponential equation,
exponential inequality, exponential function or none of the above. Write EE for exponential
equation, EI for exponential inequality, EF for exponential function and NA for none of the
above. Write your answer on the space given before the number.
[2 points each]

_____1. f ( x) = 3 x

_____2. 125 x+1 = 5 x

1 1
_____3. = 5 x (Hint: can be written also as 5−2 )
25 25

_____4. 81  3 x −2

_____5. 2 2− x  8 6 x

107
4−2 X
3
_____6. y=  
5

_____7. f ( x) = 25 2t

f ( x) = 33 x
_____8.
5 x = 1252 x
_____9.

_____10. f ( x) = 1− x

Exercise 3:
Directions: Evaluate the given expressions inside the box below. Copy each expression and
write this on the table according to its name. [2 points each]

1 3𝑥+3
1253x+7 = 25x-3 y=81 72x+4 < 492x-1

12𝑥−10 1 𝑥+13
≤ 62x-4  642x+3 f(x)=23x-2
6 216

16x-5 = 642x-3 y=12x+1 125x-1=144x-6

Exponential Function Exponential Equation Exponential Inequality


1. 1. 1.
2. 2. 2.
3. 3. 3.

Reflection:

What have you learned from this topic?


___________________________________________________________________________
___________________________________________________________________________
___________________________________________________________________________
___________________________________________________________________________

References of Learners:
http://teachtogether.chedk12.com/teaching_guides/view/14
Verzosa, D.B. , et.al., (2016). General Mathematics for Senior High School (First Edition).
Quezon City Manila; Lexicon Press Inc.

108
Answer Key
Exercise 1
1. =
2. =
3. >, <, , or 

Exercise 2
1. EF
2. EE
3. EE
4. EI
5. EI
6. NA
7. EF
8. EF
9. EE
10. NA

Exercise 3
Exponential Function Exponential Equation Exponential Inequality
1 3𝑥+3 1. 125x-1=144x-6 1. 72x+4 < 492x-1
1. y=81
2. 1253x+7 = 25x-3 2. 62x-4  642x+3
2. y=12x+1 3. 16x-5 = 642x-3 12𝑥−10 1 𝑥+13
3. f(x)=23x-2 3. 6 ≤ 216

109
GENERAL MATHEMATICS 11
Name of Learner: ____________________________ Grade Level: _____________
Section:____________________________________ Date: ___________________

LEARNING ACTIVITY SHEET


Representations of Exponential Functions

Background Information for Learners

Exponential functions can be represented through its table of values, graph, and equation.
The data in the table of values of an exponential function has common factors and equal
intervals as shown in the diagram below. This data or values are important in sketching its
graphical representation, which will describe the behavior and characteristics of the
function.
Common interval +1 +1 +1 +1

x -2 -1 0 1 2
y 0.25 0.5 1 2 4

Common factor x 0.5 x 0.5 x 0.5 x 0.5


In constructing table of values, choose appropriate x-values and substitute them to the
equation to get the respective y-values, while in graphing, it is important to plot few points
from the table of values and connect the points using a smooth curve.
Example: Represent the function 𝑦 = 2𝑥−3 through table of values.
If x = -2 If x = -1 If x = 0 If x = 1
𝑥−3 𝑥−3
𝑦=2 𝑦=2 𝑦 = 2𝑥−3 𝑦 = 2𝑥−3
𝑦 = 2−2−3 𝑦 = 2−1−3 𝑦 = 20−3 𝑦 = 21−3
𝑦 = 2−5 𝑦 = 2−4 𝑦 = 2−3 𝑦 = 2−2
1 1 1 1
𝑦 = 32 𝑜𝑟 0.03125 𝑦 = 16 𝑜𝑟 0.0625 𝑦 = 8 𝑜𝑟 0.125 𝑦 = 4 𝑜𝑟 0.25
x -2 -1 0 1
1 1 1 1
y 𝑜𝑟 0.03125 𝑜𝑟 0.0625 𝑜𝑟 0.125 𝑜𝑟 0.25
32 16 8 4

110
Learning Competency with code
The learner is able to represent an exponential function through its table of values, graph, and
equation. M11GM-If-2

Directions
Read and understand the directions in each exercise. If you have any question, feel free to
message your teacher for clarification and assistance.
EXERCISE 1. “IDENTIFY ME!”
Directions: Observe the pattern in each of the table of values. Determine whether each table of
values represents an exponential function or not. Write Exponential Function or Not an
Exponential Function on the spaces provided. [2 points each]
1. _________________________________
X -3 -2 -1 0 1 2 3 4 5
Y 1/243 1/81 1/27 1/9 1/3 1 3 9 27
2. _________________________________
X -3 -2 -1 0 1 2 3 4 5
Y 1/128 1/64 1/32 1/16 1/8 1/4 1/2 1 2
3. _________________________________
X -3 -2 -1 0 1 2 3 4 5
Y -2 0 2 4 6 8 10 12 14
4. _________________________________
X -3 -2 -1 0 1 2 3 4 5
Y 2 8/3 10/3 4 14/3 16/3 6 20/3 22/3
5. _________________________________
X -3 -2 -1 0 1 2 3 4 5
Y 27 9 3 1 1/3 1/9 1/27 1/81 1/243

EXERCISE 2. “OH! MY TABLE OF VALUES”


Directions: Represent the following exponential functions through table of values.
[Total of 10 points, 1 point every correct corresponding value of y]
Note: In completing the table of values, you need to compute the corresponding y-
value by substituting the value of x to the equation and simplifying it. Look at the example
in computing y-values below the table. Input your answer on the table of values. If you
already get the pattern of the y-values, you can put your answer directly in the table of
values.

1. 𝑦 = 22𝑥−5 where 𝑥 = 0,1,2,3,4


X 0 1 2 3 4
Y 0.03125
Example:
if x=0
𝑦 = 22𝑥−5
𝑦 = 22(0)−5
𝑦 = 20−5
𝑦 = 2−5
𝑦 = 0.03125

111
1 𝑥−2
2. 𝑦 = ቀ2ቁ where x =-1, 0, 1, 2, 3, 4, and 5
x -1 0 1 2 3 4 5
y 8
Example:
if x = -1
1 𝑥−2
𝑦=( )
2
1 −1−2
𝑦=( )
2
1 −3
𝑦=( )
2
𝑦 = 23
𝑦=8

EXERCISE 3. “DESCRIBE ME”


Directions: Use the graphs below as your bases in answering the questions.

Note: The bases of each


of the functions defined
by graphs A, C and E
are greater than 1,
while those of graphs B
and D are between 0
and 1, or greater than
zero but less than 1.

1𝑥
A. Graph of 𝑓 (𝑥 ) = 2𝑥 B. Graph of 𝑓 (𝑥 ) = 2

C. Graph of 𝑓 (𝑥 ) = 4𝑥−1 D. Graph of 𝑓 (𝑥 ) = 0.752𝑥−2 E. Graph of 𝑓 (𝑥 ) = 3𝑥+1

In 2-3 sentences, compare the graph of an exponential function with a base greater than 1, and
that of an exponential function whose base is greater than 0, but less than 1 (whose base is
between 0 and 1).
___________________________________________________________________________
___________________________________________________________________________
___________________________________________________________________________
___________________________________________________________________________
EXERCISE 4. “SHOW MY CURVES”
112
Directions: Each of the table of values below represents an exponential function. Sketch the
graph of each function.

1. x -2 -1 0 1 2 3
y 5.62 3.2 1.78 1 0.56 0.32

2. X -1 0 1 2 3 4 5 6
Y 0.0625 0.125 0.25 0.5 1 2 4 8

113
EXERCISE 5. “MY REPRESENTATIONS OF EXPONENTIAL FUNCTIONS”
Directions: Read, analyze and carefully answer each of the following problems.
1.
It is highly believed that the number of COVID-19 cases has been
growing exponentially when there is no public health response
like social distancing and enhanced community quarantine.
Assume that the number of COVID-19 cases doubled every day.
Source:https://www.statnews.
Initially, on Day 0, there was only 1 infected individual. com/2020/02/11/disease-
caused-by-the-novel-
coronavirus-has-name-covid-
19/
a. Represent the exponential model for the number of daily COVID-19 cases or express
the problem in its equation.
b. Make a table of values indicating the data of number of COVID-19 cases from Day 0
to Day 10.
c. Draw a graph representing the number of COVID-19 cases on the Cartesian plane based
on the table of values you have constructed.
2.
Do you know that we can estimate the population of Luna, Isabela?
According to the 2015 Census, as of August 1, 2015, Luna’s population
was 19,326 with a growth rate of 1.27%. Assume that the population
growth of Luna can be estimated by the model:
𝑦 = 𝑦0 𝑒 𝑟𝑡
where: 𝑦0 is the original population at t = 0
𝑟 is the population rate of growth Source:https://ww
w.kindpng.com/im
𝑒 is a transcendental number approximately equal to 2.71828 gv/Joowio_crowd-
clipart-transparent-
𝑦 is the function of time 𝑡. person-
transparent-
background-
population-clipart/
Y0ekt
a.Y=Represent the exponential model for the population of Luna, Isabela on 2025 based on
the data revealed by 2015 Census?
b. Make a table of values indicating the total population of Luna, Isabela from 2015 to
2020.
c. What is the estimated population of Luna, Isabela on 2025?

Rubric for Scoring


For Exercise 3
Elements 5 POINTS 3 POINTS 0 POINT
Student can explain Explanation Explanation Student provides no
their reasoning for provided with provided with explanation.
their ideas logical flow and is logical flow, is
mostly correct. mostly incorrect
Student correctly All properties of are Some properties are Does not give any
give the properties mentioned mentioned, some are properties of an
of an exponential not. exponential function
function
Maximum total points on Exercise 3: 10 points

114
For Exercise 4
4 POINTS 3 POINTS 2 POINTS 1 POINTS 0 POINT
Graph Correctly Correctly Some points Graphs are No
plotted points. plotted points. are correctly incomplete and attempt
plotted demonstrate a
Points are Appropriately lack of
properly Scaled Axis Appropriately understanding
labeled Scaled Axis of how to
construct a
Appropriately graph for
Scaled Axis exponential
functions.
Maximum total points on Exercise 4: 8 points

For Exercise 5
Elements 4 3 2 1 0
Writing Contains all of Contains Contain Functions are No
exponential the elements of most of the some of the written attempt
model the proper elements of elements of incorrectly and
exponential the proper the proper demonstrates a
model exponential exponential lack of
model model. understanding
of how to
exponential
functions.
Tables of All data in the Most of the Most of the None of the No
Values table of values data in the data in the values is correct attempt
are correct. table of table of but presented an
values are values are effort to solve.
correct incorrect
Graph Correctly Correctly Some points Graphs are No
For no. 1 plotted points. plotted are correctly incomplete and attempt
letter c points. plotted demonstrate a
Points are lack of
properly Appropriatel Appropriatel understanding
labeled y Scaled y Scaled of how to
Appropriately Axis Axis construct a
Scaled Axis graph for
exponential
functions.
For no. 2 Correct answer Correct Incorrect Incorrect No
letter c was obtained answer is answer and answer is attempt
and solution is presented but solution is presented. No
presented no solution presented solution
Maximum total points on Exercise 5: 24 points

115
Reflection
What did you learn about the representations of exponential functions?
___________________________________________________________________________
___________________________________________________________________________
___________________________________________________________________________
Assuming that Coronavirus Disease is increasing exponentially. Express your opinion why this
situation is so scary. What do you think are the best remedies to lessen the number COVID-19
cases?
___________________________________________________________________________
___________________________________________________________________________
___________________________________________________________________________

References for learners


Oronce, O. (2018). General Mathematics. Manila, Philippines: Rex Book Store, Inc.
Verzosa, D. et. Al. (2016). General Mathematics. Lexicon Press Inc.
https://study.com/academy/lesson/what-is-an-exponential-function.html
https://www.onlinemath4all.com/how-to-determine-an-exponential-function-from-a-table-of-
values.html
https://www.stat.berkeley.edu/~stark/SticiGui/Text/reasoning.htm

116
Answer Key
EXERCISE 1
1. Exponential Function
2. Exponential Function
3. Not an exponential function
4. Not an exponential function
5. Exponential Function

EXERCISE 2
1. x 0 1 2 3 4
y 1/32 or 0.03 1/8 or 0.13 ½ or 0.5 2 8

2. X -1 0 1 2 3 4 5 6
Y 8 4 2 1 ½ or 0.5 ¼ or 0.25 1/8 or 1/16 or
0.125 0.0625

EXERCISE 3. Answer may vary.


EXERCISE 4
1. 2.

EXERCISE 5.
1. a. 𝑦 = 1(2)𝑥
b.
bNo. of Days (x) 0 1 2 3 4 5 6 7 8 9 10
.
No. of Covid Cases
1 2 4 8 16 32 64 128 256 512 1024
(y)

c.

117
2. a. 𝑦 = 19326(2.71828)0.0127𝑡
b. 2015 2016 2017 2018 2019 2020
t
(0) (1) (2) (3) (4) (5)
y 19326 19573 19824 20077 20334 20593
c. The estimated population of Luna, Isabela on 2025 is 21,944.

118
GENERAL MATHEMATICS 11
Name of Learner:_______________________________________ Grade Level:__________
Section:_______________________________________________ Date:________________

LEARNING ACTIVITY SHEET


DOMAIN AND RANGE OF AN EXPONENTIAL FUNCTION

Background Information for Learners


What is domain and range of a function? The domain of a function is the set of all
possible values that the variable 𝑥 can take, and range is the set of all the values that 𝑓(𝑥) or
𝑦 will take.
The domain of an exponential function with base 𝑏 of the form 𝑓(𝑥 ) = 𝑏 𝑥 or 𝑦 = 𝑏 𝑥
where 𝑏 > 0, 𝑏 ≠ 1 is always the set of all real numbers, while its range is always the set of all
positive real numbers.
Constructing a table of values and sketching a graph of an exponential function can
help in easily determining the domain and range especially for the transformation of an
exponential function which is in the form 𝐹 (𝑥 ) = 𝑎 ∙ 𝑏 𝑥+𝑐 + 𝑑 where 𝑎, 𝑐 and 𝑑 are real
numbers.

Example: Find the domain and range of 𝒇(𝒙) = 𝟐𝒙 .

Graph the function on a Cartesian plane.

Based on the graph, the function is defined for all real numbers. Thus, the domain of the
function 𝑓(𝑥 ) = 2𝑥 is set of real numbers.

As the value of 𝑥 tends to +∞, the value of the function also tends to +∞, and as the value
of 𝑥 tends to −∞, the function approaches the 𝑥-axis but never touches it. Therefore, the
range of the function is set of positive real numbers or {𝑥 ∈ ℝ|𝑥 > 0}.

119
Learning Competency
Finds the domain and range of an exponential function (M11GM-If-3).

Directions/Instructions:
Activity 1. “Take it easy!”
Direction: Determine the domain and range of the following exponential functions. [1 point
each]

1. 𝑓 (𝑥 ) = 3𝑥 5. 𝑦 = 5−𝑥
𝑥
2. 𝑔(𝑥 ) = 10𝑥 1
6. 𝑦 = ቀ10ቁ
3. ℎ(𝑥 ) = 15𝑥
4. 𝑖 (𝑥 ) = 0.50𝑥 7. 𝑦 = −4𝑥
8. 𝑦 = 6𝑥−1

Activity 2. “Find Me! Find Me!”


Direction: Answer the following problems. Show complete solution in a separate sheet of
paper. [1 points each]

1. Give the domain and range of the function 𝑦 = 7𝑥


a. for all values of x
b. for 0 ≤ 𝑥 ≤ 5
1 𝑥
2. Give the domain and range of the function 𝑦 = ቀ2ቁ
a. for all values of x
b. for 5 ≤ 𝑥 < +∞
.
3. Give the domain and range of the function 𝑦 = −5𝑥 + 2
a. for all value of x
b. for 𝑥 > 0

Activity 3. “Stop, Look and Answer!”


Direction: Determine the domain and range of the following graphs of exponential functions.
[2 points each]

1. 2.

120
3. 4.

5.

Activity 4. “Construct, Sketch and State!”


Direction: For each of the following functions, construct a table of values, sketch the graph
and state the domain and range.

1. 𝑓 (𝑥 ) = 5𝑥 1 −𝑥
2. 𝑔(𝑥 ) = ቀ5ቁ

𝑥 𝑥
𝑓(𝑥) 𝑔(𝑥)

Domain:_________________________ Domain:_________________________
Range:__________________________ Range:__________________________

121
3. 𝑦 = 0.253−𝑥 4. ℎ(𝑥 ) = 6𝑥 − 5

𝑥 𝑥
𝑦 ℎ(𝑥)

Domain:_________________________ Domain:_________________________
Range:__________________________ Range:__________________________

5. 𝑑 (𝑥 ) = 4𝑥−2 + 2 1
6. 𝑦 = 2 (2)2−𝑥

𝑥 𝑥
𝑑(𝑥) 𝑦

Domain:_________________________ Domain:_________________________
Range:__________________________ Range:__________________________

122
Rubrics for Scoring the Table of Values and Graph

0 Point 1 Point 2 Points 3 Points


Table of Values Learner gives Learner gives 1 Learner gives 4 Learner gives
no correct to 3 correct to 6 correct all correct
answers. answers. answers. answers.
Graph Learner gives Learner graphs Learner graphs Learner graphs
no graph/graph the function less the function the function
is not accurate. neatly and neatly and very neatly and
organize, and a organized, and organized and
little accurate. accurate. very accurate.

Rubrics in Scoring

0 Point 1 Point 2 Points 3 Points


Amount of Learner gives Learner gives Learner gives Learner gives
Work no answer. answer but no answer with answer with
solution/proof partial complete
of answer. solution/proof solution/proof
of answer. of answer.
Understanding The learner The learner The learner The learner
shows no shows limited shows partial shows thorough
understanding understanding understanding understanding
in solving the in solving the in solving the in solving the
problem. problem. problem. problem.

Reflection

What did you learn about domain and range of exponential functions?
___________________________________________________________________________
___________________________________________________________________________
___________________________________________________________________________
___________________________________________________________________________
___________________________________________________________________________
___________________________________________________________________________

References

General Mathematics Textbook, Chapter 4: Exponential Functions (Teachers Guide),


General Mathematics Textbook, Chapter IV: Exponential Functions (Learner’s Materials),
eSolutions Manual

123
Answer Key

Activity 1.
1. Domain: Set of all real numbers
Range: Set of all positive real numbers
2. Domain: Set of all real numbers
Range: Set of all positive real numbers
3. Domain: Set of all real numbers
Range: Set of all positive real numbers
4. Domain: Set of all real numbers
Range: Set of all positive real numbers
5. Domain: Set of all real numbers
Range: Set of all positive real numbers
6. Domain: Set of all real numbers
Range: Set of all positive real numbers
7. Domain: Set of all real numbers
Range: Set of all negative real numbers
8. Domain: Set of all real numbers
Range: {𝑦|𝑦 > 2} or (2, +∞)

Activity 2.
1.
a. Domain: Set of all real numbers
Range: Set of all positive real numbers
b. Domain: {𝑥|0 ≤ 𝑥 ≤ 10} or [0,10]
Range: {𝑦|1 ≤ 𝑦 ≤ 16807} or [1,16807]

2.
a. Domain: Set of all real numbers
Range: Set of all positive real numbers
b. Domain: {𝑥 |5 ≤ 𝑥 < +∞} or [5, +∞]
1 1
Range: {𝑦|0 < 𝑥 ≤ 32} or (0, 32]

3.
a. Domain: Set of all real numbers
Range: {𝑥| − ∞ < 𝑥 ≤ 2} or (−∞, 2]
b. Domain: {𝑥 |𝑥 > 0} or (0, +∞)
Range: {𝑦|𝑦 < 0} or (−∞, 0)
Activity 3.
1. Domain: Set of all real numbers
Range: {𝑦|𝑦 > −3} or (−3, +∞)
2. Domain: Set of all real numbers
Range: {𝑦|𝑦 < 10} or (−∞, 10)
3. Domain: Set of all real numbers
Range: Set of all positive real numbers
4. Domain: Set of all real numbers
Range: {𝑦|𝑦 > −25} or (−25, +∞)
5. Domain: Set of all real numbers
Range: {𝑦|𝑦 < −2} or (−∞, −2)Activity 4.

124
1. 𝑓 (𝑥 ) = 5𝑥

a. Table of Values
𝒙 −3 −2 −1 0 1 2 3
1 1 1
𝑓 (𝑥 ) = 5 𝑥 1 5 25 125
125 25 5

b. Graph

c. Domain and Range

Domain: Set of all real numbers (ℝ)


Range: Set of all positive real numbers

1 −𝑥
2. 𝑔(𝑥 ) = ቀ5ቁ

a. Table of Values
𝒙 −3 −2 −1 0 1 2 3
−𝑥
1 1 1 1
𝑔 (𝑥 ) = ( ) 1 5 25 125
5 125 25 5

b. Graph

c. Domain and Range

Domain: Set of all real numbers (ℝ)


Range: Set of all positive real numbers

3. 𝑦 = 0.253−𝑥

125
a. Table of Values
𝒙 −3 −2 −1 0 1 2 3
1 1 1 1 1 1
𝑦 = 0.253−𝑥 1
4096 1024 256 64 16 4

b. Graph

c. Domain and Range

Domain: Set of all real numbers (ℝ)


Range: Set of all positive real numbers

4. ℎ(𝑥 ) = 6𝑥 − 5

a. Table of Values
𝒙 −3 −2 −1 0 1 2 3
1079 179 29
ℎ (𝑥 ) = 6 𝑥 − 5 − − − −4 1 31 211
216 36 6

b. Graph

c. Domain and Range

Domain: Set of all real numbers (ℝ)


Range: 𝑦 > −5 or (−5, +∞)

5. 𝑑 (𝑥 ) = 4𝑥−2 + 2

a. Table of Values
𝒙 −3 −2 −1 0 1 2 3
2049 513 129 33 9
𝑑 (𝑥 ) = 4𝑥−2 + 2 3 6
1024 256 64 16 4

126
b. Graph

c. Domain and Range

Domain: Set of all real numbers (ℝ)


Range: 𝑦 > 2 or (2, +∞)
1
6. 𝑦 = (2)2−𝑥
2

a. Table of Values
𝒙 −3 −2 −1 0 1 2 3
1 1 1
𝑦 = (2)2−𝑥 16 8 4 2 1
2 2 4

b. Graph

c. Domain and Range

Domain: Set of all real numbers (ℝ)


Range: Set of all positive real numbers

127
GENERAL MATHEMATICS 11
Name of Learner:_______________________________________ Grade Level:__________
Section:_______________________________________________ Date:________________

LEARNING ACTIVITY SHEET


INTERCEPTS, ZEROES AND ASYMPTOTES OF AN EXPONENTIAL
FUNCTION

Background Information for Learners


What are intercepts, zeroes and asymptotes of a function? Intercepts are the points at
which a graph intersects with either the x or y axis. The x-intercepts or zeroes of a function are
the values of 𝑥 which make the function zero, while the y-intercepts of a function are the
function values when 𝑥 = 0. Asymptotes are lines that continually approach a given curve but
do not meet it at any finite distance. The vertical line 𝑥 = 𝑎 is a vertical asymptote of a function
𝑓 if the graph of 𝑓 either increases or decreases without bound as the x-values approach 𝑎 from
the right or left. The horizontal line 𝑦 = 𝑏 is a horizontal asymptote of the function 𝑓 if 𝑓(𝑥)
gets closer to 𝑏 as 𝑥 increases or decreases without bound.
The y-intercept of an exponential function with base 𝑏 of the form 𝑓 (𝑥 ) = 𝑏 𝑥 or 𝑦 =
𝑏 𝑥 is always 1. The horizontal asymptote is the line 𝑦 = 0. There are no x-intercept, zeroes
and vertical asymptote.

Example: Determine the horizontal asymptote and y-intercept of 𝒇(𝒙) = 𝟒𝒙.

Graph the exponential function in a Cartesian plane.

Since y-intercept is the function


value when 𝑥 = 0, therefore the
y-intercept is 1.

It can be observed that the function is defined for all values of x, is strictly increasing, and
attains only positive y-values. As x decreases without bound, the function approaches 0 but
never meet it, thus, the line 𝑦 = 0 is a horizontal asymptote.

128
Learning Competency
Determines the intercepts, zeroes, and asymptotes of an exponential function
(M11GM-If-4).

Directions/Instructions
Activity 1. “Stop, Look and Answer!”
Direction: Determine the 𝑦-intercept and horizontal asymptote of the following graphs of
exponential functions. [1 point each]

1 𝑥
1. 𝑓 (𝑥 ) = 2𝑥 2. 𝑓 (𝑥 ) = ቀ5ቁ

3. 𝑓 (𝑥 ) = −3𝑥 4. 𝑓 (𝑥 ) = 2𝑥 + 6

5. 𝑓 (𝑥 ) = 3−𝑥+1

129
Activity 2. “Construct, Sketch and State!”
Direction: In each of the following functions, construct a table of values using 𝑥 =
−3, −2, −1, 0, 1, 2, 3, sketch the graph on a Cartesian plane, and state the y-intercept and
horizontal asymptote.

1. 𝑓 (𝑥 ) = 8𝑥 2. 𝑔(𝑥 ) = 3𝑥+2

𝑥 𝑥
𝑓(𝑥) 𝑔(𝑥)

y-intercept:_______________________ y-intercept:_______________________
Horizontal Asymptote:______________ Horizontal Asymptote:______________

3 𝑥 4. 𝑦 = 7(5)𝑥−1 + 1
3. 𝑦 = − ቀ4ቁ

𝑥 𝑥
𝑦 𝑦

y-intercept:_______________________ y-intercept:_______________________
Horizontal Asymptote:______________ Horizontal Asymptote:______________

130
𝑥
5. 𝑑 (𝑥 ) = 0.45(7)𝑥 1
6. ℎ(𝑥 ) = 5 ቀ10ቁ + 3

𝑥 𝑥
𝑑(𝑥) ℎ(𝑥)

y-intercept:_______________________ y-intercept:_______________________
Horizontal Asymptote:______________ Horizontal Asymptote:______________

Rubrics for Scoring the Table of Values and Graph

0 Point 1 Point 2 Points 3 Points


Table of Values Learner gives Learner gives 1 Learner gives 4 Learner gives
no correct to 3 correct to 6 correct all correct
answers. answers. answers. answers.
Graph Learner gives Learner graphs Learner graphs Learner graphs
no graph/graph the function less the function the function
is not accurate. neatly and neatly and very neatly and
organize, and a organized, and organized and
little accurate. accurate. very accurate.

131
Activity 3. “Match Me!”
Direction: Match the following function with their corresponding y-intercept located in the y-
intercept Column, and with their corresponding horizontal asymptote located in the Horizontal
Asymptote Column. Draw a line in matching. Note: any of the choices can be used as an
answer multiple times.

Horizontal
y-intercept
Asymptote
26 Function 𝑦=1

38880 1. 𝑦 = 12−𝑥 𝑦 = 10

1 2. 𝑦 = 1.3(7)𝑥 − 1 𝑦 = −5
13
3. 𝑦 = −7−𝑥+2 + 4 𝑦=4
10
1 𝑥−5
45 4. 𝑦 = 5 ቀ6ቁ 𝑦=0

−45 5. 𝑦 = −0.80𝑥 − 10 𝑦 = −1

−11 6. 𝑦 = 24−𝑥 + 10 𝑦 = 1.3


3
𝑦 = −10
10

Activity 4. “Solve Me!”


Direction: Solve the following problems. Show complete solution in separate sheet of paper.

In items 1-2, (a) use transformations to describe how the graph is related to its base
exponential function 𝑦 = 𝑏 𝑥 , (b) sketch the graph, (c) identify its y-intercept and horizontal
asymptote.

1 𝑥−1
1. 𝑓 (𝑥 ) = ቀ8ቁ +4 2. 𝑔(𝑥 ) = 3(2)𝑥−1

3. Find an exponential function of the form 𝑓 (𝑥 ) = 𝑎(𝑏)𝑥 + 𝑑 such that the y-intercept is
−4, the horizontal asymptote is 𝑦 = −8, and 𝑓(3) = 24. Show solution.

4. Find an exponential function of the form 𝑓 (𝑥 ) = 𝑏 𝑥−𝑐 + 𝑑 such that the y-intercept is 29,
the horizontal asymptote is 𝑦 = 4, and 𝑓 (−1) = 9. Show solution.

5. A cup of hot chocolate is ordered from a coffee shop. The temperature 𝑦(𝑡) of the hot
chocolate 𝑡 minutes after receiving it is given by 𝑦(𝑡) = 70(1.071)−𝑡 + 15 degree Celsius.
Determine the y-intercept and horizontal asymptote of the exponential model.

132
Rubrics for Scoring the Transformation and Graph in Items 1 and 2.

0 Point 1 Point 2 Points 3 Points


Transformation Learner shows Learner shows Learner shows Learner shows
no limited partial thorough
understanding understanding understanding understanding
of using of using of using of using
transformations transformations transformations transformations
to describe how to describe how to describe how to describe how
the graph is the graph is the graph is the graph is
related to its related to its related to its related to its
base base base base
exponential exponential exponential exponential
function 𝑦 = function 𝑦 = function 𝑦 = function 𝑦 =
𝑏𝑥 . 𝑏𝑥 . 𝑏𝑥 . 𝑏𝑥 .
Graph Learner gives Learner graphs Learner graphs Learner graphs
no graph/graph the function less the function the function
is not accurate. neatly and neatly and very neatly and
organize, and a organized, and organized and
little accurate. accurate. very accurate.

Rubrics in Scoring Items 3-5.

0 Point 1 Point 2 Points 3 Points


Amount of Learner gives Learner gives Learner gives Learner gives
Work no answer. answer but no answer with answer with
solution. partial solution. complete
solution.
Understanding The learner The learner The learner The learner
shows no shows limited shows partial shows thorough
understanding understanding understanding understanding
in solving the in solving the in solving the in solving the
problem. problem. problem. problem.

Reflection

What did you learn about asymptotes, intercepts and zeroes of exponential functions?
___________________________________________________________________________
___________________________________________________________________________
___________________________________________________________________________
___________________________________________________________________________
___________________________________________________________________________

References

General Mathematics Textbook, Chapter 4: Exponential Functions (Teachers Guide), General


Mathematics Textbook, Chapter IV: Exponential Functions (Learner’s Materials), eSolutions
Manual
133
Answer Key:

Activity 1.

1. y-intercept: 1
Horizontal Asymptote: 𝒚 = 𝟎
2. y-intercept: 1
Horizontal Asymptote: 𝒚 = 𝟎
3. y-intercept: -1
Horizontal Asymptote: 𝒚 = 𝟎
4. y-intercept: 7
Horizontal Asymptote: 𝒚 = 𝟔
5. y-intercept: 3
Horizontal Asymptote: 𝒚 = 𝟎

Activity 2.

1. 𝑓 (𝑥 ) = 8𝑥
a. Table of Values
𝒙 −3 −2 −1 0 1 2 3
1 1 1
𝒇(𝒙) = 𝟖𝒙 1 8 64 512
512 64 8
b. Graph

c. Intercepts and Asymptotes


x-intercept: no x-intercept
y-intercept: 1
Horizontal Asymptote: 𝑦 = 0
Vertical Asymptote: no vertical asymptote

2. 𝑔(𝑥 ) = 3𝑥+2

a. Table of Values
𝒙 −3 −2 −1 0 1 2 3
1
𝒈(𝒙) = 𝟑𝒙+𝟐 1 3 9 27 81 243
3
b. Graph

134
c. Intercepts and Asymptotes
x-intercept: no x-intercept
y-intercept: 9
Horizontal Asymptote: 𝑦 = 0
Vertical Asymptote: no vertical asymptote

3 𝑥
3. 𝑦 = − ቀ4ቁ
a. Table of Values
𝒙 −3 −2 −1 0 1 2 3
3 𝑥 64 16 4 3 9 27
𝑦 = −( ) − − − −1 − − −
4 27 9 3 4 16 64
b. Graph

c. Intercepts and Asymptotes


x-intercept: no x-intercept
y-intercept: −1
Horizontal Asymptote: 𝑦 = 0
Vertical Asymptote: no vertical asymptote

4. 𝑦 = 7(5)𝑥−1 + 1
a. Table of Values
𝒙 −3 −2 −1 0 1 2 3
632 132 32 12
𝑦 = 7(5)𝑥−1 + 1 8 36 176
625 125 25 5

b. Graph

135
c. Intercepts and Asymptotes
x-intercept: no x-intercept
12
y-intercept: 5
Horizontal Asymptote: 𝑦 = 1
Vertical Asymptote: no vertical asymptote

5. 𝑑 (𝑥 ) = 0.45(7)𝑥
a. Table of Values
𝒙 −3 −2 −1 0 1 2 3
9 9 9 9 63 441 3087
𝑑(𝑥 ) = 0.45(7)𝑥
6860 980 140 20 20 20 20
b. Graph

c. Intercepts and Asymptotes


x-intercept: no x-intercept
9
y-intercept: 20
Horizontal Asymptote: 𝑦 = 0
Vertical Asymptote: no vertical asymptote

1 𝑥
6. ℎ(𝑥 ) = 5 ቀ10ቁ + 3
a. Table of Values
𝒙 −3 −2 −1 0 1 2 3
𝑥
1 7 61 601
ℎ (𝑥 ) = 5 ( ) + 3 5003 503 53 8
10 2 20 200

b. Graph

136
c. Intercepts and Asymptotes
x-intercept: no x-intercept
y-intercept: 8
Horizontal Asymptote: 𝑦 = 3
Vertical Asymptote: no vertical asymptote

Activity 3.

Activity 4.

1 𝑥
1. Transformation: The base function 𝐹 (𝑥 ) = ቀ8ቁ will be shifted 1 unit to the
right, then it will be shifted 4 units up.
Graph:

y-intercept: 12
Horizontal Asymptote: 𝑦 = 4

2. Transformation: The base function 𝐺 (𝑥 ) = 2𝑥 will be stretched 3 units (that is


every y-value will be multiplied by 3), then it will be shifted 1 unit to the right.

Graph:

137
3
y-intercept: 2
Horizontal Asymptote: 𝑦 = 0

3. 𝑓 (𝑥 ) = 4(2)𝑥 − 8
4. 𝑓 (𝑥 ) = 5𝑥+2 + 4
5. y-intercept: 85
Horizontal asymptote: 𝑦 = 15

138
GENERAL MATHEMATICS 11
Name of Learner:________________________ Grade Level: _________________________
Section:________________________________ Date: _______________________________

Learning Activity Sheet


Representing Real-Life Situations using Logarithms

Background Information for Learners

This activity sheet was designed to help learners understand the lesson on how to
represent real-life situations using logarithmic functions. This would also enhance the learners’
ability to solve. .

Logarithm is defined as c=logba if and only if a=bc where a,b and c are positive real
numbers such that b≠1. Meanwhile, common logarithm is a logarithm with base 10 and it is
written as log x or log10x. On the other hand, natural logarithm is a logarithm to the base e
(approximately 2.71828) and is denoted by ln x or logex. Consider the examples below.

Example 1: Rewrite the following exponential equations in logarithmic form.


a. 53=125
This can be written as log 5 125=3 following the definition of logarithm
b. 104=10,000
Since a= 10,000, b=10 and c=4, then its logarithmic form is log 10 10,000=4 or
log 10,000=4 because it is a common logarithm

Example 2: Rewrite the following logarithmic equations in exponential form.


a. log 3 81=4
Its equivalent exponential form is 34=81 since a=81, b=3 and c=4
b. ln 10=x
The given is an example of natural logarithm. Hence its exponential form is e x=10.

There are common applications of logarithms in real-life. These are Richter scale,
sound intensity and pH level. The formula of the magnitude (R) of an earthquake on a Richter
2 𝐸
scale is given by 𝑅 = 3 log 104.40 where E is the energy released by the earthquake usually in
joules while 104.40 joules is the energy released by a very small reference earthquake.
𝐼
Meanwhile, the decibel level of sound is given by the formula, 𝐷 = 10 log 10−12 where I is the
sound intensity in watts/m2 and the quantity 10-12 watts/m2 is the least audible sound a human
can hear. For the pH level of a water-based solution, it is given by pH=-log[H+] where H+ is
the concentration of hydrogen ions in moles per liter. Solutions with pH level of 7 are
categorized as neutral, while those solutions with pH level less than 7 are acidic and those
solutions with greater than 7 pH level are basic. Study the given examples.

139
Example 3: Suppose that an earthquake released approximately 108 joules of energy. What is
its magnitude on a Richter scale? How much more energy does this earthquake release than the
reference earthquake?
Solution:
Since the given is a sample problem of solving the magnitude of an earthquake in a
2 𝐸
Richter scale, then the formula to be used is 𝑅 = 3 log 104.40. The value of E is given which is
equal to 108 joules. Therefore, the formula become𝑠 𝑅 =
2 108 2 2 2
log 104.40 = 3 log 108−4.40 = 3 log 103.6 = 3 (3.6) ≈ 2.4. Thus, the magnitude of the
3
earthquake is approximately equal to 2.4
To compare the energy released by the given earthquake with the energy released by
108
the reference earthquake, just divide the two energies. That is, 104.40 = 108−4.40 = 103.6 ≈
3,981. This means that the earthquake releases approximately equal to 3,981 times more
energy than the reference earthquake.

Example 4: The decibel level of sound in an office is 10−10 watts/m2. What is the
corresponding sound intensity in decibels? How much more intense is this sound than the least
audible sound a human can hear?

Solution:
𝐼
The formula to be used is 𝐷 = 10 log 10−12 . Since I is given which is 10−10 watts/m2,
10−10
then the working formula is 𝐷 = 10 log 10−12 = 10 log 10−10+12 = 10 log 102 = 10(2) =
20. This means that the corresponding intensity is 20 decibels.
10−10
To compare the intensity of the two sounds, simplify 10−12 . This will yield to 10−10+12=
102 = 100 which means that the sound is 100 times more intense than the least audible sound
a human can hear.

Example 5: A 1-liter solution contains 0.0000001 moles of hydrogen ion. Find its ph level.

Solution:
Since there are 0.0000001 moles of hydrogen ion, then its concentration is 10−7 moles
per liter. Thus, the ph level is -log10−7 =-(-7)=7.

Learning Competency:
The learners represent real-life situations using logarithmic functions (M11GM-Ih-1)

Activity 1- Write What?


Directions: Read the following problems. Do as indicated. Write the answer before the item.

1. Rewrite the following exponential equations in logarithmic form whenever possible


a. 43=64
1
b. 8-2=64
1 1
c. (4)2=16
d. (-3)2=9
e. e3=x
2. Rewrite the following logarithmic equations in exponential form
a. log c =d
140
b. log 3 81=4
1
c. log1/2 32=5
d. log 0.01=-2
e. ln 4=x

Activity 2- How are Logarithms Useful?


Directions: Read, analyze and solve the given problems. Show your solutions.

1. An earthquake has released 1015 joules of energy. Find its magnitude on a Richter scale?

Solution:

2. In a certain room, its decibel level of sound is 10-8 watts/m2. What is its corresponding
sound intensity in decibels?

Solution:

3. A solution contains 10-4 moles of hydrogen ions. Describe its pH level.

Solution:

4. What is the magnitude of an earthquake on a Richter scale that released 10,000,000,000


joules of energy?

Solution:

5. Find the pH level of a solution that contains 0.0000001 moles of hydrogen ions.

Solution:

Activity 3- Fact or Bluff


Directions: Identify whether the following statements is correct or incorrect. Write Fact if it is
correct and Bluff if it is incorrect. Explain your answer.

1. The magnitude in the Richter scale of a certain earthquake that released 10 9 joules of
energy is approximately equal to 3.1.

Answer:

2. In a 1-liter solution containing 10-9 moles of hydrogen ions, its pH level is 9. Hence,
the solution is acidic.

Answer:

3. The corresponding sound intensity in decibels of a jet during takeoff is 140 decibels.
This means that the intensity of sound of a jet during takeoff is 1000 watts/m 2.
141
Answer:
4. A 1-liter solution becomes neutral when it contains 10 -7 moles of hydrogen ions.
Answer:

5. An earthquake that released an approximately 39,811 times more energy than the
reference earthquake has 1010 joules of energy.

Answer:

Rubrics for Scoring

Activity 1- Write What?

One point is given to the learner when he/she gets a correct answer

Activity 2- How are Logarithms Useful?


3 points 2 points 1 point 0 point
Amount of Learner Learner presented Learner Learner
work presented a an incomplete presented an presented
complete solution but incomplete nothing
solution with simplified answer solution
simplified
answer
Understanding Learner shows Learner shows a Learner shows a Learner shows
a thorough partial limited no
understanding understanding on understanding understanding
on the the applications on the on the
applications of of logarithm in applications of applications of
logarithm in real-life logarithm in logarithm in
real-life real-life real-life

Reflection
Construct at least three (3) sentences on the importance of logarithm in real-life.

References
Barnett, R.A, Ziegler, M.R., Byleen, K.E., &Sobecki,D.(2008).Precalculus.New York:
McGraw-Hill
Young, C. (2012). College Algebra (3rd ed). Hoboken, NJ:John Wiley & Sons
Stewart, J., Redlin, L., & Watson, S. (2012). Pre-calculus: Mathematics for Calculus (6th
ed).Belmont, CA: Brooks/Cole, Cengage Learning
General Mathematics Learner’s Material

142
Answer Key
Activity 1- Write What?
1.a. log464=3
1
b. log8 64=-2
1
c. log1/4 16=2
d. cannot be written in logarithmic form because b<1
e. loge x=3 or ln x=3
2.a.10d=c
b. 34=81
1 1
c. (2)5=32
d. 10-2=0.01
e. ex=4

Activity 2- How are Logarithms Useful?


2
1. 𝑅 = 3 (10.6) ≈ 7.1
2. D=10(4)=40 decibels
3. pH= -(-4)=4→acidic
2
4. 𝑅 = (5.6) ≈ 3.7
3
5. pH= -(-7)=7

Activity 3- Fact or Bluff


2
1. Fact because 𝑅 = 3 (4.6) ≈ 3.1
2. Bluff because 9>7, hence, it is basic
3. Bluff because 140≠ 10(15)
4. Fact because 7=neutral
1010
5. Bluff because 104.40 ≠104.6 or 39,811

143
GENERAL MATHEMATICS 11
Name of Learner:________________________ Grade Level: _________________________
Section:________________________________ Date: _______________________________

Learning Activity Sheet


Logarithmic Function, Logarithmic Equation and Logarithmic Inequalities

Background Information for Learners


This activity sheet is intended for the enrichment of problem solving skills of learners.
This would also aid them understand the differences of logarithmic function, logarithmic
equation and logarithmic inequality.

Logarithmic equation is defined as an equation involving logarithms while logarithmic


inequality is an inequality involving logarithms. On the other hand, logarithmic function is an
equation which expresses a relationship between two variables such as x and y. Examples of
logarithmic equation, logarithmic inequality and logarithmic function are log x 25=2, log x 25<2
and log 5 𝑥 respectively.

Logarithms have basic properties. These are:


1. logb1 = 0
2. logb bx = x
3. 𝑏𝑙𝑜𝑔𝑏 𝑥 = 𝑥 if x>0
Note: The values of b and x are real numbers such that b>0 and b≠1.

To illustrate the different properties, consider the examples below.

Example 1: Use the basic properties of logarithms to find the value of the following logarithmic
expressions
a. log 102
Solution: log 102 =log10 102 =2 (Property 2)
b. log 1
Solution: log 1 =log10 1=0 (Property 1)
c. 6log6 2
Solution: 6log6 2 = 2 (Property 3)

Below are laws of logarithms.

Product Law

logb( uv)= logbu+ logbv where b>0, b≠1; u and v are positive real numbers

144
Quotient Law

logb( u/v)= logbu- logbv where b>0, b≠1; u and v are positive real numbers

Power Law

logbun= nlogbu where b>0, b≠1; n is any real number

Below are examples on how to illustrate the different laws of logarithms. Study them
carefully.

Example 2: Use the laws of logarithms to expand each expression in terms of the logarithms
of the factors. Assume each factor is positive.
a. log(𝑟𝑠 3)
Solution: log(𝑟𝑠 3) = log 𝑟 + log 𝑠 3 (Product law)
= log 𝑟 + 3 log 𝑠 (Power law)
4 5
b. log 2 ( 𝑦)
4 4
Solution: log 2 ( )5 = 5 log 2 ( ) (Power law)
𝑦 𝑦
= 5(log 2 4 − log 4 𝑦) (Quotient law)
= 5 (2 − log 4 𝑦)
4 5
log 2 ( 𝑦) = 10 − 5 log 4 𝑦

Example 3: Use the laws of logarithm to condense the expressions as a single logarithm.
a. 2 log 𝑥 + log 𝑦
Solution: 2 log 𝑥 + log 𝑦 = log 𝑥 2 𝑦 (Power law & Product law)
b. 7 log 3 𝑎 − 7 log 3 𝑏
Solution: 7 log 3 𝑎 − 7 log 3 𝑏 = (log 3 𝑎 − log 3 𝑏)7 (Power law)
= (log 3 𝑎 /𝑏)7 (Quotient law)
𝑙𝑜𝑔 𝑥
Change-of-base formula is also used in logarithm and it is given by 𝑙𝑜𝑔𝑏 𝑥 = 𝑙𝑜𝑔𝑎 𝑏
𝑎
where a, b and x are positive real numbers with a≠1 and b≠1. Consider the given examples.

Example 4: Use the change-of-base formula to rewrite the following logarithmic expressions
to the indicated base.
a. log 4 8 (change to base 2)
𝑙𝑜𝑔 8 3
Solution: 𝑙𝑜𝑔4 8 = 𝑙𝑜𝑔2 4 = 2
2
b. log 3 2 (change to base e)
ln 2
Solution: 𝑙𝑜𝑔3 2 = ln 3

Learning Competency
The learners distinguish logarithmic function, logarithmic equation, and logarithmic
inequality (M11GM-Ih-2)

145
Activity 1- Log the Appropriate log
Directions: Determine whether the given is a logarithmic equation, a logarithmic inequality, a
logarithmic function or neither. Write your answer before the number.

6. h(x) = log 𝑥
7. log 3 (𝑥 − 2) < 0
8. 𝑦 = 3 log 5 𝑥
9. log(𝑥 + 5) = log 10
10. log1 𝑥 < 2
11. ln 𝑥 2 = (ln 𝑥)2
12. g(x) = 2 log 0 𝑥
13. log 3 (𝑥 − 2) ≥ log 3 𝑥 + 5
14. y = log1 𝑥
15. log 𝑒 𝑥=3

Activity 2- Log the log value


Directions: Use the basic properties of logarithm to find the value of x of the following
logarithms. Write your answer before the number.

1. log 8 8 = 𝑥
2. log 103 = 𝑥
3. x = log 3 27
4. 𝑒 ln 4 = 𝑥
5. log 5 1 = 𝑥
1
6. log 9 81 = 𝑥
7. ln 𝑒 2 = 𝑥
8. 0 = log 𝑥
9. 5 = log 𝑥 32
10. log 𝑒 𝑥 = 1

Activity 3- Expand, Condense and Rewrite the Log


Directions: Read and answer each problem. Show your solutions.

1. Using the properties and laws of logarithm, expand the following expressions.
a. log [(∜x)(y2)]
Solution:

b. log2(4/y)3
Solution:

2. Condense the following as a single logarithm. Use the properties and laws of logarithm.
a. ½ log3 c - 4log3d
Solution:

146
1
b. 3
ln(x+1) +2 ln x
Solution:

3. Rewrite the given expressions to the indicated base. Use the change-of-base formula.
1
a. log (change to base 5)
2
Solution:

b. log 9 27 (change to base 3)


Solution:

Rubrics for Scoring


Activity 1 & 2

One point is given to the learner when he/she gets a correct answer

Activity 3
3 points 2 points 1 point 0 point
Amount of Learner Learner presented Learner Learner
work presented a an incomplete presented an presented
complete solution but incomplete nothing
solution with simplified answer solution
simplified
answer
Understanding Learner shows Learner shows a Learner shows a Learner shows
a thorough partial limited no
understanding understanding in understanding in understanding
in expanding expanding and expanding and in expanding
and condensing condensing condensing and condensing
logarithms as logarithms as well logarithms as logarithms as
well as in using as in using the well as in using well as in using
the change-of- change-of-base of the change-of- the change-of-
base of formula formula base of formula base of formula

Reflection
Identify the common error/s you committed when you differentiated logarithmic
equation, logarithmic inequality and logarithmic function.

References
Crauder, B., Evans, B., & Noell, A. (2008). Functions and Change: A Modeling Approach to
College Algebra and Trigonometry. Boston:Houghton Miffin
General Mathematics Learner’s Material. First Edition 2016

147
Answer Key
Activity 1- Log the Appropriate Log
6. logarithmic function
7. logarithmic inequality
8. logarithmic function
9. logarithmic equation
10. neither
11. logarithmic equation
12. neither
13. logarithmic inequality
14. neither
15. logarithmic equation

Activity 2- Log the Log Value


1. 1
2. 3
3. 3
4. 4
5. 0
6. -2
7. 2
8. 1
9. 2
10. e or 2.72

Activity 3- Expand, Condense and Rewrite the Log


1
1.a. 4 log x + 2log y
b. 6-log 2y
√c
2.a. log 3 d4
3
b. ln x2 √x + 1
0
3.a.log 2
5
3
b. 2

148
GENERAL MATHEMATICS 11
Name of Learner:________________________ Grade Level: _________________________
Section:________________________________ Score: ______________________________

Learning Activity Sheet


Solving Logarithmic Equations & Logarithmic Inequalities

Background Information for Learners


This activity sheet was crafted to serve as a learning guide for learners. It aims to
improve the manipulative skills of learners specifically their skills in solving logarithmic
equations and logarithmic inequalities.

In solving logarithmic equations and logarithmic inequalities, their properties should


be taken into consideration.

Property of Logarithmic Equations


If b>1, then the logarithmic function y = log 𝑏 𝑥 is increasing for all x. If 0 < b < 1, then the
logarithmic function y = log 𝑏 𝑥 is decreasing for all x. This means that log 𝑏 𝑢 = log 𝑏 𝑣 if
and only if u=v.

Property of Logarithmic Inequalities


If 0 < b < 1, then 𝑥1 < 𝑥2 if and only if log 𝑏 𝑥1 > log 𝑏 𝑥2
If b > 1, then 𝑥1 < 𝑥2 if and only if log 𝑏 𝑥1 < log 𝑏 𝑥2

There are also techniques in solving logarithmic equations and logarithmic inequalities.

Techniques in Solving Logarithmic Equations


➢ Rewriting the logarithmic equation to exponential form
➢ Using of logarithmic laws and properties
➢ Applying the one-to-one property of logarithmic functions
➢ Using the zero factor property

Techniques in Solving Logarithmic Inequalities


➢ Ensure that the logarithms are defined
➢ Make sure that the inequality is satisfied

Examine the given examples.

Example 1: Find the value of x in the following equations


a. log 8 4𝑥= log 8 24
Solution: log 8 4𝑥= log 8 24
4x = 24 (one-to-one property)
x=6
b. log 2 (𝑥 − 3) = 4
Solution: log 2 (𝑥 − 3) = 4

149
𝑥 − 3 = 24 (Changing into exponential form)
𝑥 − 3 = 16
𝑥 = 19

Example 2: Solve the following logarithmic inequalities


a. log 4 (𝑥 − 2)> log 4 8
Solution: log 4 (𝑥 − 2)> log 4 8
𝑥−2 > 8 (Since b > 1, then 𝑥1 < 𝑥2 iff log 𝑏 𝑥1 <
log 𝑏 𝑥2 )
𝑥 > 10
Hence, the solution is (10,+∞)

b. log 1 (3𝑥 + 4) ≥ 4
2
Solution: log 1 (3𝑥 + 4) ≥ 4
2
1
log 1 (3𝑥 + 4) ≥ 4 log 1 2 (Ensure that the logarithms are defined)
2 2
1
log 1 (3𝑥 + 4) ≥ log 1 ( 2)4
2 2
1
log 1 (3𝑥 + 4) ≥ log 1
2 2 16
1
3𝑥 + 4 ≤ 16 (since 0<b<1, then 𝑥1 < 𝑥2 iff log 𝑏 𝑥1 >
log 𝑏 𝑥2 )
−63
3𝑥 ≤ 16
21
𝑥 ≤ − 16
21
Thus, the solution is (−∞, - 16 ]

Example 3: A culture starts at 1,500 bacteria and doubles every 70 minutes. How long will it
take the number of bacteria to reach 6,000?
𝑡
Solution: A model for this situation is 𝑦 = 1,500 (2)70 , y is the number of bacteria at
time t.
𝑡
6,000 = 1,500 (2)70
𝑡
4 = (2)70
𝑡
𝑙𝑛 4 = 𝑙𝑛(2)70
𝑡
𝑙𝑛 4 = 𝑙𝑛(2)
70
ln 4 𝑡
=
ln 2 70
ln 4
70 ln 2 = 𝑡
70(2) = 𝑡
140 = 𝑡
Therefore, it will take 140 minutes for the bacteria to reach 6,000.

Learning Competency:
The learners solve logarithmic equations and inequalities (M11GM-Ih-3)

150
Activity 1- Think and Strategize
Directions: Solve the following logarithmic equations. Specify the technique/s used.

16. log 3 4x = log 3 24

Solution:

Technique/s:

17. log x49=2

Solution:

Technique/s:

18. log 2(8x)-log 2(x-5)=4

Solution:

Technique/s:

19. (log4 x)2 + 3 log 4 x +2=0

Solution:

Technique/s:

Activity 2- Think, Think, Think Before You Solve


Directions: Solve the following logarithmic inequalities and write the solution in interval
notation. Indicate the property used.

1. log 4 (3x+2) < log 4 (x-4)

Property:

Solution:

2. log2/3(3x+1) > log 2/3(5x+7)

Property:

Solution:

3. 2 ≥ log 5(2x-3)

151
Property:

Solution:

4. -2≤ log x< 2

Property:

Solution:

Activity 3- Read, Analyze and Solve


Directions: Read, analyze and solve the following problems. Show you solutions.

1. Covid-19 pandemic has created apprehension to everyone. Many are hoping for the
discovery of vaccine in order for this pandemic to come to an end. But since vaccine is
not yet discovered, some avid individuals in social media immediately believe posted
treatment of Covid-19 just to ease their apprehension. Suppose the equation of fake
1
news on the treatment of Covid-19 pandemic is p(t)= 1+15𝑒−0.4𝑡 , where p(t) is the
proportion of the population who has believed of the fake news on the treatment of
Covid-19 at time t days. How long will it take to reach the 90% of the population?

Solution:

2. Assume that the culture of Covid-19 starts at 3,000 virus and triples every 60 minutes.
How long will it take for the number of virus to reach 15,000?

Solution:

3. Due to Covid-19 pandemic, many Filipinos are forced to lend money from institutions
or individuals to suffice their basic needs. Suppose a lending institution uses the
formula, A=P(1+r)n where A is the future value of the investment, P is the principal, r
is the fixed annual interest rate and n is the number of years, how many years will it
take a lender to pay his/her tripled debt if he/she borrowed P15,000 at a rate of 2.5%
per annum?

Solution:

152
Rubrics for Scoring

Activity 1- Think And Strategize


3 points 2 points 1 point 0 point
Understanding Learner shows Learner shows a Learner shows a Learner shows
a thorough partial limited no
understanding understanding on understanding understanding
on how to solve how to solve on how to solve on how to
logarithmic logarithmic logarithmic solve
equations equations equations logarithmic
equations
Quantity of Learner listed Learner listed Learner listed Learner did not
listed all the possible some of the few possible write any
techniques techniques in possible techniques in technique
solving a techniques in solving a
logarithmic solving a logarithmic
equation logarithmic equation
equation

Activity 2- Think, Think, Think Before you Solve


3 points 2 points 1 point 0 point
Understanding Learner shows Learner shows a Learner shows a Learner shows
a thorough partial limited no
understanding understanding on understanding understanding
on how to solve how to solve on how to solve on how to
logarithmic logarithmic logarithmic solve
inequalities inequalities inequalities logarithmic
inequalities
Additional 1 point will be given to the learner if he/she indicated the correct property.

Reflection
Share your feelings while doing the activity.

References
Sharon L. Senk et al. UCSMP Advanced Algebra. Second Edition.(2002). Pearson Education
Inc.
General Mathematics Learner’s Material. First Edition 2016

153
Answer Key
Activity 1- Think And Strategize
1. x=6; one-to-one property
2. x=7, -7 is not a solution; changing into exponential form & factorization/extracting
the roots
𝑢
3. x=10; using the law, logb 𝑣 =logbu-log bv, & changing into exponential form
1 1
4. x=16 and x=4 ; factorization (using the zero factor property) & changing into
exponential form

Activity 2- Think, Think, Think Before you Solve


1. Since b>1, then x1<x2 if and only if log b x1< log b x2; x<-3, hence its solution is (-∞, -
3)
2. Since 0<b<1, then x1<x2 if and only if log b x1> log b x2; x>−3, hence its solution is (−3,
+∞)
3. Since b>1, then x1<x2 if and only if log b x1< log b x2; x≤14, hence its solution is (-∞,
14]
1
4. Since b>1, then x1<x2 if and only if log b x1< log b x2; 100 < 𝑥 < 100, hence its solution
1
is [100 , 100]

Activity 3- Read, Analyze and Solve


1. t ≈ 12.26
It will take approximately 13 days for the fake news on the treatment of Covid-19 to
reach the 90% of the population
2. t ≈ 87.90
It will take approximately 88 minutes or 1 hour & 28 minutes for the virus to reach
15,000
3. n ≈ 44.49
It will take approximately 45 years to pay the tripled debt.

154
GENERAL MATHEMATICS 11

Name: ________________________________ Grade Level: _______


Date: _________________________________ Score: _____________

LEARNING ACTIVITY SHEET


GRAPHING LOGARITHMIC FUNCTIONS

Background Information for Learners

Consider the exponential function represented by its exponential equation 𝑦 = 𝑏 𝑥 . If x and y


are interchanged, the result is 𝑥 = 𝑏 𝑦 . Expressing the exponential equation to its equivalent
logarithmic equation, the result is log 𝑏 𝑥 = 𝑦 or 𝑦 = log 𝑏 𝑥. This is the inverse function of the
exponential function or the logarithmic function.
Sketch the graphs of 𝑦 = 2𝑥 and 𝑦 = log 2 𝑥 in one Cartesian coordinate plane.
Construct a table of values of ordered pairs for the given functions.
1 1
• Since 2- 4 = , then log 2 ( ) = −4
16 16
1 1
• Since 2- 3 = , then log 2 ( ) = −3
8 8
1 1
• Since 2- 2 = , then log 2( ) = −2 and so on…
4 4

𝒚 = 𝟐𝒙
4
x -4 -3 -2 -1 0 1 2 3
1 1 1 1
y 1 2 4 8 16
16 8 4 2

𝒚 = 𝐥𝐨𝐠𝟐 𝒙
1 1 1 1
x 1 2 4 8 16
16 8 4 2

y -4 -3 -2 -1 0 1 2 3 4

155
The graphs are shown below.
𝑦 = 2𝑥 y=x

𝑦 = log 2 𝑥

The graphs affirm that the exponential function 𝑦 = 2𝑥 and logarithmic function
𝑦 = log 2 𝑥 are inverses of each other. Their graphs are reflections of each other on the line y =
x.
The graph of the logarithmic function 𝑦 = log 2 𝑥 exhibits the following properties:
• The domain is the set of all positive numbers, or { 𝑥 ∈ ℝ/𝑥 > 0}.
• The range is the set of all positive real numbers or { 𝑦/𝑦 ∈ ℝ}
These properties do not hold only for 𝑦 = log 2 𝑥 but also for other logarithmic functions of the
form 𝑦 = log 𝑏 𝑥, where b≠1.
Graphs of logarithms are used when you want to compress large scale data. Consider for
instance that the scale of the graph below ranges from 1000 to 100 000 on the y-axis and 1 to
100 on the x-axis—such large scales which can be typical in scientific data are often more
easily represented logarithmically.

Example 1
Sketch the graph of 𝑦 = 2log 2 𝑥. Determine the domain and range.
The graph of 𝑦 = 2log 2 𝑥 can be obtained from the graph of 𝑦 = log 2 𝑥 by multiplying each
y-coordinate by 2, as the following table shows.
1 1 1 1
x 1 2 4 8 16
16 8 4 2
4
𝒚 = 𝐥𝐨𝐠𝟐 𝒙 -4 -3 -2 -1 0 1 2 3
8
𝒚 = 𝟐𝐥𝐨𝐠𝟐 𝒙 -8 -6 -4 -2 0 2 4 6

156
The graphs are shown on the right.

𝑦 = 2log 2 𝑥
𝑦 = log 2 𝑥

Analysis:
1. Domain: { 𝑥 ∈ ℝ/𝑥 > 0}
2. Range: { 𝑦/𝑦 ∈ ℝ}

Example 2

Sketch the graph of 𝑦 = log 2 (𝑥 + 1). Determine the domain and range.
Construct a table of values for x and y.
7 3 1
x − − − 0 1 3 7
8 4 2
3
𝒚 = 𝐥𝐨𝐠𝟐 (𝒙 + 𝟏) -3 -2 -1 0 1 2
The graph is shown below.

𝑦 =
log 2 (𝑥 + 1)

Analysis:
1. Domain: { 𝑥 ∈ ℝ/ 𝑥 > −1}
2. Range: { 𝑦/𝑦 ∈ ℝ}

157
Learning Competency with code
The learner is able to represent a logarithmic function through its; (a) table of values, (b) graph
and (c) equation and find the domain and range of a logarithmic function (M11GM-Ii-2-
3_Quarter 1)

Directions/Instructions:

A. Construct a table of values for each of the functions and sketch the graph in one Cartesian
coordinate plane.
a. 𝒚 = 𝐥𝐨𝐠𝟐 𝒙 b. 𝒚 = 𝐥𝐨𝐠𝟑 𝒙 c. 𝒚 = 𝐥𝐨𝐠𝟒 𝒙 d. 𝒚 = 𝐥𝐨𝐠 𝟓 𝒙

B. Analyze each of the following functions by using the transformations to describe how the
graph is related to a logarithmic function 𝑦 = log 𝑏 𝑥 and identify the domain and range. Sketch
the graph of the function.
Function Domain Range Graph
1. 𝒚 = 𝐥𝐧 𝒙

2. 𝒚 = 𝐥𝐨𝐠 𝟏 (𝒙) − 𝟑
𝟐

3. 𝒚 = 𝐥𝐨𝐠𝟓 (𝒙 − 𝟏) + 𝟑

𝟒. 𝒚 = −𝟐 𝐥𝐨𝐠𝟑 (𝟑 − 𝒙)

5. 𝒚 = 𝐥𝐨𝐠 𝟐 |𝒙|

158
C. Match each graph to its equation by applying the transformations involving logarithmic
functions.
a. b. c.

d. e.

___ 1. 𝒚 = 𝟒 𝐥𝐨𝐠𝟐 𝒙 ___ 2. 𝒚 = 𝐥𝐨𝐠𝟐 ( 𝒙 − 𝟑)


___ 3. 𝒚 = 𝟑 + 𝐥𝐨𝐠 𝟐 𝒙 ___ 4. 𝒚 = 𝟐 − 𝐥𝐨𝐠 𝟐 𝒙
___ 5. 𝒚 = 𝐥𝐨𝐠𝟐 (𝒙 + 𝟒) − 𝟏
Reflection
Complete this statement:
What I learned in this activity
___________________________________________________________________________
___________________________________________________________________________
___________________________________________________________________________
___________________________________________________________________________
____________

References:
Department of Education General Mathematics (Teacher’s Guide).2016
Orines, Fernando B. Next Century Mathematics (General Mathematics).Phoenix
Publishing House, Inc.2016
Oronce, Orlando A. General Mathematics. Rex Book Store.2016
http://www.google.com/url?sa=t&source=web&rct=j&url=https://www.srsd119.ca/wp-
content/uploads/2016

159
Answer Key
A. a.
1 1 1
x 1 2 4 8
8 4 2
3
𝒚 = 𝐥𝐨𝐠 𝟐 𝒙 -3 -2 -1 0 1 2
b.
1 1 1
x 1 3 9 27
27 9 3
3
𝒚 = 𝐥𝐨𝐠 𝟑 𝒙 -3 -2 -1 0 1 2
c.
1 1 1
x 1 4 16 64
64 16 4
3
𝒚 = 𝐥𝐨𝐠 𝟒 𝒙 -3 -2 -1 0 1 2
d.
1 1 1
x 1 5 25 125
125 25 5
3
𝒚 = 𝐥𝐨𝐠 𝟓 𝒙 -3 -2 -1 0 1 2

𝑎. 𝑦 = log 2 𝑥

b. 𝑦 = log 3 𝑥
𝑐. 𝑦 = log 4 𝑥
𝑑. 𝑦 = log 5 𝑥

B.
Function Domain Range Graph

160
1. 𝑦 = ln 𝑥 {𝑥 ∈ ℝ/𝑥 > 0} { 𝑦/𝑦 ∈ ℝ}

2. 𝑦 = log 1 (𝑥) − 3 {𝑥 ∈ ℝ/𝑥 > 0} { 𝑦/𝑦 ∈ ℝ}


2

3. 𝑦 = log 5 (𝑥 − 1) + 3 {𝑥 ∈ ℝ/𝑥 > 1} { 𝑦/𝑦 ∈ ℝ}

4. 𝑦 = −2 log 3 (3 − 𝑥) {𝑥 ∈ ℝ/𝑥 < 3} { 𝑦/𝑦 ∈ ℝ}

161
5. 𝑦 = log 2 |𝑥 | {𝑥 ∈ ℝ/𝑥 ≠ 0} { 𝑦/𝑦 ∈ ℝ}

C. 1. b
2. a
3. e
4. c
5. d

162
GENERAL MATHEMATICS 11
Name of Learner:________________________ Grade Level: ________________________
Section:________________________________ Date: ______________________________

Learning Activity Sheet


Intercepts, Zeroes and Asymptotes of Logarithmic Functions

Background Information for Learners


This activity sheet was designed to enrich the understanding of learners on the
intercepts, zeroes and asymptotes of logarithmic functions. In addition, this would support
learners to become independent problem solvers.

The graph of a logarithmic function has different features. These are intercepts,
asymptotes and zeroes. The identification of these features can be done through construction
of its table of values and sketching its graph. In addition, the properties of logarithmic functions
should be taken into consideration.

Properties of Logarithmic Functions


➢ It is a one-to-one function. It satisfies the horizontal line test
➢ The x-intercept is 1. There is no y-intercept
➢ The vertical asymptote is the line x=0. There is no horizontal asymptote.

Study the given example.

a. Example 1. Sketch the graph of 𝑦 = 𝑙𝑜𝑔4 𝑥. Determine its intercepts, asymptotes


and zeroes.

Solution:
x 1/16 1/4 1 4 16
y -2 -1 0 1 2

163
x- y-intercept vertical Horizontal zeroes
intercept asymptote asymptote
1 none x=0 none 1

There are also guidelines for graphing transformations of logarithmic functions.

Graph of 𝑓 (𝑥 ) = 𝑎 ∙ 𝑙𝑜𝑔𝑏 (𝑥 − 𝑐 ) + 𝑑

➢ The value of b (either b >1 or 0 < b < 1) determines whether the graph is increasing
or decreasing
➢ The value of a determines the stretch or shrinking of the graph. Further, if a is
negative, there is a reflection of the graph about the x-axis
➢ Based on 𝑦 = 𝑎 ∙ 𝑙𝑜𝑔𝑏 𝑥, the vertical shifts is d units up (if d > 0) or d units down (if
d < 0), and the horizontal shift is c units to the right (if c > 0) or c units to the left ( if
c < 0)

Consider the example below.

Example 2: Analyze the function, 𝑔(𝑥) = 𝑙𝑜𝑔4 (𝑥 − 3) using the transformations to describe
how the graph is related to 𝑓(𝑥) = 𝑙𝑜𝑔4 𝑥. Identify also the intercepts, asymptotes and zeroes.

Solution:
The graph of 𝑔(𝑥 ) = 𝑙𝑜𝑔4 (𝑥 − 3)is shifted 3 units to the right from the graph of 𝑓(𝑥) = 𝑙𝑜𝑔4 𝑥

x-intercept y-intercept vertical Horizontal asymptote zeroes


asymptote
4 none x=3 none 4

Learning Competency: The learners determine the intercepts, zeroes and asymptotes of
logarithmic functions (M11GM-Ii-4)

164
Activity 1- Observe and Learn
Directions: Complete and observe the table of values of the given exponential and logarithmic
functions. Answer what is being asked.

y=2x
X -2 -1 0 1 2

y=log2x
x 1/4 1/2 1 2 4
y

20. What can you say about the given table of values?

21. Do you think the intercepts, zeroes and asymptotes of the given logarithmic function
can be found in its table of values? How?

22. What are the intercepts, zeroes and asymptotes of the given logarithmic function?

23. Can you draw conclusions in determining the intercepts, zeroes and asymptotes of a
logarithmic function based on the given? Cite at least one.

Activity 2- Graph to a Right Path

165
Directions: Sketch the graph of the following logarithmic functions. Determine their
intercepts, zero/es and asymptotes.

1. f(x)= log3x

x-intercept/s:
y-intercept/s:
horizontal asymptote/s:
vertical asymptote/s:
zero/es:

2. y=3log2x

x-intercept/s:
y-intercept/s:
horizontal asymptote/s:
vertical asymptote/s:
zero/es:

Activity 3- The Graphing Transformation


Directions: Sketch the graph of the following functions in one (1) cartesian plane. Analyze the
graph of each function by observing the transformations to describe how the graph is related to
y = log 2 𝑥. Identify the intercepts, asymptotes and zeroes of each function.

f(x)= log2x
g(x)= 3log2x
h(x)=log2(x-1)
166
j(x)= log2(x)+4

Observations:

Function x- y-intercept/s horizontal vertical Zero/es


intercept/s asymptote/s asymptote/s
f(x)= log2x

g(x)= 3log2x

h(x)=log2(x-1)

j(x)=log2 (x)+4

Rubrics for Scoring


Activity 1-Observe and Learn
3 points 2 points 1 point 0 point
Understanding Learner shows Learner shows a Learner shows a Learner shows
a thorough partial limited no
understanding understanding on understanding understanding
on how to solve how to solve on how to solve on how to
problems using problems using problems using solve problems
logarithm logarithm logarithm using
logarithm
Entries in the Learner Learner Learner Learner failed
table of values completed the completed the completed the to complete the
table of values table of values but table of values table of values
with correct some entries were but many entries
entries incorrect were incorrect

167
Activity 2-Observe and Learn
3 points 2 points 1 point 0 point
Amount of Learner Learner sketched Learner sketched Learner failed
Work sketched both both graphs at least 1 graph to sketch any
graphs accurately with graph
accurately with no label
proper label
Comprehension Learner Learner Learner identified Learner failed
identified all identified at least at least half of the to identify the
the features of half of the features of any of features of the
both features of both the given given
logarithmic logarithmic logarithmic logarithmic
functions functions functions functions

Activity 3-The Graphing Transformation


4 points 3 points 2 points 1 point 0 point
Amount of Learner Learner Learner Learner Learner
Work sketched all sketched 2-3 sketched 2-3 sketched at failed to
graphs graphs graphs least 1 graph sketch any
accurately accurately accurately graph
with proper with proper with no label
label label
Understandin Learner Learner Learner Learner Learner
g identified all identified all identified at identified at failed to
the features the features least half of least half of the identify the
of all of at least the features features of at features of
logarithmic half of the of all least half of the the given
functions logarithmic logarithmic logarithmic logarithmic
functions functions functions functions
Quantity of Learner Learner Learner Learner listed Learner
Observation listed all listed 2 listed 1 irrelevant failed to list
relevant relevant relevant observations his/her
observations observations observation observations

Reflection
Share your technique/s in doing the given task.

References
Sharon L. Senk et al. UCSMP Advanced Algebra. Second Edition.(2002). Pearson Education
Inc
General Mathematics Learner’s Material. First Edition 2016

168
Answer Key

Activity 1-Observe and Learn

y=2x
x -2 -1 0 1 2
y 1/4 1/2 1 2 4

y=log2x
x 1/4 1/2 1 2 4
y -2 -1 0 1 2

1. Exponential and logarithmic functions are inverses with each other


2. Yes because the table of values is a mechanism for sketching the graph of a function
which in return the identification of its features
3. x-intercept: 1 horizontal asymptote: none zero/es: 1
y-intercept: none vertical asymptote: x=0
4. A logarithmic function has no y-intercept and horizontal asymptote
The x-intercept, vertical asymptote and zeroes are present in a logarithmic function

Activity 2-Observe and Learn


1.

x-intercept/s: 1
y-intercept/s: none
horizontal asymptote/s: none
vertical asymptote/s: x=0
zero/es: 1

2.

169
x-intercept/s: 1
y-intercept/s: none
horizontal asymptote/s: none
vertical asymptote/s: x=0
zero/es: 1

Activity 3- The Graphing Transformation

170
Observations
➢ The graph of h(x) is the graph of f(x) shifted 1 unit to the right
➢ The graph of j(x) is a vertical shift of 4 units upward from the graph of f(x)

Function x- y-intercept/s horizontal vertical zero/es


intercept/s asymptote/s asymptote/s
f(x)= log2x 1 none none x=0 1

g(x)= 3log2x 1 none none x=0 1

h(x)=log2(x-1) 2 none none x=1 2

j(x)=log2 (x)+4 1/16 none none x=0 1/16

171
GENERAL MATHEMATICS 11
Name: _____________________ Grade Level: ____
Date: ______________________ Score: _________

Learning Activity Sheet


LOGARITHMIC FUNCTION

Background Information for Learners

In your previous lessons, you have studied about logarithmic functions, equations and
inequalities. Today you will encounter some applications of these in real life situation.
Problems involving compound interest are some of the many applications of logarithms.

Before the age of electronic calculators and digital computers, logarithms were used
widely for difficult computations like finding the products, quotients, or powers of numbers
represented by complicated decimal numerals.

In this activity sheet you will be able to learn about solving problems involving
logarithmic functions, equations and inequalities. I believe this is not new to you since you
already have background on functions, equations and inequalities plus you have also the ability
to solve problems. Remember problems solved using exponential functions are computed more
easily by logarithms.

This is a self-paced material for grade 11 where students can check and recheck their
understanding and progress about the topic. It is an enjoyable material where ‘learning is fun’
can be experienced.

Let’s get started!

Example 1: To find x in 3x = 8
log 3x = log 8
x log 3 = log 8
x = log 8
log 3
= 0.9031
0.4771
so x = 1.89

Easy, right? It is just an application of the different properties of logarithm.

Example 2: Find log4(x+1) < log42x

Ensure first that the logarithms are defined, this means x+1 > 0 and 2x > 0, which
implies, x > - 1 and x > 0, or just simply x > -1.

log4 (x+1) < log4 2x


172
x + 1 < 2x
x – 2x < - 1
-x < - 1
x>1
Therefore, the solution is (1, +)

Example 3: To find the new principal after 8 years on an investment of P200.00 earning 8%
interest compounded semi annually, use the formula P = P0 (1 + r/2) 2t

Given P = 200 (1 + 0.08/2)16 plugged-in the given in the problem to the formula

then, log P = log [200 x (1.04)16]


= log 200 + 16 log 1.04
= 2.3010 + 0.2720
= 2.5730
Therefore, P = 374

The new principal after 8 years would be Php 374, to nearest peso.
Example 4: A particular virus grows according to the formula A = A0ekt, where A is the
population of the virus after time t, and A0 is the initial population at t = 0. Suppose there were
2000 viruses at the start of the experiment. After 3 hours, there were already 320 more that
three times the initial number of viruses present. Determine the constant k. Express to the
nearest hundredths.

Solution: After 3 hours, there were already 320 more than 3 times the initial number
That is, 3 ( 2000) + 320 = 6320. Use this value to solve for k.
6320 = 2000e3k
e3k = 3.16
ln e3k = ln 3.16
3k = ln 3.16
k = ln 3.16
3
k = 0.38

Thus, the rate of growth of the virus is 0.38.

So, are you ready to take the activity?

Learning Competency 1: The learner solves problems involving logarithmic


functions, equations and inequalities. (M11GM-Ij-2)

173
Activity 1: Take it Easy ☺
Directions: Find the value/s of x in the following equations.
1. log 2x = log 6
2. logx 121 = 2
3. log3 (x + 4) = log3 (2x – 4)
4. log x2 = 2
5. log (3x - 2) = log 2

Activity 2: “Ensure it’s defined!”


Directions: Find the solution in the following inequalities.
1. log8 (3x - 5) < 2
2. logx-2 (10- +3x) < 2
3. log x (x3 –x2 – 2x) < 3
4. log5 (3x - 1) < 1
5. log4 x + 8  11

Activity 3: “Solve me”


Directions: Solve the following problems.

1. If the interest were added yearly to the amount invested at 12%, every peso would
grow to (1.12)n in n years. Find the amount to which P250.00 increases in 10 years
if invested under the same conditions.

2. If the number (N), in thousands, of bacteria in a culture is given by the equation N


= 3 x 8t where t is measured in hours. After how many hours will the number of
bacteria be 100 thousands?

3. The approximate population of a certain city in the Philippines was 460,000 in1970.
In 1980, it was 630,000. Estimate the population this 2020.

4. In 2005, it was estimated that for the succeeding 20 years the population of a
particular town was expected to be f(t) people t years from 2005, where f(t) = C *
102t , and C and k are constants. If the actual population in 2005 was 1000 and in
2010 was 4000, what is the expected population this 2020?

5. A radioactive substance is decaying according to the formula y = Ae kx, where x is


the time in years. The initial amount A = 10 grams, and 8 grams remain after 5
years. Estimate the amount remaining after 10 years.

174
Reflection
Evaluate your understanding about solving logarithmic functions, equations and
inequalities. Which is easy? Difficult? Why?

Answer:
___________________________________________________________________________
___________________________________________________________
References:

Conceptual Math and Beyond General Mathematics Philippine Copyright 2016 ISBN 978-
621-8006-33-1

DIWA Senior High School Series Philippine Copyright 2016 ISBN 978-971-46-0782-8

General Mathematics LM, 2016 Functions for High School ISBN 971-101-050-X

175
Answer Key

ACTIVITY 1
1. 2.58
2. 11
3. 8
4. -10, 10
5. 3/4

ACTIVITY 2
1. (5/3, 23)
2. (-2,3) U (3, 10/3)
3. 3. (2, ∞)
4. (1/3, 2)
5. (-∞, 64)

ACTIVITY 3
1. 0.005
2. 1.69
3. 2,211,058
4. 64,000
5. 6.40 grams

176

You might also like